Você está na página 1de 208

PREFÁCIO

O presente material tem como objetivo, apresentar aos alunos de


Licenciatura em Matemática, os conteúdos do primeiro ano do en-
sino médio, cubrindo o programa da disciplina: Matemática para o
Ensino Médio I. O programa desta dsiciplina tem como tema cen-
tral as funções reais de uma variável real, estudadas de um ponto
de vista mais elementar, que não leva em conta o Cálculo Inni-
tesimal.
O presente texto, cobre todo o programa da disciplina e tem como
principal referência bibliográca, o texto de Elon Lages Lima com
colaboração de outros autores, intitulado: A matemática do Ensino
Médio I, o qual é parte de um projeto da Sociedade Brasileira de
Matemática, que visa oferecer ao professor maior apoio bibliográ-
co.
Os conteúdos são tratados de uma maneira mais formal e em al-
guns casos, de maneira inédita, se comparado a forma que apare-
cem nos livros didáticos escolares, os quais pouco diferem um dos
outros. Procuraremos mostrar, seguindo o modelo proposto pelo
autor citado, que as funções ans, quadráticas, exponenciais, lo-
garítmicas e trigonométricas, cada uma delas é estudada como o
modelo matemático adequado para representar uma situação es-
pecíca. A m de saber qual o tipo de função que se deve empregar
ao resolver um problema, é necessário comparar as característi-
cas desse problema com as características da função que se tem
em mente. Assim se torna necessário conhecer os teoremas de ca-
racterização de cada uma destas funções. Os teoremas e suas de-
monstrações são postos de forma elementar e não exigem do aluno
grande esforço para seu entendimento. Possivelmente, quando em
sala de aula, no ensino médio, estes teoremas (na sua maioria)
não serão repassados aos alunos a menos que o professor entenda
que seja possível fazê-lo.
Também, são apresentadas aplicações para estas funções, com o
intuito a despertar o interesse e exibir a eciência e utilidade dos
métodos matemáticos apresentados.

Aracaju, março de 2010.


Sumário

Aula 1: Produto Cartesiano e o plano R2 13


1.1 Introdução . . . . . . . . . . . . . . . . . . . . . . . 14
1.2 Produto Cartesiano . . . . . . . . . . . . . . . . . . 14
1.3 O plano Numérico R2 . . . . . . . . . . . . . . . . . 17
RESUMO . . . . . . . . . . . . . . . . . . . . . . . . . 21
REFERÊNCIAS BIBLIOGRÁFICAS . . . . . . . 21

Aula 2: Função Am 23


2.1 Introdução . . . . . . . . . . . . . . . . . . . . . . . 24
2.2 Função Afim . . . . . . . . . . . . . . . . . . . . . 24
2.3 Função Linear . . . . . . . . . . . . . . . . . . . . . 28
2.4 Caracterização da Função Afim . . . . . . . . . . . . 33
2.5 Conclusão . . . . . . . . . . . . . . . . . . . . . . . 36
RESUMO . . . . . . . . . . . . . . . . . . . . . . . . . 37
ATIVIDADES . . . . . . . . . . . . . . . . . . . . . . 38
REFERÊNCIAS BILBIOGRÁFICAS . . . . . . . 40

Aula 3: Funções Quadráticas 41


3.1 Introdução . . . . . . . . . . . . . . . . . . . . . . . 42
3.2 Função Quadrática . . . . . . . . . . . . . . . . . . 42
3.2.1 Um problema Antigo . . . . . . . . . . . . . 45

7
RESUMO . . . . . . . . . . . . . . . . . . . . . . . . . 47
ATIVIDADES . . . . . . . . . . . . . . . . . . . . . . 47
REFERÊNCIAS BIBLIOGRÁFICAS . . . . . . . 48

Aula 4: Gráco da Função Quadrática 49


4.1 Introdução . . . . . . . . . . . . . . . . . . . . . . . 50
4.2 Forma Canônica do Trinômio . . . . . . . . . . . . 50
4.3 Gráfico da Função Quadrática . . . . . . . . . . . . 53
4.4 Aplicações . . . . . . . . . . . . . . . . . . . . . . . 59
4.4.1 Uma Propriedade da Parábola . . . . . . . . 60
4.4.2 O movimento uniformemente variado . . . . 64
RESUMO . . . . . . . . . . . . . . . . . . . . . . . . . 66
ATIVIDADES . . . . . . . . . . . . . . . . . . . . . . 67
REFERÊNCIAS BIBLIOGRÁFICAS . . . . . . . 71

Aula 5: Caracterização das


Funções Quadráticas 73
5.1 Introdução . . . . . . . . . . . . . . . . . . . . . . . 74
5.2 Teoremas de Caracaterização das funções quadráticas 74

5.3 Conclusão . . . . . . . . . . . . . . . . . . . . . . . 79
RESUMO . . . . . . . . . . . . . . . . . . . . . . . . . 79
REFERÊNCIAS BIBLIOGRÁFICAS . . . . . . . 79

Aula 6: Funções Polinomiais 81


6.1 Introdução . . . . . . . . . . . . . . . . . . . . . . . 82
6.2 Funções Polinomiais e Polinômios . . . . . . . . . . 82
6.3 Determinando um Polinômio a partir de seus Valores 85
6.4 Gráfico de Polinômios . . . . . . . . . . . . . . . . . 86
RESUMO . . . . . . . . . . . . . . . . . . . . . . . . . 89
ATIVIDADES . . . . . . . . . . . . . . . . . . . . . . 90
REFERÊNCIAS BIBLIOGRÁFICA . . . . . . . . 91

Aula 7: Funções Logarítmicas 93


7.1 Introdução . . . . . . . . . . . . . . . . . . . . . . . 94
7.2 Definição e Propriedades da Função Logarítmica . . 95
RESUMO . . . . . . . . . . . . . . . . . . . . . . . . . 103
REFERÊNCIAS BIBLIOGRÁFICAS . . . . . . . 104

Aula 8: Função Logarítmo Natural


e Área de Faixas de Hipérboles 105
8.1 Introdução . . . . . . . . . . . . . . . . . . . . . . . 106
8.2 Área de uma Faixa de Hipérbole . . . . . . . . . . . 106
8.3 Propriedade Fundamental . . . . . . . . . . . . . . . 110
8.4 Função Logarítmo Natural . . . . . . . . . . . . . . 113
8.4.1 Gráfico da função ln . . . . . . . . . . . . . 115
RESUMO . . . . . . . . . . . . . . . . . . . . . . . . . 115
ATIVIDADES . . . . . . . . . . . . . . . . . . . . . . 116
REFERÊNCIAS BIBLIOGRÁFICAS . . . . . . . 118

Aula 9: Funções Logarítmicas:


Uma Abordagem Geométrica 119
9.1 Introdução . . . . . . . . . . . . . . . . . . . . . . . 120
9.2 Funções Logarítmicas: Uma abordagem Geométrica 120
RESUMO . . . . . . . . . . . . . . . . . . . . . . . . . 123
ATIVIDADES . . . . . . . . . . . . . . . . . . . . . . 124
REFERÊNCIAS BIBLIOGRÁFICAS . . . . . . . 125

Aula 10: Funções Exponenciais 127


10.1 Introdução . . . . . . . . . . . . . . . . . . . . . . . 128
10.2 Potências de Expoente Racional . . . . . . . . . . . 128

10.2.1 Gráfico da Função exponencial . . . . . . . . 136

10.3 Funções Exponenciais × Funções Logarítmicas . . . 137

10.4 Função Exponencial na base e . . . . . . . . . . . . 138

RESUMO . . . . . . . . . . . . . . . . . . . . . . . . . 142

ATIVIDADES . . . . . . . . . . . . . . . . . . . . . . 143

REFERÊNCIAS BIBLIOGRÁFICAS . . . . . . . 144

Aula 11: Caracterização das funções


Exponenciais e Logarítmicas 145
11.1 Introdução . . . . . . . . . . . . . . . . . . . . . . . 146

11.2 Caracterização das Funções Exponênciais . . . . . . 146

11.2.1 Aplicações . . . . . . . . . . . . . . . . . . . 150

11.3 Caracterização das funções Logarítmicas . . . . . . 152

11.4 Conclusão . . . . . . . . . . . . . . . . . . . . . . . 154

RESUMO . . . . . . . . . . . . . . . . . . . . . . . . . 154

ATIVIDADES . . . . . . . . . . . . . . . . . . . . . . 156

REFERÊNCIAS BIBLIOGRÁFICAS . . . . . . . 156

Aula 12: Trigonometria:


Noções Elementares 157
12.1 Introdução . . . . . . . . . . . . . . . . . . . . . . . 158

12.2 Trigonometria do Triângulo Retângulo . . . . . . . . 158

12.3 Função de Euler e Medida de Ângulos . . . . . . . . 160

RESUMO . . . . . . . . . . . . . . . . . . . . . . . . . 165

ATIVIDADES . . . . . . . . . . . . . . . . . . . . . . 166

REFERÊNCIAS BIBLIOGRÁFICAS . . . . . . . 167


Aula 13: Funções Trigonométricas 169
13.1 Introdução . . . . . . . . . . . . . . . . . . . . . . . 170
13.2 Função Seno e Cosseno . . . . . . . . . . . . . . . 170
13.2.1 Propriedades . . . . . . . . . . . . . . . . . 171
13.3 Função Tangente . . . . . . . . . . . . . . . . . . . 174
13.3.1 Propriedades da função Tangente . . . . . . 175
13.4 Função Cotangente . . . . . . . . . . . . . . . . . . 177
13.4.1 Propriedades da função Cotangente . . . . . 177
13.5 Função Secante . . . . . . . . . . . . . . . . . . . . 178
13.5.1 Propriedades da função Secante . . . . . . . 179
13.6 Função Cossecante . . . . . . . . . . . . . . . . . . 180
13.6.1 Propriedades da função Cossecante . . . . . 181
13.7 Relações Fundamentais . . . . . . . . . . . . . . . . 182
RESUMO . . . . . . . . . . . . . . . . . . . . . . . . . 186
ATIVIDADES . . . . . . . . . . . . . . . . . . . . . . 187
REFERÊNCIAS BIBLIOGRÁFICAS . . . . . . . 188

Aula 14: Fórmulas de Adição e


Leis Fundamentais 189
14.1 Introdução . . . . . . . . . . . . . . . . . . . . . . . 190
14.2 Fórmulas de Adição . . . . . . . . . . . . . . . . . . 190
14.3 Lei dos Senos e Lei dos Cossenos . . . . . . . . . . 194
RESUMO . . . . . . . . . . . . . . . . . . . . . . . . . 197
ATIVIDADES . . . . . . . . . . . . . . . . . . . . . . 198
REFERÊNCIAS BIBLIOGRÁFICAS . . . . . . . 200

Aula 15: Equações e


Inequações Trigonométricas 201
15.1 Introdução . . . . . . . . . . . . . . . . . . . . . . . 202
15.2 Equações Fundamentais . . . . . . . . . . . . . . . 202
15.2.1 sen x = sen a . . . . . . . . . . . . . . . . . 202
15.2.2 cos x = cos a . . . . . . . . . . . . . . . . . . 203
15.2.3 tg x = tg a . . . . . . . . . . . . . . . . . . . 204
15.2.4 A equação a sen x + b cos x = c . . . . . . . 205
15.3 Inequações Trigonométricas . . . . . . . . . . . . . 206
15.3.1 Inequação do tipo sen x > m . . . . . . . . . 207
15.3.2 Inequação do tipo tg x > m . . . . . . . . . . 208
RESUMO . . . . . . . . . . . . . . . . . . . . . . . . . 210
ATIVIDADES . . . . . . . . . . . . . . . . . . . . . . 210
REFERÊNCIAS BIBLIOGRÁFICAS . . . . . . . 212
AULA

1
Produto Cartesiano
e o plano R2

META:
Denir elementos, que permitirão o estudo de funções reais de uma
variável real.
OBJETIVOS:
Ao m da aula os alunos deverão ser capazes de:
Denir o produto cartesiano e identicar o gráco de uma função
real como um subconjunto do produto cartesiano, satisfazendo cer-
tas condições.
Identicar quando um subconjunto do produto cartesiano dene o
gráco de uma função.
Denir e caracterizar o plano R2 .

PRÉ-REQUISITOS
Noções sobre teoria de Conjuntos.
Produto Cartesiano e o plano R2

1.1 Introdução

Neste capítulo e nos próximos capítulos, vamos estudar as funções


reais de uma variável real, ou seja, dado X ⊂ R um subconjunto de
números reais, consideramos a função f : X −→ R, onde para cada
x ∈ R o valor f (x) é um número real. Começaremos abordando os
casos mais simples.
Primeiramente estudaremos a função Am e para atingir tal obje-
tivo, faremos uma breve revisão sobre produto cartesianoe o plano
R2 .

1.2 Produto Cartesiano

Denição 1.1. Um par ordenado p = (a, b) é formado por um


objeto a, chamado de primeira coordenada de p e um objeto b,
chamado de segunda coordenada de p.

Dois pares ordenados p = (a, b) e q = (c, d) serão chamados de


iguais se, e somente se, a = c e b = d, ou seja, dois pares ordenados
são iguais se suas primeiras coordenadas são iguais e suas segundas
coordenadas são iguais.
Sejam X e Y dois conjuntos.

Denição 1.2. O Produto Cartesiano X ×Y de dois conjuntos X e


Y é o conjunto X ×Y formado por todos os pares ordenados (x, y),
cuja primeira corrdenada x pertence a X e a segunda coordenada
y pertence a Y .

Simbolicamente, denotamos por:

X × Y = {(x, y); x ∈ X, y ∈ Y }.

14
Matemática para o Ensino Médio I AULA
Exemplo 1.1.

Consideremos os subconjuntos de números reais 1
X = {3, −5, 2} e Y = {3, 0}. Então o produto cartesiano X × Y
é o conjunto formado pelos pares ordenados (3, 3), (3, 0), (−5, 3),
√ √
(−5, 0), ( 2, 3) e ( 2, 0), ou seja, o produto cartesiano neste caso
é o conjunto

√ √
X × Y = {(3, 3), (3, 0), (−5, 3), (−5, 0), ( 2, 3), ( 2, 0)}.

Note que o par ordenado (3, −5) não pertence ao produto carte-
siano X × Y , pois seu segundo elemento, −5 não pertence ao con-
junto Y . Além disso, observe que o número de elementos do pro-
duto cartesiano X × Y é seis, ou seja, o número de elementos do
conjunto X vezes o número de elementos do conjunto Y . Mais
Geralmente, se X = {x1 , x2 , . . . , xn } e Y = {y1 , y2 , . . . , ym } são
dois conjuntos nitos, com n e m elementos respectivamente, o
produto cartesiano X × Y tem um número nito de elementos com
mn elementos.

Exemplo 1.2. Seja C uma circunfêrencia e AB um segmento de


reta. O produto cartesiano C ×AB é representado por um cilindro.
Para vermos este fato, consideremos o segmento de reta AB per-

Figura 1.1: O produto Cartesiano C × AB .

15
Produto Cartesiano e o plano R2

pendicular ao plano que contém o círculo C . Cada par ordenado


(x, y) de C × AB pode ser representado pelo ponto P , interseção
da reta perpendicular ao plano de C tirada pelo ponto x em C
com o plano perpendicular ao segmento AB tirado pelo ponto y
em AB . Veja Figura 1.2.

Denição 1.3. O Gráco de uma função f : X −→ Y é o sub-


conjunto G(f ) do produto cartesiano X × Y formado por todos os
pares ordenados da forma (x, y), onde x é qualquer elemento de X
e y , para cada x, é dado por y = f (x). Assim

G(f ) = {(x, y) ∈ X × Y ; y = f (x)} = {(x, f (x)); x ∈ X}.

Uma pergunta natural que surge neste momento é: Todo subcon-


junto do produto cartesiano X ×Y , dene o gráco de uma função?
A resposta para esta questão é não. Lembramos que para termos
uma função do conjunto X no conjunto Y , para cada valor x ∈ X ,
devemos associar um único valor y = f (x) em Y . Descrevemos
agora então, quais são as condições necessárias e sucientes para
que um subconjunto G ∈ X × Y seja o gráco de alguma função
f : X −→ Y .
Condição 1: Para cada x ∈ X existe um par ordenado (x, y) ∈ G,
cuja primeira coordenada é x.
Condição 2: Se p = (x, y) e q = (x, z) são pares pertencentes a G
com a mesma primeira coordenada x, então y = z e consequente-
mente p = q .
O conceito de produto cartesiano está intimamente ligado ao con-
ceito de relação binária.

Denição 1.4. Uma relação binária R entre os elementos de um


conjunto X e de um conjunto Y é um conjunto de condições que

16
Matemática para o Ensino Médio I AULA
permitam determinar, dados x ∈ X e y ∈ Y , se x está, ou não, rela- 1
cionado com y, segundo a relação R. No caso armativo escreve-se
xRy .

Exemplo 1.3. Sejam X = Y = R, ou seja, X e Y o conjunto dos


números reais e consideremos a relação R como sendo a relação
de menor que. Uma condição que nos permite escrever x < y é
x − y < 0. Note que neste exemplo 4R8 (quatro está relacionado
com oito), pois 4 − 8 < 0.

Um exemplo particular de relação e a relação funcional. Quando


temos uma função f : X −→ Y , dizemos que o elemento x ∈ X
está relacionado com o elemento y ∈ Y quando y = f (x). O gráco
de uma relação R entre os conjuntos X e Y é o subconjunto G(R)
do produto cartesiano X × Y formado pelos pares ordenados (x, y)
tais que xRy .

Observação 1.1. Os textos escolares de matemática em nosso


país denem uma função f : X −→ Y como um subconjunto
do produto cartesiano X × Y com as propriedades descritas pelas
Condições 1 e 2, acima enunciadas. Segundo o autor Elon L. Lima,
esta denição apresenta o incoviniente de ser formal, estática e
não transmitir a idéia intuitiva de função como correspondência.
Ainda, segundo o mesmo autor, os matemáticos e os usuários da
matemática olham para uma função como uma correspondência,
não como um conjunto de pares ordenados .

1.3 O plano Numérico R2

Consideremos o produto cartesiano R × R, que denotaremos por


R2 . Os elementos (x, y) de R2 são naturalmente os pares ordenados

17
Produto Cartesiano e o plano R2

de números reais. Eles surgem como as coordenadas do ponto P


do plano Π, quando se xa neste plano um par de eixos ortogonais
OX e OY que se intersectam no ponto O, chamado origem do
sistema de coordenadas. Dado o ponto P ∈ Π, a abcissa de P é o
número x, coordenada do pé da perpendicular baixada de P sobre
o eixo OX , enquanto a ordenada de P é a coordenada y do pé da
perpendicular baixada de P sobre o eixo OY .

Figura 1.2: Coordenadas do ponto P = (x, y) no plano R2 .

Diz-se então que (x, y) é o par de coordenadas do ponto P relati-


vamente ao sistema de eixos OXY . Os eixos OX e OY devidem o
plano em quatro regiões, chamadas de quadrantes, caracterizadas
pelo sinal das coordenadas de seus pontos. Assim no primeiro
quadrante temos x ≥ 0 e y ≥ 0, no segundo quadrante x ≤ 0 e
y ≥ 0, no terceiro quadrante x ≤ 0 e y ≤ 0 e no quarto quadrante
x ≥ 0 e y ≤ 0.

Consideremos a função f : Π −→ R2 , f (P ) = (x, y), ou seja, que


associa a cada ponto P do plano Π seu par de coordenadas (x, y)
relativamente ao sistema de eixos OXY , é uma relação biunívoca.
Esta função permite traduzir conceitos e propriedades geométri-
cas para uma linguagem algébrica e, reciprocamente, interpretar

18
Matemática para o Ensino Médio I AULA
geometricamente relações entre números reais. É através desta 1
identicação, que podemos associar uma reta ( gura geométrica)
a uma equação algébrica ( como conjunto de pontos (x,y) do plano
numérico R2 ). Desta maneira podemos dizer que R2 é o modelo
aritmético do plano Π e que Π é o modelo geométrico de R2 . Do
nosso ponto de vista, olharemos para R2 como o plano numérico e
chamaremos seus elementos P = (x, y) de pontos e procuraremos
assim um melhor entendimento das propriedades das funções reais
que vamos estudar.
Neste sentido, dados dois pontos P = (x, y) e Q = (u, v) a distância
destes dois pontos em função de suas coordenadas é dada por:

p
d(P, Q) = (x − u)2 + (y − v)2 .

Figura 1.3: Distância entre dois pontos.

Para ver este fato, considere o ponto auxiliar S de coordenadas


(u, y). Desta maneira o segmento P S é paralelo ao eixo OX ( pois
P e S possuem a mesma ordenada y ) e o segmento SQ é paralelo
ao eixo OY ( pois os pontos S e Q possuem a mesma abcissa
x). Portanto P SQ forma um triângulo retângulo com hipotenusa

19
Produto Cartesiano e o plano R2

sendo o segmento P Q e cujos catetos medem |x − u| e |y − v| e


pelo toerema de Pitágoras, segue o resultado.

Exemplo 1.4. Seja C uma circunferência com centro no ponto


A = (a, b) e com raio r > 0. Então pela denição de circunferência,
um ponto P = (x, y) pertence a C se, e somente se, d(P, A) = r.
Pela fórmula de distância obtida acima, temos que a circunferência
C é o conjunto

C = {(x, y); (x − a)2 + (y − b)2 = r2 },

e diz-se então que

(x − a)2 + (y − b)2 = r2

é a equação da circunferência com centro no ponto A = (a, b) e


raio r. Por sua vez o disco D de centro em A e raio r é formado
pelos pontos P = (x, y) tais que a distância ao ponto A é menor
igual a r. Portanto

D = {(x, y); (x − a)2 + (y − b)2 ≤ r2 }.

O gráco de uma função real de variável real f : X −→ R é um


subconjunto do plano numérico R2 e portanto pode ser visualizado
( nos casos mais simples) como uma linha, formada pelos pontos
de coordenadas (x, f (x)), quando x varia em X .

Exemplo 1.5. Considere a função f : X −→ R onde X = [−1, 1]



denida por f (x) = 1 − x2 . Vamos tentar reconhecer o gráco
de f . Observe que um ponto (x, y) pertence ao gráco de f se, e
somente se,

−1 ≤ x ≤ 1 e y = 1 − x2
⇔ −1 ≤ x ≤ 1 e y 2 = 1 − x2
⇔ y ≥ 0 e x2 + y 2 = 1.

20
Matemática para o Ensino Médio I AULA
Agora usando a fórmula da distância entre dois pontos, vemos que 1
o gráco de f é a parte da circunferência com centro em (0, 0) e
raio 1, situada no semi-plano y ≥ 0.

No caso de funções reais de uma variável real, as condições 1 e 2


enunciadas acima, adquirem uma forma mais geométrica, que pode
ser rusumida como: Seja X um conjunto que consideraremos situa-
do sobre o eixo horizontal. Um subconjunto G ⊂ R2 é o gráco
de uma função f : X −→ R se, e somente se, toda reta paralela
ao eixo vertical, traçada a partir de um ponto de X, intersecta G
num único ponto.

RESUMO


Na aula de hoje, denimos os elementos preliminares para estudar


as funções reais de uma variável real. Como desejamos entender
o comportamento destas funções e esse objetivo é completamente
atingido quando obtemos o gráco da função. Por isso, se faz
necessário as denições de produto cartesiano em R2 e a denição
de gráco de uma função

REFERÊNCIAS BIBLIOGRÁFICAS


Lima, E.L., A Matemática no Ensino Médio I, 5.ed., SMB, 2006.

21
AULA

2
Função Am

META:
Denir e caracterizar função Am e função linear.
OBJETIVOS:
Ao m da aula os alunos deverão ser capazes de:
Denir função Am e Função linear e identicar suas propriedades.
Mostrar que o gráco de uma função linear é uma reta.
Caracterizar um função am e uma função linear.
Estabelecer a relação entre função am e progressões aritméticas.

PRÉ-REQUISITOS
Noções sobre produto cartesiano e o plano R2 , como também pro-
priedades básicas de progressões aritméticas.
Função Am

2.1 Introdução

Nesta seção vamos denir o conceito de função am, mostrando


que estas funções são aquelas cujos grácos, são retas.

2.2 Função Afim

Denição 2.1. Uma função f : R −→ R chama-se Am quando


existem constantes a, b ∈ R tais que f (x) = ax + b, para todo
x ∈ R.

Fica evidente identicar uma função am pela denição acima, se


lhe é dado a lei de formação da função. Porém, se a função é dada
por uma tabela de dados ou se, dado um fenômeno e queremos
representá-lo através de uma função, como identicar se a melhor
função que descreve este fenômeno é uma função am. Por isso
torna-se necessário caracterizar uma função am. Este é o objetivo
principal desta aula.

Exemplo 2.1. A função identidade f : R −→ R denida por


f (x) = x para todo x ∈ R, é am. Ainda, são casos particulares de
função am as funções lineares, f (x) = ax e as funções constantes
f (x) = b.

Seja f : R −→ R uma função am. Determinamos explicitamente


os coecientes a e b. Primeiramente é fácil ver que b = f (0), o
qual é chamado valor inicial da função f . O Coeciente a pode
ser determinado desde que conhecemos dois valores f (x1 ) e f (x2 )
da função f em dois pontos distintos x1 e x2 . Com efeito, sejam

f (x1 ) = ax1 + b e f (x2 ) = ax2 + b,

24
Matemática para o Ensino Médio I AULA
donde obtemos 2
f (x2 ) − f (x1 ) = a(x2 − x1 ),

e portanto
f (x2 ) − f (x1 )
a= .
x2 − x1

Dados x, x + h ∈ R, com h 6= 0, o número a = f (x + h) − f (x)/h


chama-se taxa de crescimento ou taxa de variação da função no
intervalo de extremos x e x + h.

Exemplo 2.2. Um exemplo de um modelo de uma função am é


o preço a pagar de uma corrida de taxi. O preço pode ser dado
pela função f : x 7−→ ax + b, onde x é a distância percorrida(
medida em quilômetros). O valor inicial b é chamada de bandeira
e o coeciente a é o preço de cada quilômetro rodado.

Sejam

P1 = (x1 , ax1 + b)
P2 = (x2 , ax2 + b)
P3 = (x3 , ax3 + b)

três pontos do gráco da função am f (x) = ax+b. Vamos mostrar


que dados três quaisquer pontos do gráco de uma função am são
colineares. Para mostrar este fato, basta vericar que a maior das
três distâncias d(P1 , P2 ), d(P2 , P3 ) e d(P1 , P3 ), seja igual a soma
dos outros dois. (Observe que se uma das distâncias fosse menor
que a soma das outras duas, os três pontos seriam os vértices de
um triângulo). Podemos supor, sem perda de generalidade que
x1 < x2 < x3 . Da fórmula da distância entre dois pontos obtemos

25
Função Am

que:
p √
d(P1 , P2 ) = (x2 − x1 )2 + a2 (x2 − x1 )2 = (x2 − x1 ) 1 + a2 ,
p √
d(P2 , P3 ) = (x3 − x2 )2 + a2 (x3 − x2 )2 = (x3 − x2 ) 1 + a2 ,
p √
d(P1 , P3 ) = (x3 − x1 )2 + a2 (x3 − x1 )2 = (x3 − x1 ) 1 + a2 .

Daí segue imediatamente que d(P1 , P3 ) = d(P1 , P2 ) + d(P2 , P3 ) e


obtemos assim o resultado.

Figura 2.1: Os pontos P1 = (x1 , ax1 + b), P2 = (x2 , ax2 + b) e


P3 = (x3 , ax3 + b) colineares.

Geometricamente, b é a ordenada do ponto de interseção da reta


que é o gráco da função f : x 7−→ ax + b com o eixo OY . O
número a chama-se inclinação ou coeciente angular dessa reta (
em relação ao eixo horizontal OX). Quanto maior o valor de a,
mais esta reta se afasta da posição horizontal. Quando a > 0 o
gráco de f é uma reta ascendente (quando se percorre no sentido
positivo de x) e quando a < 0, a reta é descendente.
De maneira geral, para conhecer-mos uma função f : X −→ Y
devemos ter uma regra que determina o valor de f (x) para todo
x ∈ X . No caso de uma função Am; como mostramos que seu
gráco é uma reta e uma reta ca determinada quando se conhece
dois de seus pontos; segue que basta conhecer dois valores da f ,

26
Matemática para o Ensino Médio I AULA
f (x1 ) e f (x2 ), em dois números distintos x1 e x2 (arbitrários) e 2
assim f ca inteiramente determinada.

Proprosição 2.1. Dados arbitrariamente (x1 , y1 ), (x2 , y2 ) ∈ R2 ,


com x1 6= x2 , existe uma, e somente uma, função am f : R −→ R
tal que f (x1 ) = y1 e f (x2 ) = y2 .

Prova: Seja f : R −→ R uma função am tal que f (x1 ) = y1 e


f (x2 ) = y2 e x1 6= x2 . Mostraremos que f é escrita de maneira
única na forma f (x) = ax + b e para isso precisamos determinar
de forma única a e b. Mas de f (x1 ) = y1 e f (x2 ) = y2 obtemos o
sistema:
ax1 + b = y1 ,
(2.1)
ax2 + b = y2 .
As incógnitas do sistema são a e b. Desde que o determinante da
matriz dos coecientes associado ao sistema é x1 − x2 e desde que
x1 6= x2 , o determinante é diferente de zero e o sistema possui
solução única, dada por:

y2 − y1 x2 y1 − x1 y2
a= b=
x2 − x1 x2 − x1

Mostramos que o gráco de uma função am é uma reta. Além


disso, é possível mostrar que toda reta r, não vertical, é o grá-
co de uma função am. Este problema está proposto como uma
atividade. Assim dada a função am f (x) = ax + b o seu grá-
co é a reta com equação ax + b = 0. Dados dois pontos (x1 , y1 ) e
(x2 , y2 ), com os valores das constantes a e b determinadas na prova
da Proposição 2.1, obtemos que a equação da reta que passa por
esses dois pontos é:

y2 − y1
y = y1 + (x − x1 )
x2 − x1

27
Função Am

ou
y2 − y1
y = y2 + (x − x2 ).
x2 − x1
a primeira equação nos informa que se começamos no ponto (x1 , y1 )
e caminharmos sobre a reta, fazendo x variar, a ordenada y começa
com o valor y1 e sofre um incremento igual ao incremento x − x1
dado a x, vezes a taxa de variação
y2 − y1
.
x2 − x1
A interpretação da segunda equação é análoga.
Comentário sobre Terminologia: Destacamos algumas obser-
vações sobre a terminologia. Considere a função am f (x) = ax+b.
Não é considerado adequado chamar o número a de coeciente an-
gular da função f e sim, devemos chamá-lo de taxa de variação.
Isto se deve, pois em geral não há ângulo no problema estudado.
O termo coeciente angular usa-se quando estamos considerando
Antonio Trajano em a a equação de uma reta. Também não é adequado chamar função
Aritmética Progressiva
(1883), dá a seguinte am de função do primeiro grau, mesmo que a expressão que re-
denição para grandezas
proporcionais: presenta a função f (x) = ax + b é um polinômio de primeiro grau
Diz-se que duas grandezas
são proporcionais quando (quando a 6= 0). Função não tem grau.
elas se correspon-
dem de tal modo que,
multiplicando-se uma 2.3 Função Linear
quantidade de uma delas
por um número, a quan-
tidade correspondente da Uma função linear é um caso particular de uma função am, mais
outra ca multiplicada
ou dividida pelo mesmo especicamente no caso em que b = 0 e desta maneira uma função
número. No primeiro
caso, a proporcionalidade linear é dada pela fórmula f (x) = ax. O conceito de função linear
se chama direta e, no
segundo caso, inversa; está estritamente ligado ao conceito de proporcionalidade. Usare-
as grandezas se dizem
diretamente propor- mos o conceito de proporcinalidade apresentado por Elon L. Lima,
cionais ou inversamente
proporcionais. o qual é uma versão moderna da denição de Antonio Trajano
(1983).

28
Matemática para o Ensino Médio I AULA
Denição 2.2. Uma proporcionalidade é uma função f : R −→ R 2
tal que, pra quaisquer números reais x e c, tenhamos f (cx) = cf (x)
(proporcionalidade direta) ou f (cx) = f (x)/c (proporcionalidade
inversa).

Exemplo 2.3. Uma rma asfaltou uma estrada de 36 km em 14


horas. Quantos dias seriam necessários para esta mesma rma,
nas mesmas condições, asfaltar 54 km. E em quantos dias asfal-
taria x km? Denotamos por f (x) o número de dias necessários
para asfaltar x km de estrada. Observe que para construir 36 km
são necessários 14 dias, logo para construir 1 km são necessários
14/36 dias. Logo para construir xkm desta estrada são necessários
f (x) = 14
36 x dias. ainda, se multiplicarmos o número de quilôme-
tros por n ∈ N, teremos f (xn) = 14
36 xn = nf (x), mostrando que f
é uma proporcionalidade.

Proprosição 2.2. Seja f : R −→ R uma proporcionalidade (di-


reta) então temos que para todo x ∈ R, f (x) = ax, onde a = f (1).

Prova: Da denição de proporcionalidade (direta) f (cx) = cf (x),


para todo c e todo x, então escrevendo a = f (1), tem-se f (c) =
f (c.1) = cf (1) = ca, ou seja f (c) = ac para todo c ∈ R. Logo
usando a notação mais comum, temos que f (x) = ax para todo
x ∈ R, portanto f é uma função linear.

Em outras palavras, a grandeza y é diretamente proporcional à


grandeza x qaundo existe uma constante a (chamada a constante
de proporcionalidade) tal que y = ax para todo x ∈ R.

Exemplo 2.4. No Exemplo 2.3, a proporcionalidade (compri-


mento da estrada) −→ ( dias necessários para asfaltá-la) tem fator

29
Função Am

de proporcionalidade k = 14/36.

Nem sempre a constante de proporcionalidade a é explicita. Mas


nos problemas relativos à proporcionalidade, o que importa é saber
que se y = f (x) e y 0 = f (x0 ) então y 0 /x0 = y/x é constante.
Vejamos um exemplo deste fato.

Exemplo 2.5. Considere o triângulo de vértices A, B e C e sejam


X e Y pontos sobre o segmento AB e AC respectivamente de modo
que o segmento XY é paralelo ao segmento BC . Pelo Teorema
de Tales, o comprimento y do segmento AY é proporcional ao
comprimento x do segmento AX . Mas qual seria a constante de
O teorema de Tales arma proporcionalidade?
que quando duas retas
transversais cortam um A questão que surge neste momento é saber quando a correspondên-
feixe de retas paralelas, as
medidas dos segmentos de- cia y 7−→ x é uma proporcionalidade? Pela denição que temos,
limitados pelas transver-
sais são proporcionais. teríamos que vericar que f (cx) = cf (x) para todos os valores de
c, x ∈ R, e em particular para todo c. Mas há uma diculdade de
vericar este fato quando c é um número inteiro ou irracional. O
teorema que apresentamos abaixo determina quando uma função
é linear. Começamos, lembrando alguns conceitos sobre funções.

Observação 2.1. Lembremos que uma função f : X −→ R, com


X ⊂ R chama-se:
(a)- Crescente quando x1 < x2 =⇒ f (x1 ) < f (x2 );
(b)- Monótona não-decrescente quando x1 < x2 =⇒ f (x1 ) ≤
f (x2 );
(c)- Monótona não-crescente quando x1 < x2 =⇒ f (x1 ) ≥ f (x2 );
Em qualquer dos quatro casos dizemos que f é monótona. Nos
casos (a) e (b) diz-se que f é estritamente monótona e nestes casos
f é injetiva.

30
Matemática para o Ensino Médio I AULA
Teorema 2.1. Teorema Fundamental da Proporcionalidade. 2
Seja f : R −→ R uma função crescente. As seguintes armações
são equivalentes:

(1) f (nx) = nf (x) para todo n ∈ Z e todo x ∈ R.

(2) Sendo a = f (1), tem-se f (x) = ax, para todo x ∈ R.

(3) f (x + y) = f (x) + f (y) para quaisquer x, y ∈ R.

Prova: Vamos mostrar que (1) =⇒ (2), (2) =⇒ (3) e (3) =⇒ (1).
Provamos primeiramente que (1) =⇒ (2), e começamos provando
que para todo racional r = m/n, a hipótese (1) implica que f (rx) =
rf (x) para todo número racional r e x ∈ R. De fato,

n.f (rx) = f (nrx) = f (mx) = m.f (x),

logo
m
f (rx) = f (x) = rf (x).
n
Ainda, sendo a = f (1) e desde que f (0) = 0 ( pois f(0)=f(0.0)=0.f(0)),
a monotocidade de f nos dá a = f (1) > f (0) = 0. Além disso,
f (r) = f (r.1) = r.f (1) = ar para todo r ∈ Q.
Mostramos agora que f (ax) = ax para todo x ∈ R. Suponha por
absurdo que exista um número real ( necessariamente irracional)
tal que f (x) 6= ax. Suponhamos, sem perda de generalidade que
f (x) > ax. ( O caso em que f (x) < ax é tratado de modo análogo.)
Temos
f (x)
> x.
a
Seja r ∈ Q tal que
f (x)
> r > x.
a

31
Função Am

Então f (x) > ar > ax, ou seja, f (x) > f (r) > ax. Mas isto é
um absurdo, pois f é crescente logo, como r > x, deveríamos ter
f (r) > f (x). Isto completa a prova de (1) =⇒ (2).

Para provar que (2) =⇒ (3), observe que assumindo (2), f (x+y) =
a(x + y) = ax + ay = f (x) + f (y).

Assuma a hipótese (3). Seja n ∈ N então segue que f (nx) =


f (x + x + . . . x) e aplicando sucessivas vezes f (x + x) = f (x) +
f (x) obtemos que f (nx) = nf (x). O caso em que n é um inteiro
negativo segue de maneira semelhante. Assim prova-se que (3) =⇒
(1).

Observação 2.2. As vezes o teorema da proporcionalidade deve


ser aplicado a grandezas, cujas medidas são expressas por números
positivos. Neste caso devemos introduzir a função F : R −→ R
denida por F (0) = 0, F (x) = f (x) e F (−x) = −f (x) para todo
x > 0 e aplicamos o teorema para F obtendo condições equiva-
lentes a (1), (2) e (3) para f . Como consideramos a função f
crescente temos que a = f (1) > 0, mas poderíamos supor que f é
decrescente com a única diferença que teríamos a < 0.

Exemplo 2.6. Suponhamos que investimos uma quantia x em


uma aplicação nanceira. Depois de um certo tempo temos um
montante f (x). É evidente que f (x) é uma função crescente em x,
pois quanto maior o capital aplicado, maior será o montante nal.
Mostramos agora que f (nx) = nf (x) para todo n ∈ N e para todo
x ∈ R. De fato, se aplicarmos um capital x0 = nx ( um capital
n vezes o capital x) e se zermos n aplicações do capital x numa
mesma data, obteremos no nal um mesmo montante.

32
Matemática para o Ensino Médio I AULA
2.4 Caracterização da Função Afim 2
Um problema que surge nos problemas práticos é saber, se numa
determinada situação, o modelo matemático a ser adotado é uma
função am. O próximo teorema caracteriza uma função am.

Teorema 2.2. Seja f : R −→ R um função monótona injetiva.


Se o acréscimo f (x + h) − f (x) = ψ(h) depender apenas de h e
não de x, então f é uma função am.

Prova: Suponhamos que f seja uma função crescente ( caso de


f ser descrescente se mostra de maneira análoga) e então ψ(h)
também é uma função crescente em h. De fato, Se h1 < h2 então
f (x + h1 ) < f (x + h2 ) pois x + h1 < x + h2 e f é crescente e assim
= ψ(h1 ) = f (x + h1 ) − f (x) < f (x + h2 ) − f (x) = ψ(h2 ). Além
disso ψ(0) = 0. Sejam h, k ∈ R arbitrários. Então

ψ(h + k) = f (x + h + k) − f (x)
= f ((x + k) + h) − f (x + k) + f (x + k) − f (x)
= ψ(h) + ψ(k)

Logo, pelo Teorema da Proporcionalidade, pondo a = ψ(1), tem-se


ψ(h) = ah para todo h ∈ R. Assim

f (x + h) − f (x) = ah

. Chamando b = f (0) e fazendo x = 0 na ultima equação, obtemos


f (h) = ah + b e, com uma mudança de variável, f (x) = ax + b
para todo x ∈ R.

Uma maneira natural de dizermos que f (x+h)−f (x) não depende


de x é dizendo "à acréscimos iguais de x correspondem acréscimos

33
Função Am

iguais em f (x)", ou de outra maneira, acréscimos sofridos por f (x)


são proporcionais aos acréscimos dados a x.

Exemplo 2.7. Suponhamos que uma partícula se move sobre uma


linha reta e sua posição é s(t), dada no instante t. O movimento
se diz uniforme quando a partícula se desloca sempre no mesmo
sentido ( ou seja a função s é monótona) e, além disso a partícula
percorre espaços iguais em tempos iguais. Ou seja, s(t + h) − s(t)
que é o espaço percorrido no tempo h, a partir da posição s(t),
depende apenas do acréscimo h e não de t. Desta forma, s é uma
função am e portanto s(t) = at + b, onde a = s(t + 1) − f (t)
o espaço percorrido na unidade de tempo, chama-se velocidade e
b = s(0) é a posição inicial.

Existe uma conexão interessante entre funções am e progressões


aritméticas. Uma Progressão Aritmética pode ser vista geométrica-
mente como uma seqüência de pontos x1 , x2 , . . . , xi , . . . igualmente
espaçadas na reta. Isto quer dizer que se h é a razão, h = xi+1 −xi ,
nào depende de i.

Proprosição 2.3. Seja f : R −→ R uma função am e x1 , x2 , . . . ,


xi , . . . uma progressão aritmética. Então f (x1 ), f (x2 ), . . . , f (xi ), . . .
também formam uma progressão aritmética.

Prova: Seja y = f (x) = ax + b a função am e h = xi+1 − xi a


razão da progressão aritmética x1 , x2 , . . . , xi , . . .. Temos que:

f (xi+1 )−f (xi ) = yi+1 −yi = axi+1 +b−(axi +b) = a(xi+1 −xi ) = ah.

Logo a sequência de pontos y1 = f (x1 ), y2 = f (x2 ), . . . , yi =


f (xi ), . . . está igualmente espaçada em ah unidades e portanto
forma uma progressão aritmética de razão ah.

34
Matemática para o Ensino Médio I AULA
Um resultado recíproco também é verdadeiro, como mostra a pro- 2
posição:

Proprosição 2.4. Seja f : R −→ R uma função monótona que


transforma qualquer progressão aritmética x1 , x2 , . . . , xi , . . . numa
progressão aritmética y1 = f (x1 ), y2 = f (x2 ), . . . , yi = f (xi ), . . ..
Então f é uma função am.

Prova: Construímos uma função auxiliar g : R −→ R, denida


por g(x) = f (x) − f (0). Então g(0) = 0 e g transforma toda
progressão aritmética noutra progressão aritmética. De fato, se
x1 , x2 , . . . , xi , . . . é uma progressão aritmética de razão h então:

g(xi+h ) − g(xi ) = f (xi+h ) − f (0) − (f (xi ) − f (0))


= f (xi+h ) − f (xi ) = h1 ,

pois y1 = f (x1 ), y2 = f (x2 ), . . . , yi = f (xi ), . . . é uma progressão


aritmética por hipótese. Mostramos agora que g é linear. Para
todo real x, os números −x, 0, x formam uma progressão arit-
mética, logo o mesmo ocorre com os números g(−x), 0, g(x). Desta
maneira temos que −g(x) = g(−x) ( pois a distância entre os pon-
tos g(−x) e 0 e 0 e g(x) são as mesmas e portanto g(−x) e g(x)
são simétricos em relação a origem). Sejam x ∈ R e n ∈ N. Então
os números 0, x, 2x, . . . , nx formam uma progressão aritmética, o
mesmo ocorrendo com os números g(0) = 0, g(x), g(2x), . . . , g(nx),
cuja razão é g(x). Segue então do termo geral de uma progressão
aritmética que g(nx) = g(0) + ng(x) = ng(x). Se n é inteiro ne-
gativo, então −n ∈ N e g(nx) = −g(−nx) = −(−ng(x)) = ng(x)
e assim g(nx) = ng(x) para todo x ∈ Z. Pelo Teorema da Pro-
porcionalidade, segue que g é linear e desta forma, g(x) = ax para

35
Função Am

todo x ∈ R. Pondo b = f (0), obtemos:

f (x) = g(x) + f (0) = ax + b,

para todo x ∈ R.

2.5 Conclusão

Nesta aula, denimos função am e estudamos alguns casos par-


ticulares como o caso da função linear. O conceito de função
linear está estritamente ligado ao conceito de proporcionalidade.
Mostramos, através do Teorema da Proporcionalidade como ter
certeza que a correspondência x 7−→ y é uma proporcionalidade
e este teorema ainda é útil para mostrar outros resultados im-
portantes sobre funçào am. Terminamos a aula caracterizando as
funções am, ou seja apresentando um teorema que permita saber,
se numa dada situação o modelo matemático a ser usado é o da
função am.

36
Matemática para o Ensino Médio I AULA
RESUMO 2


Função Am

Denição: Uma função f : R −→ R chama-se am quando exis-


tem constantes a, b ∈ R tais que f (x) = ax + b, para todo x ∈ R.
Uma função linear é um caso particular de uma função am, mais
especicamente no caso em que b 6= 0 e desta maneira uma função
linear é dada pela fórmula f (x) = ax.
Teorema Fundamental da Proporcionalidade. Seja f :
R −→ R uma função crescente. As seguintes armações são equiv-
alentes:

(1) f (nx) = nf (x) para todo n ∈ Z e todo x ∈ R.

(2) Sendo a = f (1), tem-se f (x) = ax, para todo x ∈ R.

(3) f (x + y) = f (x) + f (y) para quaisquer x, y ∈ R.

Caracterização da Função Am: Seja f : R −→ R um função


monótona injetiva. Se o acréscimo f (x+h)−f (x) = ψ(h) depender
apenas de h e não de x, então f é uma função am.
Funções Am e Progressões Aritméticas Seja f : R −→ R
uma função am e x1 , x2 , . . . , xi , . . . uma progressão aritmética.
Então f (x1 ), f (x2 ), . . . , f (xi ), . . . também formam uma progressão
aritmética.
Proposição Seja f : R −→ R uma função monótona que trans-
forma qualquer progressão aritmética x1 , x2 , . . . , xi , . . . numa pro-
gressão aritmética y1 = f (x1 ), y2 = f (x2 ), . . . , yi = f (xi ), . . .. En-
tão f é uma função am.

37
Função Am

ATIVIDADES


Atividade. 2.1. Mostre que uma função am é crescente quando


a > 0, decrescente quando a < 0 e cosntante quando a = 0.

Atividade. 2.2. Mostre que toda reta não-vertical é o gráco de


uma função am.

Atividade. 2.3. No enunciado do Teorema Fundamental da Pro-


porcionalidade, foi feita a hipótese de que a função f fosse monó-
tona. Mostre o mesmo teorema assumindo a hipótese que f é
contínua. bf obs: Note que a monotocidade, na demonstração só
foi usada para provar que f (ar) = ar para todo r ∈ Q e então
f (ax) = ax para todo x ∈ R . Admita que f é contínua e conclua
que f (ax) = ax para todo x ∈ R.

Atividade. 2.4.
As leis da física, muitas vezes descrevem relações de proporcional-
idade direta ou inversa entre grandezas. Para cada uma das leis
abaixo, escreva a expressão matemática correspondente.
(a)- (Lei de gravitação universal) Matéria atrai matéria na razão
direta das massas e na razão inversa do quadrado das distâncias.
(b)- (Gases perfeitos) A pressão exercida por uma determinada
massa de um gás é diretamente proporcional à temperatura abso-
luta e inversamente proporcional ao volume ocupado pelo gás.
(c)- (Resistência Elétrica) A resistência de um o condutor é dire-
tamente proporcional ao seu comprimento e inversamente propor-
cioanal à área de sua seção reta.

38
Matemática para o Ensino Médio I AULA
(d)- (Dilatação térmica) A dilatação térmica sofrida por uma barra 2
é diretamente proporcional ao comprimento da barra e à varaição
de temperatura.

Atividade. 2.5. As grandezas X e Y são inversamente propor-


cionais. Se X sofre um acréscimo de 25%, qual a variação per-
centual sofrida por Y .

Atividade. 2.6.
Os termos a1 , a2 , . . . , an de uma P.A são os valores f (1), f (2), . . . , f (n)
de uma função am.
(a)- Mostre que cada ai é igual a área de um trapézio delimitado
pelo gráco de f , pelo eixo OX e pelas retas verticais de equação
x = i − 1/2 e x = i + 1/2.
(b)- Mostre que a soma S = a1 + a2 + . . . + an é igual a área do
trapézio delimitado pelo gráco de f , pelo eixo OX e pelas retas
verticais x = n − 1/2 e x = n + 1/2.
(c) Conclua que S = 2 n.
a1 +an

Atividade. 2.7. As grandezas X e Y são inversamente propor-


cionais. Se X sofre um acréscimo de 25%, qual a variação per-
centual sofrida por Y .

Atividade. 2.8. Dada as progressões aritméticas

(a1 , a2 , . . . , an , . . .) (b1 , b2 , . . . , bn , . . .),

mostre que existe uma, e somente uma, função am f : R −→ R


tal que f (a1 ) = b1 , f (a2 ) = b2 ,. . . , f (an ) = bn , . . . .

Atividade. 2.9. Dena uma função f : R −→ R pondo f (x) = 2x


se x é racional e f (x) = 3x se x for irracional. Mostre que se tem
f (nx) = nf (x) para todo n ∈ Z e todo x ∈ R, mas f não é linear.

39
Função Am

Atividade. 2.10. Prove que a função f : R −→ R denida por


f (x) = 7x + sen(2πx) é crescente e, para todo x ∈ R xado, trans-
forma a progressão aritmética x, x + 1, x + 2, . . . numa progressão
aritmética. Entretanto, f não é am.

REFERÊNCIAS BILBIOGRÁFICAS


Lima, E.L., A Matemática no Ensino Médio I, 5.ed., SMB, 2006.

40
AULA
Funções Quadráticas
META:
3
Denir e identicar uma função quadrática.

OBJETIVOS:
Ao nal da aula o aluno deverá ser capaz de: Denir função
quadrática.
Entender como surgiu o estudo de funções quadráticas ao longo da
história.
PRÉ-REQUISITOS
Resolução de sistemas lineares e critérios de colinearidade de pon-
tos no plano.
Funções Quadráticas

3.1 Introdução

O estudo das funções quadráticas vem de longa data. Sua na-


tureza, está ligada ao estudo da equação quadrática como também
ao estudo de seu gráco, o qual é uma parábola.

3.2 Função Quadrática

Denição 3.1. Uma função f : R −→ R chama-se quadrática


quando existem números reais a, b, c, com a 6= 0, tais que f (x) =
ax2 + bx + c para todo x ∈ R.

Os coecientes a, b e c da função quadrática cam inteiramente


determinados pelos valores que esta função assume. Assim se ax2 +
bx + c = a1 x2 + b1 x + c + 1, para todo x ∈ R então a = a1 , b = b1 e
c = c1 . Este fato é fácil de ser vericado e deixaremos como uma
atividade.

Observação 3.1. Um trinômio do segundo grau é uma expressão


formal do tipo aX 2 + bX + c com a, b, c ∈ R, com a 6= 0. A palavra
"formal"usada na denição é para dizer que a letra X é apenas
um símbolo, sendo X 2 um modo de escrever XX . Dois trinômios
aX 2 +bX +c e a1 X 2 +b1 X +c1 são iguais se a = a1 , b = b1 e c = c1 .
Mais ainda, um trinômio pode ser identicado por uma terna de
números reais (a, b, c). Assim podemos identicar um trinômio
do segundo grau com uma função quadrática. Acada trinômio
corresponde a função quadrática denida por x 7−→ ax2 + bx + c.
Pelas observações acima esta correspondência é biunívoca.

Proprosição 3.5. Se duas funções quadráticas assumem os mes-


mos valores em três pontos distintos x1 , x2 e x3 , então estas funções

42
Matemática para o Ensino Médio I AULA
são iguais , isto é, assumem o mesmo valor para qualquer número 3
real x.

Prova: Consideramos as funções f (x) = ax2 + bx + c e g(x) =


a0 x2 + b0 x + c duas funções quadráticas que assumem os mesmos
valores f (x1 ) = g(x1 ), f (x2 ) = g(x2 ) e f (x3 ) = g(x3 ), para três
números distintos x1 , x2 e x3 . Temos que f (x1 )−g(x1 ) = 0, f (x2 )−
g(x2 ) = 0 e f (x3 ) − g(x3 ) = 0. Isto signica que:

ax21 + bx1 + c − (a0 x21 + b0 x1 + c0 ) = 0.


ax22 + bx2 + c − (a0 x22 + b0 x2 + c0 ) = 0. (3.2)
ax23 + bx3 + c − (a0 x23 + b0 x3 + c0 ) = 0.

Agora denindo α = a−a0 , β = b−b0 e γ = c−c0 . Assim o sistema


(3.2) pode ser escrito como:

αx21 + βx1 + γ = 0.
αx22 + βx2 + γ = 0. (3.3)
αx23 + βx3 + γ = 0

Subtraindo a primeira equação de cada uma das outras do sistema


(3.3), vem:

α(x22 − x21 ) + β(x2 − x1 ) = 0.


α(x23 − x21 ) + β(xx − x1 ) = 0.
Como x2 − x1 6= 0 e x3 − x1 6= 0, podemos dividir a primeira destas
equações por x2 − x1 e a segunda x3 − x1 , obtendo

α(x1 + x2 ) + β = 0.
(3.4)
α(x1 + x3 ) + β = 0.

Subtraindo membro a membro do sistema (3.4), temos que α(x3 −


x2 ) = 0. Como x3 − x2 6= 0, resultta que α = 0. Substituindo nas
equações anteriores, obtemos sucessivamente β = 0 e γ = 0.

43
Funções Quadráticas

Mais geralmente, dados arbitrariamente os números reais y1 , y2 e


y3 , existe um, e somente um terno ordenado de números a, b, c
tais que

ax21 + bx1 + c = y1 .
ax22 + bx2 + c = y2 . (3.5)
ax23 + bx3 + c = y3
Observe que neste sistema as incógnitas são a, b e c. Aplicando
os mesmos passos da proprosição acima é possível resolver este
sistema. Deixaremos esta tarefa como atividade. Em particular, ao
resolver os sistema acima obtemos o seguinte valor para a incógnita
a.
1 h y3 − y1 y2 − y1 i
a= − (3.6)
x3 − x2 x3 − x1 x2 − x1
Proprosição 3.6. Sejam x1 , x2 , x3 três números reais distintos e
y1 , y2 , y3 números tais que os pontos A = (x1 , y1 ), B = (x2 , y2 )
e C = (x3 , y3 ) são não-colineares em R2 . Existe uma, e somente
uma, função quadrática f (x) = ax2 + bx + c tal que f (x1 ) = y1 ,
f (x2 ) = y2 e f (x3 ) = y3 .

Prova: Dados três números reais distintos x1 , x2 e x3 e três núme-


ros reais arbitrários y1 , y2 e y3 , existe apenas um terno de números
a, b, c tais que a função

f (x) = ax2 + bx + c

compre f (x1 ) = y1 , f (x2 ) = y2 e f (x3 ) = y3 . Porém esta função


pode não ser quadrática, a menos que tenhamos a 6= 0. Pela
expressão obtida para a incógnita a na equação (3.6), o valor de a
é zero se, e somente se, tivermos
y3 − y1 y2 − y1
= . (3.7)
x3 − x1 x2 − x1

44
Matemática para o Ensino Médio I AULA
Se considerarmos os pontos A = (x1 , y1 ), B = (x2 , y2 ) e C = 3
(x3 , y3 ) em R2 , a condição da equação (3.7) signica que as retas
AC e AB têm a mesma inclinação, isto é, os pontos A, B e C são
colineares.

Observação 3.2. Dados os pontos A = (x1 , y1 ), B = (x2 , y2 ) e


C = (x3 , y3 ) distintos em R2 a condição apresentada nos textos
escolares do ensino médio, para que estes pontos seja colineares é
que  
x 1 y1 1
 
det  x2 y2 1  = 0 (3.8)
 
 
x 3 y3 1
Resolvendo este determinante obtemos (x2 − x1 )(y3 − y1 ) − (x3 −
x1 )(y2 − y1 ) = 0, que é equivalente a equação (3.7). Ou seja,
a mesma condiçào de colinearidade para os pontos A, B, C que
obtemos, com a vantagem que não requer o conhecimento de de-
terminantes.

3.2.1 Um problema Antigo


Encontrar as raízes da
equação do segundo grau
Um problema muito antigo, que data de mais de quatro mil anos é x2 −sx+p = 0 é um conhe-
cimento milenar. Porém
encontrar dois números conhecendo-se sua soma e seu produto. Em
até o século XVI não se
termos geométricos, este problema pode ser posto em encontrar os usavam fórmulas para tal,
pois não se representavam
lados de um retângulo, conhecendo o semi-perímetro s e a área p. por letras os coecientes
da equação. Isto começou
Chamamos um dos números de x, logo o outro número será s − x a ser feito por François
Viète (1540-1603).
( pois a soma dos dois deve ser s). Assim o produto é dado por

p = x(s − x) = sx + x2 ,

logo teremos que


x2 − sx + p = 0 (3.9)

45
Funções Quadráticas

e assim os números procurados são as raízes da equação (3.9).


Observe que se o número α é raíz desta equação, então o número
β = s − α também é. Para isto basta mostrar que β também é
raíz da equação (3.9). Deixaremos este fato como atividade para
o leitor.
A regra para encontrar dois números cuja soma e cujo produto são
dados era enunciada pelos bablilônios da seguinte maneira:
Eleve ao quadarado a metade da soma, subtraia o produto e extraia
a raíz quadrada da diferença. Some o resultado a metade da soma.
Isso dará o maior dos números procurados. Subtraia-o da soma
para obter o outro número.
Na notação moderna, esta regra fornece as raízes
r  r 
s s 2 s s 2
x= + −p e s−x= − −p
2 2 2 2
para a equação x2 −sx+p = 0. Uma explicação de como se chegou
a esta conclusão, acreditá-se ser dada assim:
Seja α e β os números procurados e digamos que α < β . Estes
α+β
números são equidistantes de sua média aritmética s
2 = 2 . Se
conhecermos a diferença d = β − (s/2) = (s/2) − α teremos os dois
números α = (s/2) − d e β = (s/2) + d. Mas observe que
s  s   s 2
p = αβ = −d +d = − d2 ,
2 2 2
logo r 
 s 2 s 2
2
d = −p e d= − p.
2 2
Assim r 
s s 2
α = (s/2) − d = − − p,
2 2
e r 
s s 2
β = (s/2) + d = + − p.
2 2

46
Matemática para o Ensino Médio I AULA
Como os dados do problema s e p eram sempre positivos, os Ba- 3
bilônios nunca tiveram a preocupação com eventuais soluções nega-
tivas fornecidas por esta regra. Porém em casos, como por exemplo
de encontrar dois números cujos produto e soma são ambos iguais
a 2, eles armavam simplesmente que tais números não existiam,
o que é verdade no âmbito dos números reais.

Observação 3.3. Se procurarmos dois números cuja soma é 6 e


cujo produto é 9, obteremos 3 e 3. Para não necessitarmos no enun-
ciado fazer esta distinção, usamos a notação de costume, segunda
a qual palavra dois signica "dois ou um". Quando quisermos
garantir que signica "dois", diremos "dois números distintos".

RESUMO


Denição: Uma função f : R −→ R chama-se quadrática quando


existem números reais a, b, c, com a 6= 0, tais que f (x) = ax2 +bx+c
para todo x ∈ R.
O resultado, mais importante, visto nesta seção é que se são dados
três pontos não colineares A = (x1 , y1 ), B = (x2 , y2 ) e C = (x3 , y3 )
são em R2 então existe uma, e somente uma, função quadrática
f (x) = ax2 + bx + c tal que f (x1 ) = y1 , f (x2 ) = y2 e f (x3 ) = y3 .

ATIVIDADES


Atividade. 3.1. Se ax2 + bx + c = a1 x2 + b1 x + c + 1, para todo


x ∈ R, mostre que então a = a1 , b = b1 e c = c1 .

47
Funções Quadráticas

Atividade. 3.2. Dados arbitrariamente os números reais y1 , y2 e


y3 , prove que existe um, e somente um terno ordenado de números
a, b, c tais que

ax21 + bx1 + c = y1 .
ax22 + bx2 + c = y2 .
ax23 + bx3 + c = y3

REFERÊNCIAS BIBLIOGRÁFICAS


Lima, E.L., A Matemática no Ensino Médio I, 5.ed., SMB, 2006.

48
AULA

4
Gráco da Função
Quadrática

META:
Obter propriedades das funções quadráticas e esboçar seu gráco.
OBJETIVOS:
Ao m da aula os alunos deverão ser capazes de:
Esboçar o gráco de uma função quadrática.
Provar a relação para as raízes de uma equação quadrática.
Estudar o movimento uniformemente variado através de uma função
quadrática.

PRÉ-REQUISITOS
Denição e propriedades da função quadrática.
Gráco da Função Quadrática

4.1 Introdução

Nesta seção vamos discutir e deduzir a forma canônica da equação


quadrática, a qual permite provar que o gráco de uma função
quadrática é uma parábola. Também vamos mostrar aplicações
da função quadrática.

4.2 Forma Canônica do Trinômio

Considere o trinômio

ax2 + bx + c.

Considerando a 6= 0 e dividindo esta equação por a, obtemos:


h b ci
ax2 + bx + c = a x2 + x + .
a a

As duas primeiras parcelas do colchete são as mesmas do desen-


b 2
volvimento do quadrado x + 2a . Assim completando quadrado,


podemos escrever
h b b2 b2 ci
ax2 + bx + c = a x2 + 2 x + 2 − 2 + ,
2a 4a 4a a

ou
h b 2 4ac − b2 i
ax2 + bx + c = a x+ + . (4.10)
2a 4a2
Esta maneira de escrever o trinômio do segundo grau é chamada
de forma canônica.

Proprosição 4.7. As raízes da equação ax2 +bx+c = 0 são dadas


por:
√ √
−b + b2 − 4ac −b − b2 − 4ac
x= e x=
2a 2a

50
Matemática para o Ensino Médio I AULA
Prova: Sendo a 6= 0, temos as seguintes equivalências: 4
b 2 4ac−b2
ax2 + bx + c = 0 ⇐⇒ x +

2a + 4a2
=0 (1)
b 2 b2 −4ac

⇐⇒ x + 2a = 4a2
(2)

b b2 −4ac

⇐⇒ x + 2a =± 2a (3)

−b± b2 −4ac
⇐⇒ x = 2a . (4)

A passagem da linha (2) para a linha (3) só tem sentido quando

b2 − 4ac := ∆ (4.11)

chamado de discriminante é maior ou igual a zero (∆ ≥ 0). Caso


tenhamos ∆ < 0, a equivalência entre as linhas (1) e (2) signica
que a equação dada não possui solução real, pois o quadrado x+ 2a
b

não pode ser negativo.


Da fórmula (4) segue imediatamente que, se o discriminante ∆ =
b2 − 4ac é positivo, a equação

ax2 + bx + c = 0.

tem duas raízes distintas


√ √
−b − ∆ −b + ∆
α= e β= ,
2a 2a

com α < β , cuja soma é s = −b/a e cujo produto é

p = (b2 − ∆)/4a2 = 4ac/4a2 = c/a.

Em particular, a média aritmética das raízes é −b/2a e desta


maneira, as raízes α e β estão equidistante do ponto −b/2a. Quando
∆ = 0, a equação dada possui somente uma raíz, chamada de raíz
dupla, igual a −b/2a.

51
Gráco da Função Quadrática

Proprosição 4.8. Seja f (x) = ax2 +bx+c uma função quadrática.


Se a > 0 então a função assume um menor valor no ponto x = − 2a
b

e se a < 0 a função assume um valor máximo em x = − 2a


b
.

Prova: Suponhamos que a > 0. A forma canônica

h b 2 4ac − b2 i
ax2 + bx + c = a x+ + .
2a 4a2
exibe no interior do colchetes, uma soma de duas parcelas. A
primeira depende de x e é sempre maior ou igual a zero. A segunda
é constante. Segue que o menor valor dessa soma é atingido quando
 b 2
x+ = 0,
2a
ou seja, quando x = − 2a
b
. Neste ponto, f (x) também assume
b2
seu valor mínimo f (−b/2a) = c − 4a . Quando a < 0, o valor
2
b
f (−b/2a) = c− 4a é o maior valor dos números f (x), para qualquer
x ∈ R.

Ainda usando a forma canônica, vemos que quando a > 0, f (x) =


ax2 + bx + c não assume valor máximo, ou seja, é uma função
ilimitada superiomente. Analogamente, quando a < 0, f (x) não
assume um valor mínimo, sendo portanto, ilimitada inferiormente.
Outro resultado que caracteriza a função quadrática, que segue da
forma canônica é o seguinte

Proprosição 4.9. Seja f (x) = ax2 +bx+c uma função quadrática


e x e x0 dois pontos equidistantes de −b/2a. Então f (x) = f (x0 ).

Prova: Olhando para a forma canônica (4.16), vemos que f (x) =


f (x0 ) se, e somente se,
b 2 b 2
x+ = x0 + ,
2a 2a

52
Matemática para o Ensino Médio I AULA
e como estamos considerando x 6= x0 , da equação acima, segue que 4
b b
x0 + =x+ ,
2a 2a

isto é,
x + x0 b
=− .
2 2a

4.3 Gráfico da Função Quadrática

Nesta seção vamos mostrar um resultado já conhecido, que nos diz


que o gráco de uma função quadrática é uma parábola. Lem-
bramos inicialmente a denição da parábola como o lugar geomé-
trico de pontos no plano.

Denição 4.1. Seja d uma reta no plano e F um ponto fora desta


reta. A Parábola de foco F e reta diretriz d é o conjunto de pontos
do plano que distam igualmente de F e d. Ou seja, se P = (x, y)
é um ponto da Parábola, necessariamente

d(P, d) = d(P, F ).

A reta perpendicular à diretriz, baixada a partir do foco F , chama-


se eixo da parábola. O ponto mais próximo da diretriz chama-se
vértice da parabola. Ele é o ponto médio do segmento cujas extrem-
idades são o foco e a interseção do eixo com a diretriz. Lembramos
que a distância de um ponto a uma reta é o comprimento do seg-
mento perpendicular baixado do ponto sobre a reta.

Exemplo 4.1. Seja f (x) = x2 . Então o gráco de f é a parábola


cujo foco é o ponto F = (0, 1/4) e cuja reta diretriz é a reta

53
Gráco da Função Quadrática

Figura 4.1: Elementos da Parábola.

horizontal y = −1/4. De fato, a distância de um ponto qualquer


P = (x, x2 ) do gráco de f ao ponto F = (0, 1/4) é
p
d(P, F ) = x2 + (x2 − 1/4))2 .

A distância do mesmo ponto P = (x, x2 ) à reta y = −1/4 é

d(P, d) = x2 + 1/4,

pois observe que a reta perpendicular a y = −1/4 baixada de P ,


intersecta a reta y = −1/4 em um ponto Q = (x, −1/4) que tem a

54
Matemática para o Ensino Médio I AULA
mesma abcissa que P . Logo a distância entre P e a reta y = −1/4 4
é a distância entre os pontos P e Q, ou seja x2 + 1/4. Como
se trata de dois números positivos, para vericarmos a igualdade
d(P, d) = d(P, F ), basta ver que seus quadrados são iguais, o que
de fato é vericado. Observe que estamos u-
sando o seguinte resul-
Exemplo 4.2. Seja f (x) = ax2 com a 6= 0. Então o gráco tado: Sejam x, y ∈ R+ , ou
seja, x > 0 e y > 0 reais.
desta função quadrática é a parábola cujo foco é F = (0, 1/4a) Então x = y se, e somente
se, x2 = y 2 .
e cuja reta diretriz é a reta horizontal y = −1/4a. De fato, a
distância de um ponto qualquer P = (x, ax2 ) do gráco de f ao
ponto F = (0, 1/4a) é
p
d(P, F ) = x2 + (ax2 − 1/4a))2 . (4.12)

Enquanto que a distância do ponto P = (x, ax2 ) do gráco de f a


reta y = −1/4a é dada por

d(P, d) = (ax2 + 1/4a). (4.13)

e agora observamos que x ∈ R vale a seguinte igualdade:


 1 2  2 1 2
x2 + ax2 − = ax + ,
4a 4a

onde o primeiro termo é o quadrado da distância d(P, F ) e o se-


gundo o quadrado da distância d(P, d). Conforme o caso em que
a > 0 a concavidade da parábola é voltada para cima e quando
a < 0 a concavidade é voltada para baixo.

Exemplo 4.3. Seja f (x) = a(x−m)2 com a 6= 0 e m ∈ R qualquer.


Então o gráco da função quadrática f é uma parábola cujo foco
é o ponto F = (m, 1/4a) e cuja reta diretriz é a reta horizontal
1
y = − 4a . Para vericar este fato observamos que o gráco de
f (x) = a(x − m)2 resulta do gráco de g(x) = ax2 pela translação

55
Gráco da Função Quadrática

horizontal (x, y) 7−→ (x + m, y), a qual leva o eixo x = 0 no eixo


x = m.

Exemplo 4.4. Dados a, m, k ∈ R, com a 6= 0, o gráco da função


quadrática f (x) = a(x − m)2 + k é uam parábola cujo foco é
1
F = (m, k + 4a ) e cuja reta diretriz é a reta horizontal y = k − 4a
1
.
Para vericar este fato, observe que o gráco da função quadrática
f (x) = a(x − m)2 + k é obtido do gráco da função quadrática
f (x) = a(x − m)2 por meio da translação vertical (x, y) 7−→ (x, y +
k), que leva o eixo OX na reta y = k e a reta y = − 4a
1
na reta

4a .
1
y=k−

56
Matemática para o Ensino Médio I AULA
Proprosição 4.10. O gráco da função quadrática f (x) = ax2 + 4
bx + c, com a 6= 0 é uma parábola cujo foco é o ponto F =
) e cuja reta diretriz é a reta horizontal y = .
2 +1 4ac−b2 −1
( −b
2a ,
4ac−b
4a 4a

Prova: A forma canônica nos dá:

ax2 + bx + c = a(x − m)2 + k,

onde m = −b
2a e k = (4ac − b2 )/4a. Usando o Exemplo 4.4 concluí-
mos a prova.

O ponto mais próximo da diretriz é aquele cuja abcissa é x̄ =


−b/2a. Neste ponto, f (x̄) atinge seu valor mínimo quando a > 0
e seu valor máximo quando a < 0. Ainda quando x̄ = −b/2a, o
ponto (x̄, f (x̄)) é o vértice da parábola que constitui o gráco de
f (x). Observamos ainda que a reta vertical x = −b/2a é chamada
eixo da parábola e é um eixo de simetria do gráco de f . De fato,
provamos na aula anterior que que a função quadrática f assume
valores iguais f (x) = f (x0 ) se, e somente se, x = x0 .
O gráco da função quadrática, é de vital importância para en-
tendermos o comportamento desta função. Por exemplo os pontos
deste gráco onde a função intersecta o eixo OX são as raízes da
equação quadrática
ax2 + bx + c = 0.

Se denotarmos por α e β as abcissas destes pontos então a abcissa


do vértice da parábola é o ponto médio do segmento [α, β].

Observação 4.1. Consideremos algumas observações sobre trans-


lações de grácos de funções reais.

• Aplicando a translação horizontal (x, y) 7−→ (x + m, y) ao


gráco da função f : R −→ R, obtém-se o gráco da função

57
Gráco da Função Quadrática

g : R −→ R, tal que g(x) = f (x − m) para todo x ∈ R. De


fato, todo ponto (x, f (x)) é levado pela translação no ponto
(x+m, f (x)). Escrevendo x̄ = x+m donde x = x̄−m, vemos
que a transformação leva cada ponto (x, f (x)) do gráco de
f no ponto (x̄, f (x̄ − m)) = (x̄, g(x̄)) do gráco de g .

• A Translação vertical (x, y) 7−→ (x, y + k) transforma o grá-


co da função f : R −→ R no gráco da função h : R −→
R, tal que h(x) = f (x) + k . Com efeito, esta transfor-
mação, leva cada ponto (x, f (x)) do gráco de f (x), no ponto
(x, f (x) + k) = (x, g(x)) do gráco da g(x).

Mostraremos agora que a parábola que é o gráco da função f (x) =


ax2 + bx + c é transformado no gráco da função h(x) = ax2
mediante a uma translação horizontal, seguida de uma translação
vertical. De fato, dada a função quadrática

f (x) = ax2 + bx + c,

cujo gráco é uma parábola e tem vértice cuja abcissa tem coor-
denada m = −b/2a, submetida a translação horizontal (x, y) 7−→
(x − m, y), determina uma nova parábola, cujo vértice tem abcissa
zero e está sobre o eixo OY . Pelo que vimos anteriormente na Ob-
servação 4.1, esta nova parábola é o gráco da função quadrática

b
g(x) = f (x − m) = f (x − 2a )
 2  
b b
= a x − 2a +b x− 2a +c
= ax2 + k

4ac−b2
onde k = 4a . Em seguinda aplicamos a translação vertical
(x, y) 7−→ (x, y + k), obtendo-se uma nova parábola cujo vértice

58
Matemática para o Ensino Médio I AULA
agora é a origem (0, 0). Segue da Observação 4.1 que esta parábola 4
é o gráco da função

h(x) = g(x) − k = ax2 + k − k = ax2 .

Os grácos das funções f (x) = ax2 e ψ(x) = −ax2 são congruentes,


pois se usarmos a relexão em torno do eixo horizontal, ou seja, a
transformação (x, y) 7−→ (x, −y), leva o gráco de ψ(x) = −ax2
no gráco de f (x) = ax2 . Diante de tudo que discutimos acima,
podemos enunciar o seguinte resultado:

Proprosição 4.11. Se a = ±a0 então os grácos das funções


f (x) = ax2 + bx + c e g(x) = a0 x2 + b0 x + c0 são parábolas congru-
entes.

Também vale a reciproca do resultado acima, ou seja, se os grá-


cos de f (x) = ax2 + bx + c e g(x) = a0 x2 + b0 x + c0 são parábolas
congruentes então teremos a = ±a0 . Para vericar este fato, basta
considerar as funções f1 (x) = ax2 e g1 (x) = a0 x2 ,(pelo que já dis-
cutimos anteriormente, o gráco da função f pode ser levado ao
gráco da função f1 , via translações, de modo que sejam congru-
entes) com a > 0 e a0 > 0. Se a 6= a0 então ou a > a0 e então
ax2 > a0 x2 para todo x ∈ R, ou a < a0 e então ax2 < a0 x2 para
todo x ∈ R. Em ambos os casos teremos que as parábolas que são
grácos de f1 e g1 não são congruentes.

4.4 Aplicações

Nesta seção vamos apresentar dois exemplos que mostram a apli-


cabilidade e a importância da função quadrática.

59
Gráco da Função Quadrática

4.4.1 Uma Propriedade da Parábola

Denição 4.2. Uma superfície parabólica ou parabolóide de revo-


lução é a superfície obtida quando giramos uma parábola em torno
do seu eixo.
Um exemplo do uso destas
superfícies parabólicas,
é dado pelas antenas Esta superfície possuí uma série de aplicações, decorrente de uma
parabólicas empregadas
na rádio-astronomia ou propriedade da parábola que vamos discutir nesta seção.
até mesmo em apare-
lhos de televisão, onde Começamos nossa discussão lembrando do princípio físico que esta-
estas reetem os sinais belece que num raio reetido em uma superfície reetora, o ângulo
proviniente de um satélite
sobre sua superfície, de incidência é igual ao ângulo reetido. No caso da superfície
fazendo-os convergir para
um único ponto, o foco e parabólica, para entendermos os ângulos de incidência e reetido,
desta forma tornando o
sinal mais forte. substituímos esta superfície pela parábola obtida pela intersecção
da superfície com o plano que contem o ângulo incidente, o ângulo
reetido e o eixo de rotação. Observamos, que necessitamos denir
o ângulo entre uma curva e uma reta. O ângulo entre uma curva e
uma reta que se intersectam num ponto P , é por denição o ângulo
entre esta reta e a reta tangente a esta curva no ponto P .

Figura 4.2: Ângulo entre uma reta e uma curva.

60
Matemática para o Ensino Médio I AULA
Proprosição 4.12. Se a parábola é o gráco da função f (x) = 4
ax2 + bx + c, então a reta tangente a esta parábola no ponto P =
(x0 , y0 ) é a reta que passa por este ponto e tem equação y = 2ax0 +
b.

Lembramos que a reta tangente a uma parábola no ponto P é a


reta que passa por este ponto e todos os demais pontos da parábola
estão num mesmo lado desta reta. Vamos a demonstração da
proposição.
Prova:

Figura 4.3: Equação da reta tangente a Parábola.

Suponhamos, sem perda de generalidade que a > 0. Lembramos


que a reta que passa pelo ponto P0 = (x0 , y0 ) = (x0 , ax20 + bx0 + c)
e tem inclinação 2ax0 + b tem equação

y = (2ax0 + b)(x − x0 ) + ax20 + bx0 + c.

61
Gráco da Função Quadrática

Então segue que:

x 6= x0 =⇒ ax2 + bx + c − [(2ax0 + b)(x − x0 ) + ax20 + bx0 + c]


= a(x − x0 )2 > 0.

Isto mostra que a reta que passa pelo ponto P0 = (x0 , y0 ) e tem
inclinação 2ax0 + b é tal que para todo ponto de abcissa x 6= x0 da
parábola está acima da reta mencionada. Logo esta reta é tangente
a parábola neste ponto.

Como acabamos de ver na prova da proposição acima, se a > 0


então a parábola se situa acima de qualquer de suas tangentes e
analogamente, se a < 0 ela se situa abaixo de qualquer uma de
suas tangentes.
Consideremos a parábola que é o gráco da função quadrática

f (x) = ax2 + bx + c,

cuja tangente no ponto P = (x, y) tem inclinação 2ax + b. Seja s


a reta que une o foco F ao ponto Q, pé da perpendicular baixada
de P sobre a diretriz d. Vamos supor que 2ax + b 6= 0, ou seja,
x 6= 2a/b e assim o ponto P não é o vértice da parábola. A
inclinação da reta s é a fração cujo numerador é a diferença entre
as ordenadas de Q e F e cujo denominador é a diferença entre as
abcissas destes dois pontos. Vimos já que F = (m, k + 1/4a) e
Q = (x, k − 1/4a), onde m = −b/2a e k = ordenada do vértice da
parábola. Logo a inclinação de s é:
1 1

k − 4a − k + 4a −1 −1 1
= = b
 =− .
x−m 2a(x − m) 2a x + 2a
2ax + b
(4.14)
Isto mostra que a reta s é perpendicular à reta tangente à parabola
no ponto P . Se P é o vértice da parábola (x = −b/2a) a reta s

62
Matemática para o Ensino Médio I AULA
seria vertical e a reta tangente no ponto P teria inclinação nula e 4
logo seria horizontal e neste caso, também teríamos as duas retas
perpendiculares. As retas y = ax = b e y =
a0 x + b0 , com a 6= 0 e a0 6=
0, são perpendiculares se,
Proprosição 4.13. A tangente à parábola num ponto P faz ângu- e somente se, a0 = 1/a.

los iguais com a paralela ao eixo e com a reta que une o foco F a
esse ponto.

Prova: Seja Q o pé da perpendicular baixada de P sobre a diretriz.


Pela denição da parábola, temos que F¯P = P¯Q e desta forma o
triângulo F QP é isósceles. Além disso, acabamos de ver que F Q
é perpendicular à tangente, ou seja, a tangente é a altura deste
triângulo isóceles, logo é também bissetriz. Logo os ângulos F P̂ T 0
e T 0 P̂ Q são iguais. Logo F P̂ T 0 =T 0 P̂ Q = α.

Figura 4.4: A tangente à parábola num ponto P faz ângulos iguais


com a paralela ao eixo e com a reta que une o foco F a esse ponto.

63
Gráco da Função Quadrática

No exemplo de uma antena parabólica, se esta estiver voltada para


a posição do satélite, a grande distância faz com que os sinais emi-
tidos por este seja trajetórias praticamente paralelas ao eixo da
superfície da antena. Logo eles se reetem na superfície e con-
vergem para o foco, de acordo com a proposição que acabamos de
demonstrar.

4.4.2 O movimento uniformemente variado

Suponhamos que a posição de um corpo f (t) no instante t é dada


pela função quadrática

1
f (t) = at2 + bt + c. (4.15)
2
O movimento dado pela função dada pela Equação (4.15), é cham-
dado de movimento uniformemente variado. A constante a chama-
se aceleração, b é a velocidade inicial ( no instante t = 0) e c é a
posição inicial do corpo.
Em qualquer movimento, dado pela função f , o quociente

f (t + h) − f (t) espaço percorrido


=
h tempo percorrido

chama-se velocidade média do ponto no intervalo cujos extremos


são t e t + h. No caso em que f é dada pela fórmula (4.15), a
velocidade média do móvel entre os instantes t e t + h é igual a
at + b + ah/2. De fato:
1
f (t+h)−f (t) a(t+h)2 +b(t+h)+c−( 12 at2 +bt+c)
h = 2
h

= at + b + ah/2.

Se tomarmos h cada vez menor, este valor se aproxima de at + b.


Por isso diz-se que
v(t) = at + b

64
Matemática para o Ensino Médio I AULA
é a velocidade do ponto no instante t. Quando temos t = 0 temos 4
v(0) = b, por isso b se chama a velocidade inicial. Além disso,
vê-se que a = [v(t + h) − v(t)]/h para quaisquer t, h, logo a acele-
ração constante a é a taxa de variação da velocidade. Por isso o
movimento se chama uniformemente variado.( Uniformemente a-
celerado ou retardado, conforme v tenha o mesmo sinal de a ( isto
é, t > −b/a) ou v tenha sinal oposto ao de a ( ou seja, t < −b/a)).
Um exemplo prático do Movimento Uniformemente Variado é a
queda de corpos no vácuo, sujeitos à ação apenas da gravidade.
Neste caso a aceleração é a da gravidade, normalmente indicada
pela letra g . Através de nosso conhecimento da função quadrática
podemos obter uma descrição completa do movimento uniforme-
mente variado.
Outro exemplo do movimento uniformemente variado é o movi-
mento de um projétil (uma bala, por exemplo) lançado por uma
força instantânea e a partir daí, sujeito apenas à ação da gravidade,
sendo desprezada a resistência do ar. Embora o processo ocorra
no espaço tridimensional a trajetória do projétil está contida no
plano determinado pela reta vertical no ponto de partida e pela
direção da velocidade.

Observação 4.2. Quando o movimento ocorre no plano ou no es-


paço, a velocidade é expressa por um vetor (segmento de reta ori-
entado), cujo comprimento se chama velocidade escalar do móvel.
A direção e o sentido desse valor indicam a direção e o sentido do
movimento.

No plano em que se dá o movimento, tomemos um sistema de


coordenadas cuja origem é o ponto de partida do projétil e cujo
eixo OY é a reta vertical que passa por este ponto. A velocidade

65
Gráco da Função Quadrática

inicial do projétil é o vetor v = (v1 , v2 ) cuja primeira coordenada


v1 fornece a velocidade da componente horizontal do movimento
(deslocamento da sombra, ou projeção do projétil sobre o eixo
horizontal OX ). Como a única força atuando sobre o projétil é a
força gravitacional que não possuí componente horizontal, resulta
que , se P = (x, y) é a posição do projetil no instante t, tem-se
x = v1 t.
A aceleração da gravidade é constante, vertical, e igual a −g . (
O sinal menos se deve ao sentido da gravidade ser oposto à orien-
tação do eixo vertical OY .) Portanto, a componente vertical do
movimento de P é um movimento uniformemente acelerado sobre
o eixo OY , com aceleração igaul a −g e velocidade inicial v2 .
Logo em cada instante t, a ordenada y do ponto P = (x, y) é dada
por y = − 21 gt2 + v2 t ( não há termo cosntante, pois y = 0 quando
t = 0). Se v1 = 0 então, para todo t, tem-se x = v1 t = 0, logo
P = (0, y), com
1
y = − gt2 + v2 t.
2
Neste caso, a trajetória do projétil é vertical. Suponhamos agora
que v1 6= 0. Então, de x = v1 t vem t = x/v1 . Substituindo t por
este valor na expressão de y , obtemos

y = ax2 + bx, onde a = −g/2v12 , e b = v2 /v1

isto mostra que a trajetória do projétil é uma parábola.

RESUMO


Funções Quadráticas Dada a função quadrática

ax2 + bx + c.

66
Matemática para o Ensino Médio I AULA
a mesma pode ser escrita da seguinte maneira: 4
h b 2 4ac − b2 i
ax2 + bx + c = a x+ + . (4.16)
2a 4a2

a qual é chamada forma canônica. A forma canônica permite obter


importantes informações sobre o comportamento desta função, como
demonstrar que o gráco de uma função quadrática é uma parábola,
ou seja,
Proposição: O gráco da função quadrática f (x) = ax2 + bx + c,
2 +1
com a 6= 0 é uma parábola cujo foco é o ponto F = ( −b
2a ,
4ac−b
4a )
4ac−b2 −1
e cuja reta diretriz é a reta horizontal y = 4a .
Ainda, a forma canônica, permite provar as fórmula clássica para
as raízes de uma equação quadrática
√ √
−b + b2 − 4ac −b − b2 − 4ac
x= e x= .
2a 2a

Outra propriedade da função quadrática, é enunciada como:


Proposição: Seja f (x) = ax2 + bx + c uma função quadrática.
Se a > 0 então a função assume um menor valor no ponto x = − 2a
b

e se a < 0 a função assume um valor máximo em x = − 2a


b
.
Ainda obtemos como aplicação da função quadrática, uma pro-
priedade interessante da parábola, a qual nos informa que retas
paralelas ao eixo da parábola reetem passando pelo foco. Esta
propriedade tem inúmeras aplicações no cotidiano. Outra apli-
cação é no estudo do Movimento Uniformemente Variado.

ATIVIDADES


67
Gráco da Função Quadrática

Atividade. 4.1. Considere a função quadrática f (x) = ax2 +bx+


c, onde a, b, c ∈ R, a 6= 0 se seja x1 , x2 e x3 três números distintos.
Dados arbitrariamente os números reais y1 , y2 e y3 então existe
um, e somente um, terno de números a, b e c tais que a função f
cumpre f (x1 ) = y1 , f (x2 ) = y2 e f (x3 ) = y3 .

Atividade. 4.2. Qual o valor máximo do produto de dois números


cuja soma é constante.

Atividade. 4.3. Considere a função quadrática

f (x) = ax2 + bx + c,

onde onde a, b, c ∈ R, a 6= 0. De acordo com o que foi visto no


texto, determine:
(a)- As coordenadas do vértice do gráco da função.
(b)- O eixo da parábola.
(c)- Suponha que a função f possua duas raízes α e β , α 6= β .
Conclua que a abcissa do vértice é o ponto médio do segmento
[α, β].

Atividade. 4.4. (a)- Considere a função quadrática f (x) =


ax2 + bx + c. Dena as operações (translações) para trasnformar
esta parábola na parábola h(x) = ax2 . Dizemos então que estas
parábolas são congruentes.
(b)- Considere a transformação, que consiste em uma reexão
em torno do eixo horizontal, ou seja, a transformação (x, y) 7−→
(x, −y). Esta transformação leva o gráco de g(x) = −ax2 no
gráco de h(x) = ax2 . Dena as operações (translações e/ou
reeções) que transforma a parábola f (x) = −ax2 + bx + c na
parábola h(x) = ax2 .

68
Matemática para o Ensino Médio I AULA
Obs: Dos itens (a) e (b) acima, podemos resumir: Se a0 = ±a 4
então os grácos das funções quadráticas f (x) = ax2 + bx + c e
ψ(x) = a0 x2 + b0 x + c0 são congruentes.
(c) Considere as funções quadráticas f (x) = 31 x2 − 83 x + 5 e g(x) =
− 13 x2 − 5x + 25. Determine o conjuntos de operações que leva uma
delas sobre a outra. Esboce o gráco destas duas parábolas.

Atividade. 4.5. Encontre a função quadrática cujo gráco é dado


pela gura:

Atividade. 4.6. Escreva cada uma das funções quadráticas abaixo


na forma f (x) = a(x − m)2 + k . A seguir, calcule sua raízes (se
existirem), o eixo de simetria de seu gráco e seu valor mínimo ou
máximo.
(a)- f (x) = x2 − 8x + 23.
(b)- f (x) = 8x − 2x2 .

Atividade. 4.7. Seja f (x) = ax2 + bx + c com a > 0.


(a)- Mostre que

x1 + x2  f (x1 ) + f (x2 )
f < .
2 2

(b)- Mais geralmente prove que se 0 < α < 1, então

f (αx1 + (1 − α)x2 ) < αf (x1 ) + (1 − α)f (x2 ).

Interprete geometricamnete este resultado.

69
Gráco da Função Quadrática

Atividade. 4.8. Considerando que uma ta se enrola em um car-


retel segundo círculos concentrícos, igualmente espaçados, Mostre
que o tempo T (n) de gravação após n voltas é dada por uma função
do tipo T (n) = an2 + bn. Suponha que a velocidade da ta seja
constante.

Atividade. 4.9. Dado um conjunto de retas no plano, elas deter-


minam um número máximo de regiões quando estão na chamada
posição geral: isto é, elas são concorrentes duas a duas e três retas
nunca têm um ponto em comum. Seja Rn o número máximo de
regiões determinadas por n retas no plano.
(a)- Quando se adiciona mais uma reta na posição geral a um
conjunto de n retas em posição geral, quantas novas regiões são
criadas?
(b)-Prove que Rn = n2 +n+2
2 . (Sugestão: Escreva Rn da seguinte
maneira Rn = (Rn −Rn−1 )+(Rn−1 −Rn−2 )+. . .+(R2 −R1 )+R1 .)

Atividade. 4.10. Se x e y são números reais tais que 3x+4y = 12,


determine o valor mínimo de z = x2 + y 2 .

Atividade. 4.11. Qual o valor máximo de 21n − n2 , onde n é


inteiro.

Atividade. 4.12. Determine explicitamente os coecientes a, b e


c do trinômio f (x) = ax2 + bx + c em função de f (0), f (1) e f (2).

Atividade. 4.13. Que forma tem o gráco da função f : [0, ∞] −→



R dada por f (x) = x.

Atividade. 4.14. Mostre que a equação x + m = x possui uma
raiz se m > 0, duas raízes quando 1
4 < m ≤ 0, uma raiz para
m = −1/4 e nehuma raiz caso m < −1/4.

70
Matemática para o Ensino Médio I AULA
Atividade. 4.15. Prove que a função contínua f : R −→ R 4
é quadrática se, e somente se, para todo h ∈ Rxado, a função
ψ(x) = f (x + h) − f (x) é am e não constante.

REFERÊNCIAS BIBLIOGRÁFICAS


Lima, E.L., A Matemática no Ensino Médio I, 5.ed., SMB, 2006.

71
AULA

5
Caracterização das
Funções Quadráticas

META:
Obter os teoremas de caracterização das funções quadráticas.
OBJETIVOS:
Ao m da aula os alunos deverão ser capazes de:
Identicar a relação entre funções quadráticas e progressões arit-
méticas.

PRÉ-REQUISITOS
Denição e propriedades da função quadrática.
Caracterização das
Funções Quadráticas
5.1 Introdução

Uma questão importante na matemática, e nas ciências que fazem


uso da matemática, é saber se, numa dada situação, a função que
dever ser adatada para modelar o problema de maneira a reetir
a realidade do fenômeno estudado. Com base, neste pensamento,
vamos caracterizar as funções quadráticas.

5.2 Teoremas de Caracaterização das funções


quadráticas

Começamos observando que uma função quadrática nunca é monó-


tona e desta forma, nos teoremas de caracterização que apresen-
tamos a seguir, trabalharemos com a hipótese de continuidade.
Ou seja, admitiremos conhecido que uma função quadrática é con-
tínua e que se duas funções contínuas f, g : R −→ R são tais que
f (r) = g(r) para todo racional r então f (x) = g(x) para todo x
real.

Denição 5.1. Uma progressão aritmética de segunda ordem é


uma sequência y1 , y2 , . . . tal que as diferenças sucessivas

d1 = y2 − y1 , d 2 = y3 − y2 , d3 = y4 − y3 , ...

formam uma progressão aritmética usual.

Exemplo 5.1. Considere a sequência 1, 4, 9, 16, 25, . . . dos quadra-


dos dos números naturais é uma progressão aritmética de segunda
ordem. De fato, as diferenças sucessivas

d1 = 4 − 1 = 3, d2 = 9 − 4 = 5, d3 = 16 − 9 = 7,
d4 = 25 − 16 = 9 . . .
(5.17)

74
Matemática para o Ensino Médio I AULA
formam uma progressão aritmética de razão 2. 5
Proprosição 5.14. Seja f : R −→ R uma função quadrática e
x1 , x2 , x3 , . . . é uma progressão aritmética arbitrária então os nú-
meros y1 = f (x1 ), y2 = f (x2 ), y3 = f (x3 ), . . . formam uma
progressão aritmética de segunda ordem.

Prova: Seja xi+1 − xi = r a razão da progressão aritmética


x1 , x2 , x3 , . . ., então segue que xi+1 = xi + r. Temos que:

yi+1 = f (xi+1 ) = ax2i+1 + bxi+1 + c,


yi = f (xi ) = ax2i + bxi + c,
yi−1 = f (xi−1 ) = ax2i−1 + bxi−1 + c.

e desta forma

di+1 = yi+1 − yi = a(x2i+1 − x2i ) + b(xi+1 − xi )


= a(2rxi + r2 ) + br
di = yi − yi−1 = a(x2i − x2i−1 ) + b(xi − xi−1 )
= a(2rxi−1 + r2 ) + br.

e nalmente

di+1 − di = a(2rxi − 2rxi−1 ) = 2ar(xi − xi−1 ) = 2ar2

o que mostra que a sequência que d1 , d2 , d3 , . . . é uma progressão


aritmética de razão 2ar2 .

Agora vamos mostrar que vale a reciproca deste resultado.

Proprosição 5.15. Toda função f : R −→ R contínua que trans-


forma progressões aritméticas em progressões aritméticas de se-
gunda ordem é da forma f (x) = ax2 + bx + c.

75
Caracterização das
Funções Quadráticas

Prova: Começamos mostrando que se y1 , y2 , y3 , . . . é uma pro-


gressão aritmética de segunda ordem, existem números reais a, b, c
tais que yn = an2 + bn + c para todo n ∈ N. Assim considerando
a função f (x) = ax2 + bx + c, temos yn = f (n) para todo n ∈ N.
Desta maneira a restrição de f ao números naturais fornece os
termos de uma P.A de segunda ordem dada. Para ver este fato,
observamos que as diferenças sucessivas

d1 = y2 − y1 , d 2 = y3 − y2 , d3 = y4 − y3 , ...

formam uma P.A ordinária, cujo primeiro termo é d = y2 − y1 e


cuja razão chamamos de ri . Portanto seu n−ésimo termo é:

yn+1 = yn = d + (n − 1)ri ,

Sejam para n = 1, 2, 3, . . .. Temos então:


a1 , a2 , a3 , . . . , an , . . .
os termos de uma P.A de
razão r. Então o n-ésimo yn+1 = (yn+1 − yn ) + (yn − yn−1 ) + . . . + (y2 − y1 ) + y1
termo da P.A é dado por
an+1 = a1 + (n − 1)r. = [d + (n − 1)r] + [d + (n − 2)r] + . . . + [d + r] + d + y1
n(n−1)
= nd + 2 r + y1 ,

para todo n ∈ N. Observamos que a última igualdade foi obtida


usando a fórmula da soma para os n−1 inteiros n−1, n−2, . . . , 2, 1.
Considere os n primeiros Substituindo n+1 por n na equação acima ( desde que esta é válida
inteiros 1, 2, 3, . . . , n. En-
tão a fórmula para a soma para n = 0), obtemos:
destes é dada por Sn =
(n+1)n
2
.
(n−1)(n−2)
yn = (n − 1)d + 2 r + y1 ,
= 2r n2 + d − 3r

2 n + r − d + y − 1,
= an2 + bn + c.

para todo n ∈ N, com a = r/2, b = d − 3r/2 e c = r − d + y1 .

76
Matemática para o Ensino Médio I AULA
Exemplo 5.2. A sequência 3, 7, 13, 21, 31, 43, . . . é uma P.A de 5
segunda ordem, pois as diferenças sucessivas

d1 = y2 − y1 = 7 − 3, d2 = y3 − y2 = 13 − 7,
(5.18)
d3 = y4 − y3 = 21 − 13, ...,

formam a P.A ordinária 4, 6, 8, 10, 12, . . . de razão r = 2 e primeiro


termo d = 4. Assim o n-ésimo termo da sequência inicial pode ser
escrito da forma yn = an2 +bn+c. As constantes são determinadas
de acordo com as expressões obtidas na prova do teorema acima,
ou seja, a = r/2 = 1, b = d − 3r/2 = 4 − 3 = 1 e c = r − d + y1 =
2 − 4 + 3 = 1. Assim o termo geral é yn = n2 + n + 1.

Vamos supor que a P.A de segunda ordem é não degenerada, ou


seja, não é uma P.A ordinária. Seja y1 , y2 , y3 , . . . , yn , . . .
uma P.A de segunda or-
dem. Esta é dita degene-
Teorema 5.1. Caracterização das Funções Quadráticas A rada se as diferenças su-
cessivas formam uma se-
m de que a função contínua f : R −→ R seja quadrática é quência constante, ou seja,
necessário e suciente que toda progressão aritmética não cons- uma P.A de razão nula e
assim, a primeira sequên-
tante x1 , x2 , x3 , . . . , xn . . . seja transformada numa progressão arit- cia é uma P.A ordinária

mética de segunda ordem não-degenerada y1 = f (x1 ), y2 = f (x2 ),


y3 = f (x3 ),. . . , yn = f (xn ), . . . .

Prova: A necessidade já foi provada acima. Seja f : R −→ R


função contínua com a propriedade de transformar toda P.A não
constante numa P.A de segunda ordem não degenerada. Fazendo
g(x) = f (x) − f (0), obtemos que g tem as mesmas propriedades
que f e ainda g(0) = 0. Consideremos a P.A 1, 2, 3, . . .. Então
os valores g(1), g(2), g(3), . . . formam uma P.A de segunda ordem
não-degenerada. Logo existem constantes a 6= 0 e b tais que

g(n) = an2 + bn

77
Caracterização das
Funções Quadráticas

para todo n ∈ R. Observe que o termo c = 0 pois g(0) = 0 e por


isso não aparece na expressão acima.
Fixamos arbritariamente um número p ∈ N e consideremos a pro-
gressão aritmética
1 2 3 n
, , ,..., ,...
p p p p
De modo análogo, concluímos que existem constantes a0 6= 0 e b0
tais que
n
g = a0 n2 + b0 n
p
para todo n ∈ N. Assim para todo n ∈ N, temos:
np 
an2 + bn = g(n) = g = a0 (np)2 + b0 (np) = (a0 p2 )n2 + (b0 p)n
p
Portanto as funções quadráticas an2 + bn e (a0 p2 )n2 + (b0 p)n coin-
cidem para todo x = n ∈ N. Como vimos no ínicio do estudo de
função quadrática, isto implica que a = a0 p2 e b = b0 p, ou seja,
a0 = a/p2 e b0 = b/p. Logo para quaisquer números naturais n e p
vale:
n
= a0 n2 + b0 n

g p
a 2
= p2
n + pb n
n 2
+ b( np .
 
=a p

Vemos que as funções contínuas g(x) e ax2 +bx são tais que g(r) =
ar2 + br para todo número racional positivo r = n/p. Segue-se que
g(x) = ax2 + bx para todo número real positivo x. De modo
análogo, considerando-se a P.A −1, −2, −3, . . ., concluiríamos que
g(x) = ax2 + bx para todo x ≤ 0. Logo pondo f (0) = c, temos que
f (x) = g(x) + c, ou seja,

f (x) = ax2 + bx + c

para todo x ∈ R.

78
Matemática para o Ensino Médio I AULA
5.3 Conclusão 5
Vimos uma maneira de caracterizar uma função quadrática por
meio de progressões aritméticas.

RESUMO


Caracterização da Função Quadrática


Denição Uma progressão aritmética de segunda ordem é uma
sequência y1 , y2 , . . . tal que as diferenças sucessivas

d1 = y2 − y1 , d 2 = y3 − y2 , d3 = y4 − y3 , ...

formam uam progressão aritmética usual.


Teorema da Caracterização da Função Quadrática A m de
que a função contínua f : R −→ R seja quadrática é necessário e
suciente que toda progressão aritmética não constante x1 , x2 , x3 ,
. . . , xn . . . seja transformada numa progressão aritmética de se-
gunda ordem não-degenerada y1 = f (x1 ), y2 = f (x2 ), y3 = f (x3 ),
. . . , yn = f (xn ), . . . .

REFERÊNCIAS BIBLIOGRÁFICAS


Lima, E.L., A Matemática no Ensino Médio I, 5.ed., SMB, 2006.

79
AULA

Funções Polinomiais
6
META:
Denir e obter propriedades de funções polinomiais.

OBJETIVOS:
Ao m da aula os alunos deverão ser capazes de:
Identicar e conhecer informações de natureza geral sobre funções
polinomiais.
Calcular uma aproximação para as raízes de um função polinomial.
Aplicar o método de Newton para determinar uma aproximação
para as raízes.

PRÉ-REQUISITOS
Denição de polinômio de grau n, como também conhecimento so-
bre continuidade e diferenciabilidade de funções polinômiais.
Funções Polinomiais

6.1 Introdução

As funções polinomiais, onde as funções lineares e quadráticas são


um caso particular, estão presentes na matemática com várias apli-
cações.

6.2 Funções Polinomiais e Polinômios

Denição 6.1. Dizemos que p : R −→ R é uma função polinomial


quando existem números a0 , a1 , . . . , an tais que, para todo x ∈ R,
tem-se
p(x) = an xn + an−1 xn−1 + . . . + a1 x + a0 . (6.19)

A soma e o produto de funções polinomiais, ainda é uma função


polinomial. Um exemplo interessante de produto é:

(x − α)(xn−1 + αxn−2 + . . . + αn−2 x + αn−1 ) = xn − αn .

Dizemos então que xn − αn é divisível por x − α. Seja p uma


função polinomial como na equação (6.19). Para quaisquer x, α
reais, temos que

p(x) − p(α) = αn (xn − αn ) + αn−1 (xn−1 − αn−1 ) + . . . + a1 (x − α).

Como cada parcela do segundo menbro é divisível por x − α, pode-


mos escrever, para cada x ∈ R,

p(x) − p(α) = (x − α)q(x).

onde q é uma função polinomial.


Em particular, se α é uma raíz de p, isto é, p(α) = 0, então

p(x) = (x − α)q(x).

82
Matemática para o Ensino Médio I AULA
para todo x ∈ R. A recíproca é óbvia. Portanto, α é uma raíz 6
de p se, e somente se, p(x) é divisível por x − α. Mais geralmente
α1 , α2 , . . . , αk são raízes de p se, e somente se, para todo x ∈ R,
vale

p(x) = (x − α1 )(x − α2 ) . . . (x − αk )q(x).

Uma função polinomial p, chama-se identicamente nula quando se


tem p(x) = 0 para todo x ∈ R. Neste caso, p tem uma innidade
de raízes, ou seja, todo número real é uma raiz de p.
Dadas duas funções polinomiais p e q , completando com zeros,
se necessário, os coecientes que faltam, podemos escrever sob as
formas

p(x) = an xn + an−1 xn−1 + . . . + a1 x + a0

q(x) = bn xn + bn−1 xn−1 + . . . + b1 x + b0 .

Suponhamos que p(x) = q(x) para todo x ∈ R, ou seja, que p e


q sejam funções iguais. Então a diferença d = p − q é a função
identicamente nula, pois d(x) = p(x) − q(x) = 0 para todo x ∈ R.
Mas, para todo x ∈ R, tem-se

d(x) = (an −bn )xn +(an−1 −bn−1 )xn−1 +. . .+(a1 −b1 )x+(a0 −b0 ).

Pelo que acabamos de ver sobre funções polinomiais identicamente


nulas, segue-se que an − bn = 0, . . . , a1 − b1 = 0, a0 − b0 =
0. Portanto as funções polinomiais p, q assumem o mesmo valor
p(x) = q(x) para todo x ∈ R se, e somente se, têm os mesmo
coecientes.

83
Funções Polinomiais

Como no caso de funções quadráticas, existe um sutil diferença


entre o conceito de função polinomial e o conceito de polinômio,
que apresentemos agora.

Denição 6.2. Um polinômio é uma expressão formal do tipo

p(X) = an X n + an−1 X n−1 + . . . + a1 X + a0 , (6.20)

onde (a0 , a1 , . . . , an ) é uma lista ordenada de números reais e X


é um símbolo (chamada de indeterminada), sendo X i uma abre-
viatura para X · X · X · . . . · X (i fatores).

Em essência, o polinômio p(X) é o mesmo que a lista ordenada


dos seus coecientes. ao escrevê-lo da maneira acima, estamos
deixando explícita a intenção de somar e multiplicar plonômios
como se fossem funções polinomiais, usando a regra X i ·X j = X i+j .
Por denição, os polinômios

p(X) = an X n + an−1 X n−1 + . . . + a1 X + a0

e
q(X) = bn X n + bn−1 X n−1 + . . . + b1 X + b0 .

são iguais (ou idênticos) quando a0 = b0 , a1 = b1 , . . ., an = bn .


A cada polinômio

p(X) = an X n + an−1 X n−1 + . . . + a1 X + a0 ,

faz-se corresponder a função polinomial p̄ : R −→ R, denida por


p̄(x) = an xn + an−1 xn−1 + . . . + a1 x + a0 , para todo x ∈ R. Esta
correspondência (polinômio)7−→ (função polinomial) é sobrejetiva,
pela própria denicão destas funções. A discussão que zemos
acima sobre os coecientes de funções polinomiais iguais signica

84
Matemática para o Ensino Médio I AULA
que a polinômios distintos correspondem funções polinomiais dis- 6
tintas. Logo trata-se de uma correspondência biunívoca.
Por este motivo, não há necessidade de fazer distinção enter o
polinômio p e a função polinomial p̄. Ambos serão representa-
dos pelo mesmo símbolo p e serão chamados indiferentemente de
polinômio ou de função polinomial. Além disso, diremos a função
p(x) sempre que não houver perrigo de confundí-la com o número
real que é o valor da função assumido num certo ponto x.

6.3 Determinando um Polinômio a partir de seus


Valores

Um polinômio de grau n é dado quando se conhecem seus n + 1


coecientes. Segundo a boa prática matemática, para determinar
n + 1 números é necessário ( e muitas vezes suciente) ter (n + 1)
informações. No nosso caso, vale o seguinte resultado:

Proprosição 6.16. Dados n+1 números reais distintos x0 , x1 , . . . , xn


e xados arbitrariamente os valores y0 , y1 , . . . , yn , existe um, e so-
mente um, polinômio p, de grau ≤ n, tal que

p(x0 ) = y0 , p(x1 ) = y1 , . . . , p(xn ) = yn .

A parte somente um decorre do que já discutimos na seção an-


terior, pois e p e q são polinômios de grau ≤ n que assumem os
mesmo valores em n + 1 pontos distintos então a diferença p − q
é um polinômio de grau ≤ n com n + 1 raízes, logo p − q = 0 e
p = q.
A existência de um polinômio p de grau ≤ n que assume valores
pré-xados em n + 1 pontos distintos dados consiste em resolver o

85
Funções Polinomiais

ssitema de n + 1 equações nas n + 1 incógnitas a0 , a1 , . . . , an . Mais


precisamente o sistema é dado pelas equações

an xn0 + an−1 xn−1


0 + . . . + a1 x0 + a0 = y0 ,
an xn1 + an−1 xn−1
1 + . . . + a1 x1 + a0 = y1 ,
..
.
an xnn + an−1 xn−1
n + . . . + a1 xn + a0 = yn .

Este sistema, no qual as quantidades conhecidas são as potências


sucessivas de x0 , x1 , . . . , xn , tem sempre solução única quando estes
n + 1 números são dois a dois diferentes. ( Seu determinante é o
Y
determinante de Vandermonde, igual (xi − xj ).)
i<j

Exemplo 6.1. Pondo x0 = −1, x1 = 0, x2 = 1, x3 = 2, x4 = 3


e procurarmos o polinômio de grau ≤ 4 que assume os valores
−7, 1, 5, 6, 25 respectivamente, obteremos

p(x) = x3 − 2x2 + 5x + 1,

que tem grau 3.

6.4 Gráfico de Polinômios

Quando se deseja traçar, ao menos aproximadamente, o gráco de


um polinômio, certas informações de natureza geral são de grande
utilidade. Vejamos algumas delas.
Seja p(x) = an xn + an−1 xn−1 + . . . + a1 x + a0 com an 6= 0. Se
n é par, então para |x| sucientemente grande, p(x) tem o mesmo
sinal de an . Este sinal é, portanto, o mesmo, não importando se
x < 0 ou x > 0, desde que |x| seja sucientemente grande. Se
entretanto, n é ímpar, p(x) tem o mesmo sinal de an para valores

86
Matemática para o Ensino Médio I AULA
positivos muito grandes de x e tem o sinal oposto de an para valores 6
negativos muito grande de x.

Em ambos os casos, quando |x| cresce ilimitadamente, |p(x)| tam-


bém cresce ilimitadamente. Vejamos abaixo dois exemplos.

Figura 6.1: Gráco da função p(x) = x5 − 7x4 + 4x3 − 2x2 + x − 4.


As raízes de polinômios
de segundo grau foram
Outra informação útil diz respeito à comparação entre dois po- expressas em função
de seus coecientes há
linômios. Se o grau de p é maior do que o grau de q então, milênios. em meados
do século XV I , foram
para todo x com valor absoluto sucientemente grande, têm-se obtidas fórmulas para ex-
|p(x)| > |q(x)|. Mais ainda, a diferença entre |p(x)| e |q(x)| pode primir, mediante radicais
as raízes de polinômios de
se tormar tão grande quanto se queira, desde que se tome |x| su- terceiro e quarto graus em
função dos coecientes.
cientemente grande. Mais um dado relevante para traçar o gráco
de um polinômio é a localização de sua raízes. É claro que, por
motivo da continuidade, se p(x1 ) < 0 e p(x2 ) > 0 então p deve
possuir uma raiz entre x1 e x2 . Esta observação já assegura que
todo polinômio de grau ímpar possuí ao menos uma raíz real. A
questão que surge é como localizar estas raízes.

87
Funções Polinomiais

Figura 6.2: Gráco da função x6 − 7x4 + 4x3 − 2x2 + x − 4.

Os métodos que se usam atualmente para determinar uma raiz do


polinômio p localizada no intervalo [a, b], quando se sabe que p(a)
e p(b) tem sinais opostos não se baseiam em fórmulas fechadas,
como foram obtidas para as equações de grau ≤ 4. Em vez disso,
esses métodos se baseiam em algorítmos aproximativos, os quais
instruem, passo a passo, como proceder para obter uma sequên-
cia de números x1 , x2 , . . . , xn , . . . tais que os valores p(x1 ), p(x2 ),
. . . , p(xn ), . . . estão cada vez mais próximos de zero.
Um exemplo de algorítmo grandemente eciente, para obter uma
raíz da equação p(x) = 0 é o Método de Newton. Segundo este
método, se x1 é um valor próximo de uma raiz, a sequência x1 , x2 ,
. . . , xn , . . . de números reais obtidos pela fórmula iterativa
p(xn )
xn+1 = xn − ,
p0 (xn )
tem com o limite uma raiz de p. Os termos desta sequência se
aproximam bastante rapidamente do limite. Um caso particular

88
Matemática para o Ensino Médio I AULA
do método de Newton, já era conhecido pelos babilônicos, que 6
calculavam a raiz quadrada de um número positivo a, ou seja,
uma rais da equação x2 − a = 0, tomando um valor inicial x1 e a

partir daí, construir as aproximações x1 , x2 , . . . , xn , . . . de a pela
fórmula iterativa
1 a
xn+1 = xn + .
2 xn
Observação 6.1. No denominador da fórmula de Newton, p0 (x)
representa a derivada do polinômio

p(x) = an xn + an−1 xn−1 + . . . + a1 x + a0 ,

a qual é

p(x) = nan xn−1 + (n − 1)an−1 xn−2 + . . . + 2a2 x + a1 .

RESUMO


Nesta seção, denimos um função polinomial, a qual é uma gene-


ralização da função Am e Quadrática. Vimos que toda função
polinomial

p(x) = an xn + an−1 xn−1 + . . . + a1 x + a0 , an 6= 0

pode ser identicada com o polinômio de grau n: p(X) = an X n +


an−1 X n−1 +. . .+a1 X+a0 . Um resultado importante, sobre funções
polinomias é o seguinte:
Um polinômio de grau n é dado quando se conhecem seus n+1 coe-
cientes. Ou seja, dados n+1 números reais distintos x0 , x1 , . . . , xn
e xados arbitrariamente os valores y0 , y1 , . . . , yn , existe um, e so-
mente um, polinômio p, de grau ≤ n, tal que

p(x0 ) = y0 , p(x1 ) = y1 , . . . , p(xn ) = yn .

89
Funções Polinomiais

Uma maneira de determinar completamente a análise de uma função


polinomial, é obter o gráco desta função. Dentre as informações
de natureza geral, está a de como obter uma boa aproximação para
as raízes do polinômio que representa a função. Um exemplo de
algorítmo grandemente eciente, para obter uma raíz da equação
p(x) = 0 é o Método de Newton. Segundo este método, se x1 é
um valor próximo de uma raiz, a sequência x1 , x2 , . . . , xn , . . . de
números reais obtidos pela fórmula iterativa

p(xn )
xn+1 = xn − ,
p0 (xn )

tem com o limite a rais de p.

ATIVIDADES


Atividade. 6.1. Sejam P (x) e p(x) polinômios não identicamente


nulos, com grau P (x) ≥ grau p(x). Prove que existe um polinômio
q(x) tal que grau [P (x) − p(x)q(x)] < grau P (x). Usando repeti-
vamente este fato, mostre que existem polinômios q(x) e r(x) tais
que P (x) = p(x)q(x) + r(x), com grau r(x) ≥ grau p(x). Os
polinômios q(x) e r(x) tais que P (x) = p(x)q(x) + r(x), com grau
r(x) ≥ grau p(x), chamam-se respectivamente o quociente e o resto
da divisão de P (x) por p(x).

Atividade. 6.2. Prove a unicidade do quociente e do resto, isto é,


se P (x) = p(x)q1 (x) + r1 (x), com grau r1 (x) ≥ grau p(x) e P (x) =
p(x)q2 (x) + r2 (x), com grau r2 (x) ≥ grau p(x) então q1 (x) = q2 (x)
e r1 (x) = r2 (x) para todo x ∈ R.

90
Matemática para o Ensino Médio I AULA
Atividade. 6.3. Verique se é verdadeiro ou falso: Se α é a raiz 6
dupla de p(x) se, e somente se, é raiz simples de p0 (x).

Atividade. 6.4. Determine o polinômio p(x) de menor grau pos-


sível tal que p(1) = 2, p(2) = 1, p(3) = 4 e p(4) = 3.

Atividade. 6.5. Seja p(x) um polinômio cujo grau n é um número


ímpar. Mostre que existem números reais x1 , x2 tais que p(x1 ) < 0
e p(x2 ) > 0. Conclua daí que todo polinômio de grau ímpar admite
pelo menos uma raíz real.

Atividade. 6.6. Tomando x0 = 3 use a relação de recorrência

1 5 
xn+1 = xn + .
2 xn

para calcular 5 com três algarismos decimais exatos.

Atividade. 6.7. Usando o método de Newton, estabeleça um



processo interativo para calcular 3 a e aplique-o a m de obter um

valor aproximado de 3 2.

REFERÊNCIAS BIBLIOGRÁFICA


Lima, E.L., A Matemática no Ensino Médio I, 5.ed., SMB, 2006.

91
AULA

Funções Logarítmicas
7
META:
Denir e obter propriedades de funções polinomiais.

OBJETIVOS:
Ao m da aula os alunos deverão ser capazes de:
Identicar e conhecer informações de natureza geral sobre funções
polinomiais.
Calcular uma aproximação para as raízes de um função polinomial.
Aplicar o método de Newton para determinar uma aproximação
para as raízes.

PRÉ-REQUISITOS
Denição de polinômio de grau n, como também conhecimento so-
bre continuidade e diferenciabilidade de funções polinômiais.
Funções Logarítmicas

7.1 Introdução

Os logaritmos são estudados a longa data, e desempenharam um


papel importante como instrumento de simplicar o cálculo aritmé-
tico, permitindo que se efetuasse, por exemplo, a multiplicação de
dois números com muitos algarismos. Porém, na atualidade, este
papel se torna obsoleto, dado o surgimento de ecientes máquinas
de calcular. Mas os logaritmos continuam a merecer destaque
no ensino da matemática. A função logarítmica, e sua inversa,
a função exponencial, constituem a maneira única de descrever
matematicamente uma grandeza cuja taxa de crescimento, ou de-
crescimento, é proporcional à quantidade daquela grandeza exis-
tente nun dado momento.

Vamos inverter a ordem da apresentação das funções exponen-


ciais e logarítmicas em relação aos textos tradicionais, que de-
nem função exponencial e depois função logarítmica como sua
inversa. Assim, não vamos fazer uso da função de logaritmos usual-
mente apresentada nos livros de ensino, onde uma maneira denir
a função L : R+ −→ R consiste em por L(x) = y se, e somente se,
ay = x , ou seja, chamar de logaritmo de x na base a ao expoente y
ao qual se deve elevar a base a para obter x. Para fugir de alguns in-
convinientes desta denição, vamos apresentar uma denição geo-
métrica dos logaritmos que apresenta vantagens do ponto de vista
conceitual e técnica como veremos no decorrer da aula. A denição
geométrica depende apenas do conceito de área de uma gura plana
e a propriedade fundamental L(xy) = L(x) + L(y) resulta mera-
mente do fato de que a área de um retângulo não se altera quando
se muultiplica sua base por um número e se divide a altura pelo

94
Matemática para o Ensino Médio I AULA
mesmo número. 7
7.2 Definição e Propriedades da Função Loga-
rítmica

Nesta aula, daremos a denição de função logarítmica, estabele-


cendo suas propriedades básicas e mostraremos que, a menos de um
fator constante, duas quasiquer funções logarítmicas coincidem.

Denição 7.1. Um Sistema de Logaritmos ou uma função loga-


rítmica é uma função real L : R+ −→ R, cujo domínio é o con-
junto R+ dos números reais positivos, que satisfaz as seguintes
propriedades:
A) L é uma função crescente, isto é, x < y =⇒ L(x) < L(y);
B) L(xy) = L(x) + L(y) para quaisquer x, y ∈ R+ .

Para todo x ∈ R+ , o número L(x) chama-se logaritmo de x. ( Se


estivermos contemplando outras funções logarítmicas além de L,
diremos que L(x) é o logaritmo de x segundo L.)

Observação 7.1. Na Denição 7.1 podemos assumir que a função


L seja decrescente. Porém as propriedades que enunciaremos a
seguir, vamos asumir a hipótese que L é crescente. No caso em que
L é descrescente estas propriedades devem ser reescritas, fazendo
as alterações quando necessário.

No que segue, vamos apresentar algumas propriedades das funções


logarítmicas, que são consequência imediata das condições A) e B)
da denição.

Propriedade 7.1. Uma função logarítmica L : R+ −→ R é sem-


pre injetiva.

95
Funções Logarítmicas

Com efeito, sejam x, y ∈ R+ distintos, então ou x < y ou y < x.


No primeiro caso, resulta de A) que L(x) < L(y) e no segundo
caso tem-se L(y) < L(x). Em qualquer um dos casos, de x 6= y
Uma denição para uma conclui-se que L(x) 6= L(y).
função injetiva pode ser
dada da seguinte forma:
Seja f : R −→ R então f é Propriedade 7.2. O logaritmo de 1 é zero.
injetiva se sempre que x 6=
y implica que f (x) 6= f (y).
De fato, por B) temos

L(1) = L(1.1) = L(1) + L(1),

logo L(1) = 0.

Propriedade 7.3. Se x > 1 (x<1) então L(x) > 0 (L(x) < 0),
ou seja, os números maiores que um têm logarítmos positivos e os
números positivos menores que um têm logarimos negativos.

Sendo L crescente, de 0 < x < 1 < y resulta que L(x) < L(1) <
L(y), isto é, L(x) < 0 < L(y).

Observação 7.2. Se supormos que L é descrescente, então a pro-


priedade acima, assume a seguinte formulação:
Se x < 1 então L(x) > 0 e se x > 1 então L(x) < 0.
Para ver este fato, basta ver que se 0 < x < 1 < y resulta que
L(x) > L(1) > L(y) ou seja, L(x) > 0 > L(y).

Propriedade 7.4. Para todo x > 0, tem-se L(1/x) = −L(x).

Com efeito, de x.(1/x) = 1 resulta que L(x) + L(1/x) = L(1) = 0,


donde L(1/x) = −L(x).

Propriedade 7.5. Para quaisquer x, y ∈ R+ vale:

L(x/y) = L(x) − L(y).

96
Matemática para o Ensino Médio I AULA
Com efeito, 7
L(x/y) = L(x.(1/y)) = L(x) + L(1/y) = L(x) − L(y),

donde na ultima igualdade, usamos a propriedade anterior.

Propriedade 7.6. Para todo x ∈ R+ e todo número racional r =


p/q tem-se L(xr ) = rL(x).

Primeiramente, observamos que segue da condição B) da denição


aplicada n vezes que:

L(xn ) = L(x·x·x·. . .·x) = L(x)+L(x)+L(x)+. . .+L(x) = n·L(x)

para todo n ∈ N. Logo a Propriedade 7.6 vale quando r = n é


um número natural. Claramente também vale quando n = 0, pois
L(x0 ) = L(1) = 0 = 0.L(x). Seja r = −n, n ∈ N, ou seja, r é um
inteiro negativo. Então para todo x > 0, temos xn · x−n = 1. Logo

L(xn ) + L(x−n ) = L(xn · x−n ) = L(1) = 0,

e assim
L(x−n ) = −L(xn ) = −nL(x)

Logo a propriedade vale quando r é um número inteiro. No caso


em que r = p/q , onde p ∈ Z e q ∈ N, para todo x ∈ R+ , temos

(xr )q = (xp/q )q = xp .

Logo q · L(xr ) = L((xr )q ) = L(xp ) = pL(x), pelo que prova-


mos acima. Da igualdade q · L(xr ) = pL(x), resulta que L(xr ) =
p/qL(x) = rL(x).

Propriedade 7.7. Uma função logarítmica L : R+ −→ R é ilimi-


tada superior e inferiormente.

97
Funções Logarítmicas

Seja B um número real e n um número natural n tão grande que


n > B/L(2). Como L(2) > 0 (pela Propriedade 7.3), temos que n ·
L(2) > B . Usando a Propriedade 7.5, vemos que n · L(2) = L(2n ).
Portanto, L(2n ) > B . Agora é só escolher x = 2n e desta maneira,
Seja f : R+ −→ R, então teremos L(x) > B , o que mostra que L é ilimitada superiormente.
dizemos que f é ilimitada
superiormente (respecti- Seja A um número real, pelo que vimos acima podemos encontrar
vamente, inferiormente)
se dado arbitrariamente x ∈ R+ tal que L(x) > −A. Então pondo y = 1/x, teremos
A (respectivamente
B ), números reais, é
L(y) = −L(x) < A (usamos que L(1/x) = −L(x)). Isto mostra
possível encontrar x
que L é ilimitada inferiormente.
(respectivamente y)
números positivos, tais Seja L : R+ −→ R uma função logarítmica e c uma constante
que L(x) < A (respectiva-
mente L(x) > B . positiva arbitrária. Então a função M : R+ −→ R, denida por
M (x) = c · L(x) é também uma função logarítmica. Este fato é
facilmente vericado, pois se L satisfaz as condições A) e B) então
M satisfaz A) e B).

Teorema 7.1. Dadas as funções logarítmicas L, M : R+ −→ R,


existe uma constante c > 0 tal que M (x) = cL(x) para todo x > 0.

Prova: Suponhamos que existe um inteiro a > 1 tal que L(a) =


M (a), mostraremos que neste caso L(x) = M (x) para todo x > 0.
Observe que de L(a) = M (a), concluímos que L(ar ) = M (ar ).
Suponhamos, por absurdo, que existe algum b > 0 tal que L(b) 6=
M (b). Para zar idéias, suponhamos L(b) < M (b). Escolhemos
um número natural n tão grande de maneira que:

n[M (b) − L(b)] > L(a).

Então
L(a1/n ) = L(a)/n < M (b) − L(b).

Escrevemos c = L(a1/n ). Os números c, 2c, 3c, . . . dividem R+


em intervalos justapostos, de mesmo comprimento c. Como c <

98
Matemática para o Ensino Médio I AULA
M (b)−L(b), pelo menos um desses números, digamos m·c pertence 7
ao interior do intervalo (L(b), M (b)), ou seja, L(b) < m · c < M (b).
Ora,

m · c = m · L(a1/n ) = L(am/n ) = M (am/n ).

Então

L(b) < L(am/n ) = M (am/n ) < M (b).

Como L é crescente, a primeira das desigualdades acima implica


b < am/n . Por outro lado, como M também é crescente, a segunda
desigualdade implica que am/n < b. Esta contradição mostra que
b não existe. Assim M (x) = L(x) para todo x > 0.
Agora dadas L, M funções logarítmicas arbitrárias, temos L(2) > 0
e M (2) > 0, por que 2 > 1 e L, M são crescentes. Seja c =
M (2)/L(2). Consideremos a função logarítmica N : R+ −→ R,
denida por N (x) = c · L(x). como N (2) = [M (2)/L(2)] · L(2) =
M (2), segue que, pelo que provamos acima N (x) = M (x) para
todo x > 0, ou seja, M (x) = cL(x) para todo x > 0, como
queríamos.

Pelo teorema que acabamos de provar, podemos observar que, para


estudar logarítmos, basta obter uma função crescente L : R+ −→ R
tal que L(x · y) = L(x) + L(y). Todas as demais funções logarít-
micas resultarão de L pela multiplicação de uma constante conve-
niente.

Teorema 7.2. Toda função logarítmica L é sobrejetiva, isto é,


dado qualquer número real c, existe um único real positivo x tal
que L(x) = c.

99
Funções Logarítmicas

Para a demonstração do teorema acima, necessitaremos do seguinte


lema técnico, cujos detalhes da demonstração deixaremos para o
Todo número real α tem a aluno.
representação decimal
α = a0 , a1 , . . . , an , . . . Lema 7.1. Seja L : R+ −→ R uma função logarítmica. Dados

= a0 +
a1 an
+. . .+ n +. . . arbitrariamente dois números reais u < v , existe x > 0 tal que
10 10
onde a0 é a parte inteira u < L(x) < v .
e os algarismos an , n ≥ 1
podem assumir os valores
0, 1, . . . , 9. Para todo n ≥ Prova: Fixe um natural n maior que (v − u)/L(2). Ponha c =
0 escrevemos:
L(2)/n. Concluímos que os números inteiros m · c = L(2m/n ),
αn = a0 , a1 , . . . , an
onde m ∈ Z devidem a reta real em intervalos justapostos, cujo
a1 an
= a0 + + ... + n.
10 10 comprimento c é menor que v − u. Assim temos que, pelo menos
Temos então αn ≥ α e α −
αn < 1/10n para todo n ≥ um destes míltiplos m · c = L(2m/n caino intervalo I = (u, v).
0. Também, se um número
real x < α, então existe Finalmente pondo x = 2m/n , temos que u < L(x) < v .
n ≥ 0 tal que x < αn . De
fato, x < α signica que
α − x é um número real Com o lema acima, concluímos que todo intervalo aberto I = (u, v)
positivo. Tomemos n tão
grande que 1/10n < α − x contém ao menos um valor L(x).
e então teremos α − αn <
1/10n < α−x e daí resulta Prova do Teorema 7.2: Seja α um número real. Determinamos
que x < αn .
sua representação decimal

α = a0 , a1 , a2 , . . . , an , . . .

Para determinar a parte inteira a0 , lembramos que L é uma função


crescente ilimitada. Logo existem inteiros k tais que L(k) > b.
Seja a0 + 1 o menor inteiro tal que L(a0 + 1) > b. Então temos
L(a0 ) ≤ b < L(a0 + 1).
Em seguida consideremos os números

1 2 9
a0 , a0 + , a0 + , . . . , a0 + , a0 + 1.
10 10 10

Como L(a0 ) ≤ b < L(a0 + 1), devem existir dois elementos con-
secutivos α1 e α1 + 1/10, nessa sequência, tais que L(α1 ) ≤ b <

100
Matemática para o Ensino Médio I AULA
L(α1 + 1/10), isto é, deve existir a1 inteiro, 0 ≤ a1 ≤ 9, tal que, 7
pondo
a1
α1 = a0 , a1 = a0 +
10
temos que L(α1 ) ≤ b < L(α1 +1/10). Analogamente considerando O método usado para de-
terminar a representação
os números decimal do número real

1 2 9 1 α = a0 , a1 a2 . . . an
a1 , a1 + , a1 + 2 , . . . , a1 + 2 , a1 + ,
10 2 10 10 10 é uma versão moderna
de um processo milenar
vemos que existe a2 , 0 ≤ a2 ≤ 9 tal que pondo para resolução de equações
que os chineses antigos
a1 a2 chamavam o método do e-
α2 = a0 , a1 , a2 = a0 + + , lemento celestial.
10 102

Têm-se L(α2 ) ≤ b < L(α2 + 1/102 ).


Procedendo desta maneira, encontramos uma representação deci-
nal de um número real

a1 an
α = a0 , a1 , . . . , an , . . . = a0 + + ... + n + ...,
10 10

tal que pondo αn = a − 0, a1 , a2 , . . . , an , temos

L(αn ) ≤ b < L(αn + 1/10n )

para todo n ≥ 0.
Armamos agora que L(α) = b. De fato, se L(α) < b, usaríamos
o Lema 7.1 para obter x > 0 tal que L(α) < L(x) < b. Como L
é crescente, segue que α < x. Então tomando n sucientemente
grande tal que x − α > 1
10n teríamos α + 1/10n < x e portanto

1 1
αn + n
≤ α + n < x.
10 10

Como L é crescente, de x > αn + 1/10n resultaria

1 
L(x) > L αn + > b,
10n

101
Funções Logarítmicas

um absurdo, pois o número x foi obtido de modo que L(x) <


b. Analogamente não podemos ter L(α) > b. De fato, usando
novamente o Lema 7.1, teríamos x > 0 tal que

b < L(x) < L(α).

Como l é crescente, de L(x) < L(α) concluiríamos que x < α. As-


sim x < αn para algum n. Então L(x) < L(αn ) ≥ b, contrariando
o fato de que x foi obtido de modo a satisfazer b < L(x).

Corolário 7.1. Toda função logarítmica L : R+ −→ R é uma


correspondência biunívoca (bijeção) entre R+ e R.

Segue ainda do Teorema 7.2 que, dada a função logarítmica L :


R+ −→ R, existe um único número a > 0 tal que L(a) = 1.
Este número é chamado de base do sistema de logaritmos L. Para
explicitar tal base, muitas vezes se escreve La (x) em vez de L(x).
Se Lb e La são duas funções logarítmicas, com La (a) = Lb (b) = 1
então pelo Teorema 7.1 existe c > 0 constante tal que Lb (x) =
c · La (x) para todo x > 0. Pondo x = a, resulta que Lb (a) = c.
Portanto temos que

Lb (x) = Lb (a) · La (x)

para todo x > 0. Esta é a fórmula mudança de base de logaritmos.

Observação 7.3. A notação usual para o ;logaritmo de um número,


na base x é loga (x).

RESUMO


102
Matemática para o Ensino Médio I AULA
Denição: Um Sistema de Logaritmos ou uma função logarít- 7
mica é uma função real L : R+ −→ R, cujo domínio é o con-
junto R+ dos números reais positivos, que satisfaz as seguintes
propriedades:
A) L é uma função crescente, isto é, x < y =⇒ L(x) < L(y);
B) L(xy) = L(x) + L(y) para quaisquer x, y ∈ R+ .
A função Logarítmica tem as seguintes propriedades:

1. Uma função logarítmica L : R+ −→ R é sempre injetiva.

2. O logaritmo de 1 é zero.

3. Se x > 1 (x<1) então L(x) > 0 (L(x) < 0), ou seja, os


números maiores que um têm logarítmos positivos e os números
positivos menores que um têm logarimos negativos.

4. Para todo x > 0, tem-se L(1/x) = −L(x).

5. Para quaisquer x, y ∈ R+ vale:

L(x/y) = L(x) − L(y).

6. Para todo x ∈ R+ e todo número racional r = p/q tem-se


L(xr ) = rL(x).

7. Uma função logarítmica L : R+ −→ R é ilimitada superior e


inferiormente.

Temos os seguintes resultados para funções logarítmicas:


Teorema: Dadas as funções logarítmicas L, M : R+ −→ R,
existe uma constante c > 0 tal que M (x) = cL(x) para todo x > 0.
Teorema: Toda função logarítmica L é sobrejetiva, isto é, dado
qualquer número real c, existe um único real positivo x tal que
L(x) = c.

103
Funções Logarítmicas

Corolário: Toda função logarítmica L : R+ −→ R é uma corre-


spondência biunívoca (bijeção) entre R+ e R.

REFERÊNCIAS BIBLIOGRÁFICAS


IEZZI, Gelson., Fundamentos de Matemática Elementar: Logarti-


mos vol.2, 3.ed., Editora Atual, 1977.
Lima, E.L., A Matemática no Ensino Médio I, 5.ed., SMB, 2006.

104
AULA

Função Logarítmo Natural 8


e Área de Faixas de Hipérboles

META:
Dar uma interpretação geométrica ao conceito de Função Logarít-
mica Natural através da área de regiões que consistem em faixas
de hipérboles.

OBJETIVOS:
Ao m da aula os alunos deverão ser capazes de:
Denir a função logarítmica natural através da área de uma faixa
de hipérbole.
Obter e interpretar as propriedades da função Logarítmo Natural,
na sua forma geométrica.

PRÉ-REQUISITOS
Noções de área entre curvas e denições de funções Logarítmicas.
Função Logarítmo Natural
e Área de Faixas de Hipérboles

8.1 Introdução

Nesta aula, vamos denir a função Logarítmo Natural de uma


forma geométrica e para isso começamos por introduzir a denição
de área de uma faixa de hipérbole. A concepção geométrica de
uma função logarítmica é uma idéia muito antiga. Além de antiga,
ela é natural, intuitiva e instrutiva porque constitui uma excelente
introdução ao cálculo integral.

8.2 Área de uma Faixa de Hipérbole

Figura 8.1: Ramo de hipérbole H da função y = 1/x.

Consideremos um sistema de eixos cartesianos xado no plano.


Seja H o ramo positivo do gráco da função y = 1/x, isto é, da
função que associa a cada número real x o número 1/x. Então,
1
H = {(x, y); x > 0, y = }
x
Denição 8.1. Uma faixa de hipérbole é obtida quando xamos
dois números reais positivos a, b, com a < b, e tomamos a região

106
Matemática para o Ensino Médio I AULA
do plano limitada pela duas retas verticais x = a, x = b, pelo eixo 8
da abscissas, e pela hipérbole H . Indicaremos essa região pelo
símbolo Hab .

Figura 8.2: Faixa de hipérbole Hab .

Figura 8.3: Polígono retangular inscrito na faixa Hab .

Nestas condições, na notação da teoria dos conjuntos

1
Hab = {(x, y); a ≤ x ≤ b, 0 ≤ y ≤ }
x

107
Função Logarítmo Natural
e Área de Faixas de Hipérboles

Agora vamos mostrar como proceder a m de calcular a área


de uma faixa Hab . Por meio de pontos intermediários a0 = a,
a1 , a2 , . . ., an = b, decompomos o intervalo [a, b] num número nito
de intervalos justapostos. Com base em cada um dos intervalos
[ai , aj ] da decomposição, (onde i < j ) consideramos o retângulo
de altura igual a 1/d. O vértice superior direito desse retângulo
toca a hipérbole H . É o que chamaremos um retângulo inscrito na
faixa Hab . A reunião desse retângulos cosntitui o que chamaremos
um polígono retangular inscrito na faixa Hab .

Exemplo 8.1. Considere a faixa H13 . Tomamos a decomposição


do intervalo [1, 3] através dos pontos intermediários 1, 3/2, 2, 5/2, 3.
Obtemos assim, um polígono regular cuja área é igual a soma das
áreas dos quatro retângulos hachurados, conforme a Figura 8.4, ou
seja:
A = ( 21 · 23 ) + ( 12 · 12 ) + ( 12 · 25 ) + ( 12 · 13 )
1 1 1 1 57
= 3 + 4 + 5 + 6 = 60

Agora façamos uma subdivisão mais na do intervalo [1, 3], ou

Figura 8.4: Uma aproximação para a área de H13 .

108
Matemática para o Ensino Médio I AULA
seja aumentamos o número de pontos intermediários, por meio dos 8
pontos
5 6 8 9 10 11
1, , , , , , , 3.
4 4 4 4 4 4
Obtemos assim um polígono retangular inscrito em H13 , formado
por 8 retângulos justapostos, cuja área total vale

Figura 8.5: Uma aproximação melhor para a área de H13 .

1 1 1 1 1 1 1 1 84.813
A= + + + + + + + = ∼ 1, 019.
5 6 7 8 9 10 11 12 83.160
Cada polígono retangular inscrito na faixa Hab fornece um valor
aproximado por falta para a área de Hab . Tanto mais será apro- Observe que a maneira que
denimos a área, é exa-
ximado este valor quanto mais na for a subdivisão do intervalo tamente da mesma forma
que denimos a soma de
[a, b]. Isto é, quanto mais próximos estiverem uns dos outros os Riemann de uma função
f em relação a uma par-
pontos da subdivisão, menor será a diferença entre o valor exato tição, ou seja, em relação
da área de Hab do seguinte modo: a uma subdivisão do in-
tervalo [a, b]. Assim uma
soma de Riemann é a soma
Denição 8.2. A área de Hab é o número real cujas aproximações das áreas dos retângulos
inscritos no gráco de f ,
por falta são as áreas dos polígonos retangulares inscritos em Hab . ou seja, a área do polígono
retangular P .
Se escrevemos A = área de Hab , teremos A ≥ área de P , qualquer
que seja o polígono retangular inscrito em Hab .

109
Função Logarítmo Natural
e Área de Faixas de Hipérboles

Além disso, renando sucientemente a subdivisão do intervalo


[a, b], podemos obter polígonos retangulares cujas áreas seja tão
próximas da área de Hab quanto se deseje. Mais precisamente, dado
qualquer número α < área de Hab , existe um polígono P inscrito
em Hab tal que α < área de P < área de Hab .
Podemos também dizer que a área de Hab é o extremo superior
do conjunto das áreas dos polígonos retangulares inscritos em Hab .
Isto signica que A = área de Hab é o menor número real tal que
A ≥ área de P para todo polígono P inscrito em [a, b]. Dizer que
A é o extremo superior do conjunto das áreas dos polígonos retan-
gulares P inscritos em Hab tem exatamente o mesmo siginicado
que armar que os valores aproximados por falta da área Hab são
as áreas dos polígonos retangulares inscritos nesta faixa.

8.3 Propriedade Fundamental

O fato mais importante a respeito das áreas de hipérbole é expresso


pelo teorema abaixo:

Teorema 8.1. Para todo real bk têm a


k > 0, as faixas Hab e Hak
mesma área.

Prova: Seja R1 o retângulo inscrito em H , cuja base é o segmento


[c, d] do eixo das abcissas, e R2 o retângulo inscrito em H , cuja
base é [kc, kd] sobre o eixo das abcissas. Então área de R1 é igual
a área de R2 . De fato, a área de R1 é dada por

1 c
A(R1 ) = (d − c) · =1− ,
d d

enquanto que a área de R2 é

110
Matemática para o Ensino Médio I AULA
8
1 c
A(R2 ) = (kd − kc) · =1− ,
dk d

Consideramos agora um polígono retangular P , inscrito em Hab .

Figura 8.6: Os retângulos R1 e R2 que possuem a mesma área.

Se multiplicarmos por k cada uma das abcissas dos pontos de sub-


divisão de [a, b], determinados por P , obteremos uma subdivisão
do intervalo [ak, bk] e portanto um polígono P 0 , inscrito na faixa
bk . Cada um dos retângulos que compõem P 0 tem a mesma área
Hak
que o retângulo correspondente em P . Logo a área de P 0 é igual
à de P . Concluímos que para cada polígono retangular inscrito
em Hab , existe um inscrito em Hak
bk com a mesma área. Analoga-

mente (dividindo abcissas por k ) veríamos que, para cada polígono


retangular Q0 inscrito em Hak
bk existe outro Q, de mesma área, in-

scrito em Hab . Isto signica que as áreas destas duas faixas são
números que possuem exatamente as mesmas aproximações infe-
riores, e portanto são iguais.

Uma consequência do teorema acima, é que podemos restringir

111
Função Logarítmo Natural
e Área de Faixas de Hipérboles

nossa consideração às áreas das faixas da forma H1c , pois


b/a
A(Hab ) = A(H1 ) = A(H1c ), c = b/a.

Quando a < b < c, temos que

A(Hab ) + A(Hbc ) = A(Hac ). (8.21)

cuja vericação é imediata, pois basta observar que a faixa Hac é a


justaposição das faixas Hab e Hbc .
Normalmente, a área de uma gura não é um número negativo.
Mas as vezes é conviniente usar áreas orientadas, ou seja, providas
de sinal + ou −. Convencionaremos que a área da faixa de hipér-
bole Hab será positiva quando a < b, negativa quando a > b e zero
quando a = b. Para deixar mais clara esta convenção, escrevemos

AREA Hab

com letras maíusculas, para indicar que a área orientada (provida


de sinal). A área usual, com valores ≥ 0, sera escrita como A =
área Hab . Assim

AREA (Haa ) = 0;
AREA (Hab ) = A(Hab ), se a < b; (8.22)
AREA (Hab ) = −A(Hab ), se b < a.
Observação 8.1. Observe que a última convenção da equação
(8.22) implica em considerar áreas negativas. Isto de uma certa
forma, contraria a tradição mas, em compensação, a igualdade
acima, dada pela Equação (8.21) torna-se válida sem restrições.

Observação 8.2. O Teorema 8.1 continua válido mesmo com esta


convenção de sinais. De fato, ainda que se tenha b < a, será
também bk < ak pois k > 0. Portanto, se for b < a teremos

A(Hba ) = −A(Hab ) = −A(Hak


bk ak
) = A(Hbk ). (8.23)

112
Matemática para o Ensino Médio I AULA
8.4 Função Logarítmo Natural 8
Nesta seção vamos denir a função logarítmo natural l : R+ −→ R
de uma maneira geométrica e independente da função logarítmo
em uma base qualquer.

Denição 8.3. Seja x ∈ R+ . O logaritmo natural de x, denotado


por ln(x), é a área da faixa de hipérbole H1x , ou seja,

ln(x) = AREA (H1x ).

Lembramos que pela convenção de sinal que zemos teremos que


A(H1x ) < 0 sempre que 0 < x < 1, ou seja, ln(x) < 0 sempre que
0 < x < 1. Segue também que ln(x) > 0 sempre que x > 1 e que
ln(x) = 0 quando x = 1, pois em x = 1 temos A(H11 ) = 0.

Denição 8.4. A função Logarítmo Natural é a função ln : R+ −→


R, que faz corresponder a cada número real x > 0 o número
f (x) = AREA (H1x )

Figura 8.7: Logaritmo Natural.

113
Função Logarítmo Natural
e Área de Faixas de Hipérboles

Teorema 8.2. A função ln : R+ −→ R é uma função logarítmica.


Prova: Devemos mostrar que a função ln goza das propriedades
A) e B) da denição de um sistema de logaritmos. Começamos
provando A). Já provamos acima os seguintes fatos:

A(Hxxy ) = A(H1x ) + A(Hxxy )


(8.24)
A(Hxxy ) = A(H1y )
onde o primeiro termo da Equação (8.24) acima, vale para qual for
a posição relativa dos pontos abcissa 1, x, xy sobre o eixo horizon-
tal. Assim segue que

A(H1xy ) = A(H1x ) + A(H1y ), (8.25)

isto é,
ln(xy) = ln(x) + ln(y) (8.26)

e portanto, vale a propriedade A). Provamos agora que ln é uma


função crescente. Dados x, y ∈ R+ , dizer que x < y signica dizer
que existe um número real a > 1 tal que y = ax. Segue que

ln(y) = ln(ax) = ln(a) + ln(x). (8.27)

Como a > 1, segue que ln(a) > 0. Portanto ln(y) > ln(x), o que
prova a propriedade B).
Alguns aurotres chamam o
logaritmo natural de log-
aritmo neperiano, em ho- Observação 8.3. Da denição de um sistema de logaritmos, segue
menagem a John Napier,
autor da primeira tábua de que existe um número real positivo, e tal que
logaritmos, em 1614. En-
tretanto, tal denominação Le (x) = ln(x)
não é inteiramente apro-
priada, pois o logaritmo
ou seja, e é a base do sistema de logaritmos. O número e, base
originalmente denido por
Napier não coincide com o dos logarítmos naturais, é caracterizado pelo fato que seu logaritmo
logaritmo natural
natural é igual a 1, ou seja, AREA H1e = 1. O número e é irracional
e um valor aproximado para esta constante é e = 2, 718281828459.

114
Matemática para o Ensino Médio I AULA
8.4.1 Gráfico da função ln 8
O gráco da função ln é o conjunto

G = {(x, ln(x)); x > 0}

Lembramos que pelo fato de ln ser uma função logarítmica, é uma

função crescente, ilimitada nos dois sentidos (superior e inferior-


mente) e sobrejetiva. Estes fatos mostram que o gráco de ln é
uma curva contida no primeiro e quarto quadrantes, que corta o
eixo da abcissas no ponto x = 1 e que, quando x varia entre 0 e
+∞, a ordenada do ponto (x, ln(x)) sobre a curva cresce de −∞
a +∞.

RESUMO


Função Logaritmo Natural: Forma Geométrica


Denição: Uma faixa de hipérbole é obtida quando xamos dois
números reais positivos a, b, com a < b, e tomamos a região do
plano limitada pela duas retas verticais x = a, x = b, pelo eixo da

115
Função Logarítmo Natural
e Área de Faixas de Hipérboles

abscissas, e pela hipérbole H . Indicaremos essa região pelo símbolo


Hab . Então
1
Hab = {(x, y); a ≤ x ≤ b, 0 ≤ y ≤ }
x
Denição: A área de Hab é o número real cujas aproximações
por falta são as áreas dos polígonos retangulares inscritos em Hab .
Teorema: Para todo real k > 0, as faixas Hab e Hak
bk têm a mesma

área.
Denição: A função Logarítmo Natural é a função ln : R+ −→ R,
que faz corresponder a cada número real x > 0 o número f (x) =
AREA (H1x ).

ATIVIDADES


Atividade. 8.1. Mostre que o número e pode ser introduzido


como e := lim (1 + 1/n)n .
n−→∞
(a)- Para isso, primeiramente considere a faixa H11+x (x > 0) e
os retângulos inscritos e escritos nesta faixa, ambos com a mesma
base [1, 1 + x]. Prove que
x
< ln(1 + x) < x.
1+x
(b)-Considere −1 < x < 0 e assim a faixa H1+x
x . Considerando o

retângulo de base −x e altura 1 inscrito na faixa H1+x


x e o retângulo
de mesma base e algura 1/(1 + x). Segue que
x
−x < − ln(1 + x) < − .
1+x
Use esta desigualdade, para mostrar que e := lim (1 + 1/n)n
n−→∞
quando −1 < x < 0.

116
Matemática para o Ensino Médio I AULA
(c)- Faça x = a/n emostre que ea = lim (1 + a/n)n .
n−→∞
8
Atividade. 8.2. Considere a parábola y = x2 . Dena a área
A(x2 )ba da faixa dessa parábola. Divida o intervalo [0, 1] em n + 1
partes iguais. Mostre que o polígono retangular inscrito na faixa
da parábola y = x2 tem área igual a

1
(1 + 22 + 32 + ...n2 ).
(n + 1)3

Fazendo o uso da formula 1+22 +32 +. . .+n2 = n3 /3+n2 /2+n/6,


verique que

1
area Pn+1 = 1/3 (1 + 3/2n + 1/2n2 ).
(1 + 1/n)3

Conclua que A(x2 )10 = 1/3. Isto prova o Teorema de Arquimedes,


segundo o qual a área do triângulo prabólico de base [0, 1] é um
terço do área do quadrado de mesmo lado.

Atividade. 8.3. Mostre que se os números positivos a1 , a2 , . . . , an


forma uma progressão geométrica então os termos ln(a1 ), ln(a2 ), . . . , ln(an )
formam uma progressão aritmética.

Atividade. 8.4. Mostre que para todo x > 0 e todo h > −x não
nulo têm-se que

ln(x + h) − ln(x)
= ln(1 + h/x)1/h
h

Atividade. 8.5. Mostre que a soma

Sp = 1 + 1/2 + 1/3 + 1/4 + . . . + 1/n

é maior que ln(p + 1) e conclua que lim Sp = ∞.


n−→∞

Atividade. 8.6. Mostre que para todo inteiro p > 0, temos

0 < Sp − ln(p + 1) < 1

117
Função Logarítmo Natural
e Área de Faixas de Hipérboles

e também Sp − ln(p + 1) cresce com p.


Obs: O limite lim Sp − ln(p + 1)é chamado constante de Euler.
n−→∞

Atividade. 8.7. Sejam a ≤ x ≤ b números positivos. Prove que

b−x x−a
ln b − ln x ≤ e ln x − ln a ≥ .
x x

conclua que

ln b − ln x b−x ln b − ln a b−a
≤ e ≤ .
ln x − ln a x−a ln x − ln a x−a

Atividade. 8.8. Determine o valor de x nas equações abaixo:


(a)- ln x = ln(a − b) + ln(a + b).
(b)- ln x = 3 ln a + −4 ln b + 5 ln c.
(c)- 1/7 ln r + 2 ln s = 2/3 ln t3 + ln x.

REFERÊNCIAS BIBLIOGRÁFICAS


Lima, E.L., A Matemática no Ensino Médio I, 5.ed., SMB, 2006.


Lima, E.L., Logaritmos, 2.ed., SMB, 1996.

118
AULA

Funções Logarítmicas:
9
Uma Abordagem Geométrica

META:
Dar uma interpretação geométrica ao conceito de Função Logarít-
mica através da área de regiões que consistem em faixas de hipér-
boles.

OBJETIVOS:
Ao m da aula os alunos deverão ser capazes de:
Denir a função logarítmica através da área de uma faixa de hipér-
bole.
Obter e interpretar as propriedades da função Logarítmo através
da função logaritmo natural.

PRÉ-REQUISITOS
Noções de área entre curvas e denições de funções Logarítmicas.
Funções Logarítmicas:
Uma Abordagem Geométrica

9.1 Introdução

Agora vamos denir um sistema de logaritmo, em qualquer base,


como a área de uma faixa de hipérbole y = k/x para alguma
constante k > 0.

9.2 Funções Logarítmicas: Uma abordagem Geo-


métrica

Seja k uma constante positiva e consideramos o ramo de hipérbole


y = k/x tal que x > 0. Para cada valor de k teremos um novo
sistema de logaritmos, onde o caso k = 1, corresponde aos lo-
garitmos naturais estudados anteriormente. Dados dois pontos de
abcissas x e y , indiquemos com H(k)ba a faixa de hipérbole y = k/x
compreendida entre as retas x = a e x = b. Quando k = 1
continuaremos a indicar com Hab a faixa da hipérbole y = 1/x
situada entre as retas x = a e x = b.

Proprosição 9.17. A área da faixa H(k)ba é igual a k vezes a área


da faixa Hab

Prova: Dado um segmento [c, d] contido em [a, b], um retângulo de


base [c, d], inscrito na hipérbole y = 1/x, tem altura 1/d, equanto
que o retângulo de mesma base, inscrito na hipérbole y = k/x,
tem altura k/d. Logo, a área do segundo retângulo é k vezes a ára
do primeiro. Toda subdivisão do intervalo [a, b], determina dois
polígonos retangulares, um inscrito na faixa Hab e o outro inscrito
na faixa H(k)ba . Segue que a área do segundo é k vezes a área do
primeiro. Concluímos que:

AREA H(k)ba = k · AREA Hab ,

120
Matemática para o Ensino Médio I AULA
pois são dois números reais com as mesmas aproximações inferiores. 9

Figura 9.1: Faixas de hipérboles H(k)ba e Hab .

Fixada a constante k > 0, indroduzimos a função, pondo, por


denição, para cada x > 0,

log(x) = AREA H(k)x1 .

Como acabamos de ver, isto equivale a dizer que

log(x) = AREA H(k)x1 = k · AREA H1x = k · ln(x).

Claramente esta nova função dene um sistema de logaritmos. De


fato, Desde que ln(x) é crescente, segue que log(x) é crescente e
assim ca provado a proprieade A). Por outro lado, temos:

log(xy) = k · ln(xy) = k · (ln(x) + ln(y)) = k · ln(x) + k · ln(y)


= log(x) + log(y)

121
Funções Logarítmicas:
Uma Abordagem Geométrica

e assim ca provado a propriedade B) da denição de um sistema


de logaritmos.
Seja a a base do sistema de logaritmos log , tal que log(a) = 1.
Temos então que log(a) = k · ln(a) = 1. Então vale a seguinte
relação:
1
k=
ln(a)
A notação para o logaritmo de base a de um número x > 0 é

loga (x).

Da maneira que denimos log(x)a é a área da faixa de hipérbole


y = 1/(x · ln(a)) compreendida entre 1 e x. Observamos que com
a determinação de k obtemos a seguinte relação

ln(x)
loga (x) = .
ln(a)

Observação 9.1. Qualquer número real positivo poderia ser es-


colhido como base do sistema de logaritmos. Mas como zemos
no começo a hipótese de que k > 0 (onde k = 1/ ln(a)), estamos
considerando apenas logaritmos onde a base a é maior que 1. A
denição
ln(x)
loga (x) =
ln(a)
poderia também servir de denição de loga (x) mesmo quando 0 <
a < 1, notando-se que, neste caso, como ln(a) < 0, os números
entre 0 e 1 terão logaritmos positivos, enquanto os números maiores
do que 1 terão logaritmos negativos. Quando 0 < a < 1, podemos
por b = 1/a. Então b > 1 e loga (x) = − logb (x). Assim, não
há enorme necessidade de estudar logaritmos com base a < 1. A
rigor, se considerarmos 0 < a < 1 então loga (x) será decrescente.

122
Matemática para o Ensino Médio I AULA
9

Figura 9.2: Gráco da função Logarítmica y = loga (x) com base


a < 1.

Exemplo 9.1. Sejam a e b números reais maiores que um. Para


todo x > 0 tem-se

logb (x) = logb (a) · loga (x).

De fato, Como logb (x) e loga (x) são dois sistemas de logaritmos,
então logb (x) = c loga (x). Fazendo x = a na ultima relação obte-
mos logb (a) = c loga (a), ou seja, c = logb (a).

RESUMO


Função Logarítmica Fixada a constante k > 0, indroduzimos a


função, pondo, por denição, para cada x > 0,

log(x) = AREA H(k)x1 .

Como acabamos de ver, isto equivale a dizer que

log(x) = AREA H(k)x1 = k · AREA H1x = k · ln(x).

123
Funções Logarítmicas:
Uma Abordagem Geométrica

Ainda Valem as seguintes relações importantes:

ln(x)
loga (x) = .
ln(a)

logb (x) = logb (a) · loga (x).

ATIVIDADES


Atividade. 9.1. Sejam 0 < a < b e k > 0. Prove que a faixa da


parábola y = x2 situada sobre o intervalo [ak, bk] tem área igual a
k 3 vezes a faixa situada sobre o intervalo [a, b].

Atividade. 9.2. Dena uma função real de variável real f : R −→


R ponde f (x) = área da faixa da parábola y = x2 situada no
intervalo [0, x], isto é, f (x) =A(x2 )x0 e adote a conveção de que tal
áera é negativa se x < 0. Mostre que f satisfaz

f (kx) = k 3 f (x),

para cada k ∈ R. Seja a = f (1). Conclua que f (x) = ax3 para


todo x ∈ R. A partir do exercício anterior conclua que a = 1/3 e
então f (x) = x3 /3.

Atividade. 9.3. Quanto vale a área da faixa da parábola y = x2


sobre o intervalo [2, 3]? A mesma pergunta para um intervalo
qualquer [a, b].

Atividade. 9.4. Mostre que se a < b então a faixa Hab da hipér-


bole y = 1/x tem área A que satisfaz

1 − a/b < A < b/a − 1

124
Matemática para o Ensino Médio I AULA
Atividade. 9.5. Prove que H160
480 tem área maior que 1 e menor 9
que 1, 2.

REFERÊNCIAS BIBLIOGRÁFICAS


Lima, E.L., A Matemática no Ensino Médio I, 5.ed., SMB, 2006.


Lima, E.L., Logaritmos, 2.ed., SMB, 1996.

125
AULA

10
Funções Exponenciais

META:
Denir e obter as propriedades da função exponencial.

OBJETIVOS:
Ao m da aula os alunos deverão ser capazes de:
Denir potencias de números reais com expoente, natural, inteiro
e racional e depois extender o conceito para expoentes irracionais.
Obter a denição da função exponencial como inversa da função
logarítmica.
Esboçar o gráco de uma função exponencial.

PRÉ-REQUISITOS
Propriedades básicas de números reais e funções logarítmicas.
Funções Exponenciais

10.1 Introdução

As funções exponenciais, são uma das classes de funções mais


importantes da matemática, pois estas são usadas como modelo
matemático de muitos problemas da matemática de das ciências
exatas. Um dos exemplos mais clássicos é no modelo matemática
que representa o número de indivíduos de uma determinada popu-
lação no instante t. Esta função tem a propriedade que a taxa de
crescimento relativo [f (t + h) − f (t)]/f (t), só depende de h e não
de x.

10.2 Potências de Expoente Racional

Seja a um número real positivo. Para todo n natural, a potência


an de base a e expoente n é denida como o produto de n fatores
iguais a a. Ou seja,
n vezes
an = a · a · a · . . . · a .
z }| {

Observamos que se n = 1 não temos produto de fatores e então


pomos por denição a1 = a. Assim a denição indutiva de an é:
a1 = a e an+1 = a · an . Temos ainda que, para quaisquer m, n ∈ N
vale a seguinte relação:

am+n = an · am . (10.28)

De fato, em ambos os membros da igualdade temos o produto de


m + n fatores iguais a a. Aplicando sucessivamente a fórmula
(10.28) obtemos a relação

am1 +m2 +...+mk = am m m


1 · a2 · . . . · ak , (10.29)

128
Matemática para o Ensino Médio I AULA
para qualquer m1 , m2 , . . . , mk ∈ N. Em particular obtemos que 10
(am )k = amk .
Seja a > 1, então multiplicando esta desigualdade por an , obtemos
que an+1 > an . Segue então que

a>1 =⇒ 1 < a < a2 < . . . < an < an+1 < . . . . (10.30)

De maneira análoga, concluímos que Desigualdade de


Bernoulli: Sejam
x > −1 e n inteiro não
0<a<1 =⇒ 1 > a > a2 > . . . > an > an+1 > . . . . (10.31) nulo. Então
(1 + x)n ≥ 1 + nx.
Seguem das equações (10.30) e (10.31), que a sequência cujo n−ésimo
De fato, procedendo por
termo é an é crescente quando a > 1 e decrescente quando 0 < a < indução sobre n, obtemos
(1 + x)0 = 1 e então
1. vale a igualdade. Supo-
nhamos, por hipótese de
Para a > 1 a sequência formada pelas potências an , com n ∈ N, é indução que (1+x)n ≥ 1+
nx. Multiplicamos ambos
ilimitada superiormente, ou seja, nenhum número real c, pode ser os lados da desigualdade
por 1+x, o qual é positivo,
superior a todas as potências an , mais especicamente, dado arbi- desde que x > −1. Assim
segue que:
trariamente c ∈ R, pode-se encontrar sempre n ∈ N tal que an > c.
Para vericar isto, basta escrever a = 1 + d. Pela Desigualdade de (1+x)n+1 ≥ (1+nx)(1+x)
Bernoulli, temos que an > 1+nd. Logo, se tomarmos n > (c−1)/d = 1 + (n + 1)x + nx2

teremos 1 + nd > c e assim an > c. ≥ 1 + (n + 1)x.


donde a ultima desigual-
De modo análogo, se 0 < a < 1, se escrevemos b = 1/a teremos dade segue do fato que
nx2 ≥ 0.
b > 1. Logo, pelo que vimos acima, considerando agora a sequência
bn , dado c ∈ R podemos encontrar n ∈ N tal que bn > 1/c, ou seja,
1
an > 1
c e assim an < c. Concluímos que se 0 < a < 1, a sequência
an decresce abaixo de qualquer cota positiva, ou seja, a sequência
decrescente é ilimitada inferiormente.
No que segue, vamos dar um signicado à potencia an , quando n ∈
Z é um número inteiro e de maneira que que seja mantida válida

129
Funções Exponenciais

a propriedade dada pela equação (10.28), ou seja am+n = am · an .


Primeiramente de

a1 = a0+1 = a0 · a1 = a0 · a

donde resulta que a0 = 1. Por outro lado

1
a−n · an = a−n+n = a0 = 1 logo a−n = . (10.32)
an

Assim, a denição de an (a > 0) quando n é um inteiro qualquer


de tal forma que vale a igualdade am+n = am · an , é dada por:
a0 = 1 e a−n = an , para todo n ∈ N.
A função f : Z −→ Z, dada por f (n) = an , além de cumprir a
igualdade fundamental

f (m + n) = f (m) · f (n),

e ainda é crescente quando a > 1 e descrescente quando 0 < a < 1.


Segue em particular que, para a > 1 e n ∈ N, tem-se a−n < 1 < an
e a0 = 1.
Prosseguindo, vamos dar um sentido à potência ar quando r =
m/n é um número racional ( onde m ∈ Z e n ∈ N), de modo que
continue válida a regra ar · as = ar+s . Desta igualdade resulta que:

(ar )n = ar · ar · . . . · ar = ar+r+...+r = arn = am .

Portanto ar é o número real positivo cuja n−ésima potência é igual



a am . Por denição de raiz, este número é n am , a raiz n−ésima de
am . Assim, a única maneira de denir a potência ar , com r = m/n,
m ∈ Z, n ∈ N, consiste em pôr:


(ar ) = am/n = n
am .

130
Matemática para o Ensino Médio I AULA
Sejam r = m1 /n1 e s = m2 /n2 , m1 , m2 ∈ Z, n1 , n2 ∈ N. Temos 10
que:
ar+s = ar · as .

De fato, sabemos que:

(ar1 )n1 = am1 (ar2 )n2 = am2 .

Logo:

(ar1 · ar2 )n1 n2 = (ar1 )n1 n2 · (ar2 )n1 n2 = ar1 n1 n2 · ar2 n1 n2


= am1 n2 · am2 n1 = am1 n2 +m2 n1 .

Vemos que ar1 · ar2 é o número cuja n1 n2 -ésima potência vale


am1 n2 +m2 n1 , ou seja,

ar1 · ar2 = a(m1 n2 +m2 n1 )/n1 n2 .

e observando que

m1 n2 + m2 n1 m1 m2
= = r1 + r2 ,
n1 n2 n1 n2

obtemos ar1 · ar2 = ar1 +r2 .


Consideramos a sequência, cujo r−ésimo termo é ar , r ∈ Q. Então
se a > 1 temos que a sequência é crescente. De fato, se r = m/n;
m ∈ Z, n ∈ N e s = p/q ; p ∈ Z, q ∈ N e r > s temos que mq > np (
com mq, np ∈ Z). Segue, como vimos anteriormente que se a > 1
então
amq > anp =⇒ (am )q > (ap )n , (10.33)

Lembrando que segue da denição que (ar )n = am e (as )q = ap .


Logo, da equação (10.33) segue que:

(am )q > (ap )n , =⇒ (ar )nq > (as )nq , (10.34)

131
Funções Exponenciais

o que nos dá ar > as e portanto se a > 1 a sequência ar é crescente.


De maneira análoga mostra-se que se 0 < a < 1 a sequência ar ,
r ∈ Q é decrescente.
Consideremos a função f : Q −→ R+ , denida por f (r) = ar .
Temos que f não é sobrejetiva, ou seja, xado um número a > 0
nem todo número real positivo y é da forma ar com r racional. Isto
pode ser observado do fato que Q é um conjunto enumerável, e o
mesmo ocorre com sua imagem f (Q), mas R+ não é enumerável.

Exemplo 10.1. Seja a = 10 e indagamos se existe algum número


racional r = m/n tal que 10m/n = 11, ou seja, tal que 10m = 11n ,
onde m, n ∈ N. É claro que 10m se escreve como 1 seguindo de m
zeros enquanto 11n não pode ter esta forma. Logo o número 11
não pertence a imagem da função r 7−→ 10r , r ∈ Q.

Lema 10.2. Fixado o número real a 6= 1, em todo intervalo de R+


existe alguma potência ar , com r ∈ Q.

Antes de demonstrar este lema, enfatizamos seu signicado. As


potências ar embora não contenham todos os números reais posi-
tivos, estão espalhadas por toda parte de R+ .
Prova: Sejam 0 < α < β , vamos mostrar que existe r ∈ Q tal que
a potência ar pertença ao intervalo [α, β]. Por simplicidade vamos
supor que a e α sejam maiores que um. Os demais casos podem
ser tratados de modo análogo. Desde que as potências de expoente
natural de números maiores que 1 crescem acima de qualquer cota
prexada, podemos obter números naturais M e n tais que:
 β − α n
α < β < aM , e 1 < a < 1 + M .
a
Da última relação acima decorrem sucessivamente
β−α
1 < a1/n < 1 + , e 0 < aM (a1/n − 1) < β − α.
aM

132
Matemática para o Ensino Médio I AULA
Logo
m m 1
10
n ≥M =⇒ 0 < a n (a n − 1) < β − α
m+1 m
⇐⇒ 0<a n − a n < β − α.
Assim as potências

a0 = 1, a1/n , a2/n , ..., aM

são extremos de intervalos consectivos, todos de comprimento me-


nor do que o comprimento β − α do intervalo [α, β]. Como [α, β] ⊂
[1, aM ], pelo menos um desses extremos, digamos am/n está contido
em [α, β].

Denição 10.1. Seja a um número real positivo, a 6= 1. A função


exponencial de base a, f : R −→ R+ , denotada por f (x) = ax é
denida de modo que tenha as seguintes propriedades para quais-
quer x, y ∈ R.
(i) ax+y = ax · ay ;
(ii) a1 = a;
(iii) x < y =⇒ ax < ay quando a > 1 e x < y =⇒ ax > ay quando
0 < a, 1

Se uma função f : R −→ R tem a propriedade f (x + y) = f (x) ·


f (y), então se f assume o valor 0 então f é identicamente nula e
se não for identicamente nula, então f (x) > 0 para todo x > 0.
Deixaremos como atividade a demonstração destes fatos, a qual
segue imediatamente da propriedade (i).
Se a função f : R −→ R tem as propriedades (i) e (ii), então para
todo n ∈ N temos que

f (n) = f (1 + 1 + . . . + 1) = f (1) · f (1) · . . . · f (1) = a · a · . . . · a = an .

133
Funções Exponenciais

Usando ainda a propriedade (i), resulta ainda como vimos antes,


que para todo número racional r = m/n, n ∈ N, deve-se ter f (r) =

ar = n am . Portanto f (r) = ar é a única função f : Q −→ R+ ,
tal que f (r + s) = f (r) · f (s) para quaisquer r, s ∈ Q e f (1) = a.
A propriedade (iii) nos informa que a funçào exponencial deve ser
crescente quando a > 1 e decrescente quando 0 < a < 1.
A partir de tudo isso que acabamos de ver, concluiremos que existe
uma única maneira de denir o valor de f (x) = ax quando x é
irracional. Para xar idéias, vamos supor que a > 1. Então ax
tem a seguinte propriedade:

r < x < s, com r, s ∈ Q =⇒ ar < ax < as .

Ou seja, ax é o número real cujas aproximações por falta são ar ,


com r < x, r ∈ Q, e cujas aproximações por excesso são as , com
x < s, s ∈ Q. Não podem existir dois números reais diferentes,
digamos A < B , com a propriedade acima. Se existissem tais A e
B teríamos

r < x < s, com r, s ∈ Q =⇒ ar < A < B < as ,

e então o intervalo [A, B] não conteria nenhuma potência de a com


expoente racional, contrariando o Lema 10.2.
Portanto, quando x é irracional, a é o único número real cujas
aproximações por falta são as potências ar , com r racional menor
que x e cujas aproximações por excesso são as potências as , com s
racional maior que x.
Temos ainda, que valem as seguintes propriedades:

Propriedade 10.8. A função f : R −→ R+ , denida por f (x) =


ax , é ilimitada superiormente.

134
Matemática para o Ensino Médio I AULA
De fato, todo intervalo em R+ contém valores f (r) = ar , segundo 10
o Lema 10.2. Mais precisamente, se a > 1 então ax cresce sem
limites quando x > 0 é muito grande. E se 0 < a < 1 então ax
torna-se arbritariamente grande quando x < 0 tem valor absoluto
grande.

Propriedade 10.9. A função exponencial é contínua.

Isto signica que dado x0 ∈ R, é possível tornar a diferença |ax −


ax0 | tão pequena quanto se deseje, desde que x seja tomado su-
cientemente próximo de x0 . Dito de outro modo, o limite de ax
quando x tende a x0 é igual a ax0 . Para provar este fato, escreve-
mos x = x0 + h, logo x − x0 = h e entãp |ax − ax0 | = ax0 |ah − 1|.
Ora, sabemos que ah pode ser tomado tão próximo de 1 quanto
se deseje, desde que h seja sucientemente pequeno. Como ax0 é
constante, podemos fazer o produto ax0 |ah −1| tão pequeno quanto
queriamos, logo teremos que ax tende a ax0 quando x tende a x0 .

Propriedade 10.10. A função exponencial f : R −→ R+ , f (x) =


ax , a 6= 1 é sobrejetiva.

Usando o Lema 10.2 e escolhemos para cada n ∈ N, uma potência


arn , com rn ∈ Q, no intervalo (b − 1/n, b + 1/n), de modo que
|b − arn | < 1/n e assim lim arn = b. Para xar idéias, supomos
x−→x0
a > 1. Escolhemos as potências arn , de tal maneira que:

ar1 < ar2 < . . . < arn < . . . < b.

Certamente, podemos xar s ∈ Q tal que b < as . Então a mono-


tonicidade da função ax nos assegura que r1 < r2 < . . . < rn <
. . . < s. Assim (rn ) é uma sequência monótona, limitada superior-
mente por s. A completeza dos números reais garante então que os

135
Funções Exponenciais

rn são valores aproximados por falta de um número real x, ou seja,


ax = lim rn = x. A função exponencial sendo contínua, implica
x−→x0
que ax = lim arn = b.
x−→x0

10.2.1 Gráfico da Função exponencial

Vemos que se a > 1 a função exponencial ax é crescente, contínua,


ilimitada superiormente e tende a zero quando x se torna muito
negativo. Quando 0 < a < 1 a função ax é descrescente, contínua,
ilimitada superiormente e tende a zero quando x se torna muito
grande. Veja o gráco na Figura 10.2.1 nos dois casos, num mesmo
sistema de coordenadas.

Figura 10.1: Gráco da função exponencial, num mesmo sistema


de eixos, para os casos a > 1 e 0 < a < 1.

136
Matemática para o Ensino Médio I AULA
10.3 Funções Exponenciais × Funções Loga- 10
rítmicas

Nesta aula denimos a função exponencial f (x) = ax , para todo


x ∈ R. Porém o zemos, indenpendente do conceito de função
logarítmica. As duas funções estão intimamente ligadas já que
uma é a inversa da outra. Vejamos, assim uma forma de denir a
função exponencial a partir da função logarítmica.

Denição 10.2. Dado a > 0 e x ∈ R, a potência ax é o único


número real positivo cujo logaritmo natural é igual a x · ln(a).

Seja b > 0. Então:

ln(ax ) ln(a)
logb (ax ) = =x = x · logb (a)
ln(b) ln(b)

Logo, a fórmula que serviu para a denição de ax , dada em ter-


mos de logaritmos naturais, é portanto válida para logaritmos em
qualquer base. Portanto, em particular fazendo b = a, temos que:

loga (ax ) = x.

Recaímos assim na denição tradicional de logarítmo, ou seja,


loga (x) = y ⇐⇒ ay = x.
Precisamos mostrar a equivalência entre as duas denições para
função exponencial. Então precisamos vericar que são válidas as
propriedades (i), (ii) e (iii) da Denição 10.2. Temos que:

loga (ax · ay ) = loga (ax ) + log( ay )

e
loga (ax+y ) = x + y.

137
Funções Exponenciais

Portanto os números ax · ay e ax+y têm o mesmo logaritmo na base


a. Concluímos que ax · ay = ax+y . Também loga (a0 ) = 0 e como
loga (1) = 0 segue que a0 = 1.
Ainda é fácil vericar se a > 1, a função x 7−→ ax é crescente e
quando 0 < a < 1 é decrescente.

10.4 Função Exponencial na base e

Já vimos que o número e é tal que a área da faixa de hipérbole


y = 1/x, de 1 até x vale e, ou seja AREAH1e = 1. O número
e é chamado a base dos lugaritmos naturais e ln(e) = 1. Vamos
mostrar agora que o número e também é o limite de (1 + 1/n)n .

Proprosição 10.18.
 1 n
lim 1+ =e
n−→∞ n

Prova: Considere a Figura 10.2 abaixo:


Nela temos um retângulo menor, cuja base mede x e cuja altura
mede 1/(1 + x), contido na faixa H11+x e esta faixa, por sua vez,
contida no retângulo maior, com a mesma base medida x e al-
tura igual a 1. Comparando as áreas dessas três guras, podemos
escrever, para todo x > 0:

x
< ln(1 + x) < x.
1+x
Dividindo por x:
1 ln(1 + x)
< < 1.
1+x x
Tomando x = 1/n:

n  1 n
< ln 1 + < 1,
n+1 n

138
Matemática para o Ensino Médio I AULA
10

Figura 10.2: Construção geométrica que mostra a desigualdade


x/(1 + x) < ln(1 + x) < x.

e portanto
n
 1 n
e n+1 < 1 + < e,
n

para todo n ∈ N. Quando n cresce indenidamente, n/(n + 1)


se aproxima de 1, logo en/(n+1) tende a e. Segue desta últimas
desigualdades que
 1 n
lim 1+ =e
n−→∞ n
 1 n
Observamos que o resultado lim 1 + = e foi obtido para
n−→∞ n
x > 0. Mas seja x > 0 ou −1 < x < 0 temos que vale o resultado

 1
x
lim 1+x =e (10.35)
x−→0

onde zemos x = 1/n. Agora Tomando x = α/n, vemos que


1/x = n/α e que x −→ 0 se , e somente se, n −→ ∞. Logo, pela

139
Funções Exponenciais

equação (10.35) segue que:


 α n h α  αn iα h  1 iα
x
lim 1+ = lim 1+ = lim 1+x = eα
n−→∞ n n−→∞ n x−→0

Denição 10.3. A função f : R −→ R+ tal que f (x) = ex pode


 x n
ser denida por meio do limite lim 1 + = ex ou então
n−→∞ n
geometricamente pelo fato de que y = ex é o único número real
positivo tal que a área da faixa de hipérbole H1x é igual a x.

A taxa de crescimento de uma função f no intervalo de extremi-


dades x, x + h é por denição , o quociente

f (x + h) − f (x)
,
h

e pode ser interpretado como a inclinação da reta secante que une


os pontos (x, f (x)) e (x + h, f (x + h)) do gráco de f .

Figura 10.3: Interpretação geométrica da taxa de variação


f (x+h)−f (x)
h .

No caso particular da função f (x) = b · eαx , temos

f (x + h) − f (x) eαh − 1
= beαx .
h h

140
Matemática para o Ensino Médio I AULA
A derivada da função f no ponto x ao limite da taxa f (x+h)−f (x)
h
10
quando h tende a zero. Este número, cujo signicado é o de taxa
instantânea de crescimento de f no ponto x, é representado por
f 0 (x). Ele é o número real cujos valores aproximados são os quo-
f (x+h)−f (x)
cientes h para valores muito pequenos de h. Geometri-
camente f 0 (x) é a inclinação da reta tangente ao gráco de f no
ponto x.
O sinal de f 0 (x) indica a tendência da variação de f a partir de
x. Se f 0 (x) > 0 então f (x + h) > f (x), para pequenos valores
de h positivo. Se f 0 (x) < 0 tem-se ao contrário, f (x + h) < f (x)
para h pequeno e positivo. Mostraremos a seguir que a derivada
da função f (x) = beαx é igual a α · f (x). Em outras palavras, a
taxa instantânea de variação de uma função de tipo exponencial é,
em cada ponto x, proporcional ao valor da função naquele ponto.
E a constante α é justamente o fator de proporcionalidade.
Usando a interpretação geométrica de logaritmo natural, é fácil
calcular a derivada da função f (x) = b · eαx . O ponto de partida
eh − 1
consiste em mostrar que lim = 1. A demostração deste
h−→0 h
fato, segue de um argumento geométrico, usando área de retângu-
los e faixa de hipérboles.
Assim, é imediato ver que:
ex+h − ex eh − 1
lim = ex · lim = ex
h−→0 h h−→0 h
e mais geralmente,
eα(x+h) − ex eαh − 1 eαh − 1
lim = eαx · lim = α · eαx · lim .
h−→0 h h−→0 h h−→0 αh
Escrevendo k = αh, vemos que h −→ 0 ⇐⇒ k −→ 0. Portanto

eα(x+h) − eαx ek − 1
lim = α · eαx · lim = α · eαx .
h−→0 h k−→0 αk

141
Funções Exponenciais

Isto conclui a demosntração de que que a derivada da função


f (x) = eαx é f 0 (x) = α · f (x), logo é proporcional ao valor de
f (x) da função f , sendo α o fator de proporcionalidade.

RESUMO


Funções Exponenciais
Denição: Seja a um número real positivo, a 6= 1. A função
exponencial de base a, f : R −→ R+ , denotada por f (x) = ax é
denida de modo que tenha as seguintes propriedades para quais-
quer x, y ∈ R.
(i) ax+y = ax · ay ;
(ii) a1 = a;
(iii) x < y =⇒ ax < ay quando a > 1 e x < y =⇒ ax > ay quando
0 < a, 1
A função exponencial, goza das seguintes propriedades:

142
Matemática para o Ensino Médio I AULA
• A função f : R −→ R+ , denida por f (x) = ax , é ilimitada 10
superiormente.

• A função exponencial é contínua.

• A função exponencial f : R −→ R+ , f (x) = ax , a 6= 1 é


sobrejetiva.

Funções Exponenciais × Funções Logarítmicas


Denição: Dado a > 0 e x ∈ R, a potência ax é o único número
real positivo cujo logaritmo natural é igual a x · ln(a).

ATIVIDADES


Atividade. 10.1. Por que se pode assegurar que existe um número


y > 0 tal que a faixa de hipérbole H1y tem área igual a π ? Quantos
números y existem com esta propriedade? Qual a relação entre y
e o número e?

Atividade. 10.2. Dados a > 0 e b > 0, qual a propriedade da


funçào exponencial que assegura a existência de h 6= 0 tal que
bx = ax/h para todo x ∈ R? Mostre como obter o gráco de y = bx
a partir do gráco de y = ax . Use sua conclusão para traçar o

gráco de y = (1/ 3 4)x a partir do gráco de y = 2x .

Atividade. 10.3. Para todo x > 0, mostre que se tem

x2
ex > 1 + x + .
4

Atividade. 10.4. Dado a > 0, determinar x tal que a faixa de


hipérbole Hax tenha área igual a um número b dado.

143
Funções Exponenciais
√ √

Atividade. 10.5. Use a igualdade 2
2 2
= 2 para concluir
que existem números irracionais a e b tais que ab é racional.

REFERÊNCIAS BIBLIOGRÁFICAS


Lima, E.L., A Matemática no Ensino Médio I, 5.ed., SMB, 2006.


Lima, E.L., Logaritmos, 2.ed., SMB, 1996.

144
AULA

Caracterização das funções


11
Exponenciais e Logarítmicas

META:
Caracterizar as funções exponenciais e Logarítmicas.

OBJETIVOS:
Ao m da aula os alunos deverão ser capazes de:
Identicar uma função exponencial ou do tipo exponêncial, a par-
tir de seus teoremas de caracterização.
Identicar uma função logarítmica a partir de seu teorema de car-
acterização.
Aplicar os teoremas de caracterização, para modelar problemas el-
ementares.

PRÉ-REQUISITOS
Denições elementares e propriedades das funções logarítmica e
exponencial.
Caracterização das funções
Exponenciais e Logarítmicas

11.1 Introdução

As funções exponenciais são juntamente com as funções ans e as


quadráticas, os modelos matemáticos mais utilizados para resolver
problemas elementares. Uma vez decidido o modelo adequado para
um determinado problema a tratamento matemático não apresen-
tará maiores problemas. As dúvidas podem surgir na escolha do
instrumento matemático apropriado para o problema em estudo.
Para que a escolha possa ser feita corretamente é preciso conhecer
as propriedades características de cada tipo de funções. Vejamos
os teoremas de caracaterização das funções esponenciais e logarít-
micas.

11.2 Caracterização das Funções Exponênci-


ais

Teorema 11.1. Seja f : R −→ R+ uma função monótona injetiva


(isto é, crescente ou descrescente). As seguintes armações são
equivalentes:
(1) f (nx) = f (x)n para todo n ∈ Z e todo x ∈ R;
(2) f (x) = ax para todo x ∈ R, onde a = f (1);
(3) f (x + y) = f (x) · f (y) para quaisquer x, y ∈ R.

Prova: Provaremos as implicações (1) =⇒ (2) =⇒ (3) =⇒ (1).


Para mostrar (1) =⇒ (2) observamos inicialmente que a hipótese
(1) acarreta que, para todo número racional r = m/n (com m ∈
Z e n ∈ N) tem-se f (rx) = f (x)r . Com efeito, com nr = m,
podemos escrever f (rx)n = f (nrx) = f (mx) = f (x)m , e logo
f (rx) = f (x)m/n = f (x)r .

146
Matemática para o Ensino Médio I AULA
Assim, se pusermos f (1) = a, teremos f (r) = f (r · 1) = f (1)r = ar 11
para todo r ∈ Q. Para completar a demonstração de que (1) =⇒
(2) suponhamos, a m de xar idéias que f seja crescente, logo
1 = f (0) < f (1) = a. Admitamos, por absurdo, que exista um
x ∈ R tal que f (x) 6= ax . Digamos, por exemplo, que seja f (x) <
ax ( o caso em que f (x) > ax é tratado analogamente). Então
pelo Lema da aula anterior anterior, existe um número racional
r tal que f (x) < ar < ax , ou seja, f (x) < f (r) < ax . Como
f é crescente, tendo f (x) < f (r), concluímos que x < r. Esta
contradição termina (1) =⇒ (2). As demais implicações seguem
naturalmente da denição.

Observação 11.1. No enunciado, poderíamos substituir a hipótese


de monotonicidade pela suposição e que f seja contínua. A de-
monstração de (1) =⇒ (2) muda apenas no caso de que x é ir-
racional. Então tem-se x = lim f (rn ) = rn , rn ∈ Q, logo, pela
n−→∞
continuidade de f , deve ser

f (x) = lim f (rn ) = lim arn = ax .


n−→∞ n−→∞

Denição 11.1. Uma função g : R −→ R é do tipo exponencial


quando se tem g(x) = bax para todo x ∈ R, onde a e b são cons-
tantes positivas

Se a > 1, a função g é crescente e se 0 < a < 1, g é decrescente. Se


a função g : R −→ R é do tipo exponencial então, para quaisquer
x, h ∈ R, os quocientes
g(x + h) − g(x) g(x + h)
= ah − 1 = ah
g(x) g(x)
dependem apenas de h, mas não de x. Mostraremos agora que vale
a recíproca deste resultado.

147
Caracterização das funções
Exponenciais e Logarítmicas

Teorema 11.2. Seja g : R −→ R uma função monótona injetiva


(isto é crescente ou decrescente) tal que para todo x, h ∈ R quais-
quer, o acréscimo relativo [g(x + h) − g(x)]/g(x) dependa apenas
de h, mas não de x. Então se b = g(0) e a = g(1)/g(0), tem-se
g(x) = bax para todo x ∈ R.

Prova: A hipótese feita equivale a supor que ψ(h) = g(x+h)/g(x)


independe de x. Substituindo, se necessário, g(x) por f (x) =
g(x)/b, onde b = g(0), f continua monótona injetiva, com f (x +
h)/f (x) independente de x e, agora, com f (0) = 1. Então, pondo
x = 0 na relação ψ(h) = f (x + h)/f (x), obtemos ψ(h) = f (h),
para todo h ∈ R. Vemos assim que a função monótona injetiva
f cumpre f (x + h) = f (x) · f (h), ou seja f (x + y) = f (x) · f (y)
para quaisquer x, y ∈ R. Segue-se então do teorema anterior que
f (x) = ax , logo g(x) = bf (x) = bax .

Seja f : R −→ R, f (x) = bax , uma função do tipo exponencial e


x1 , x2 , . . . , xn , . . . é uma progressão aritmética de razão h, isto é,
xn+1 = xn + h, então os valores

f (x1 ) = bax1 , f (x2 ) = bax2 , . . . , f (xn ) = baxn , . . . ,

formam uma progressão aritmética de razão ah pois

f (xn+1 ) = baxn+1 = baxn+1 = baxn +h = (baxn ) · ah .

Como o (n + 1)-ésimo termo da progressão aritmética dada é


xn+1 = x1 +nh, segue-se que f (xn+1 ) = f (x1 )An onde A = ah . Em
particular, se x1 = 0 então f (x1 ) = b e portanto f (xn=1 ) = bAn .
Vejamos o seguinte teorema:

Teorema 11.3. Seja f : R −→ R+ uma função monótona in-


jetiva que transforma toda progressão aritmética x1 , x2 , . . . , xn , . . .

148
Matemática para o Ensino Médio I AULA
numa progressão geométrica y1 , y2 , . . . , yn , . . ., onde yn = f (xn ). 11
Se pusermos b = f (0) e a = f (1)/f (0) teremos f (x) = bax para
todo x ∈ R.

Prova: Seja b = f (0). A função g : R −→ R+ , denida por


g(x) = f (x)/b, é monótona injetiva, contínua transformando pro-
gressões aritméticas em progressões geométricas e agora tem-se
g(0) = 1. Dado x ∈ R qualquer, a sequência x, 0, −x é uma
progressão aritmética, logo g(x), 1, g(−x) é uma progressão geo-
métrica de razão g(−x). Segue-se g(−x) = 1/g(x). Sejam agora
n ∈ N e x ∈ R. A sequência 0, x, 2x, . . . , nx é uma progressão arit-
mética, logo 1, g(x), g(2x), . . . , g(nx) é uma progresão geométrica,
cuja razão evidentemente é g(x). Então seu (n + 1)-ésimo termo
é g(nx) = g(x)n . Se −n é um inteiro negativo então g(−nx) =
1/g(nx) = 1/g(x)n = g(x)−n . Portanto, vale g(nx) = g(x)n para
quaisquer n ∈ Z e x ∈ R. Segue-se do Teorema de Caracterização
acima que pondo a = g(1) = f (1)/f (0), tem-se g(x) = ax , ou seja,
f (x) = bax , para todo x ∈ R.

Exemplo 11.1. Se um capital c0 é aplicado a juros xos en-


tão, depois de decorrido um tempo t, o capital existente é dado
por c(t) = c0 · at . Se tirarmos extratos da conta nos tempos
0, h, 2h, 3h, . . . teremos c(0) = c0 , c(h) = c0 · A, c(2h) = c0 · A2 ,
. . . onde A = ah . Portanto a evolução do saldo, quando calcu-
lado em intervalos de h unidades de tempo, é dada pela progressão
geométrica c0 , c0 · A, c0 · A2 , . . ..

149
Caracterização das funções
Exponenciais e Logarítmicas

11.2.1 Aplicações

No teorema de caracterização das funções do tipo exponencial,


vimos um critério elegante e matematicamente simples para deter-
minar quando uma bijeção crescente ou decrescente f : R −→ R+
Dada a função exponencial é da forma f (x) = b · ax , ou seja, f (x) = b · eαx
f (x) = ax pode ser escrita,
por conviniencia, na base Para aplicar esse critério em situações concretas é indispensável
e, na forma f (x) = eαx .
Para isso basta denir a = saber decidir, em cada caso especíco, se f (x + h)/f (x) independe
eα , ou seja ln(a) = α.
de x ou não. Fixando h como constante, isto equivale a indagar se
f (x + h)/f (x) é constante, isto é, se f (x + h) = c · f (x) para todo
x ∈ R, ou ainda, se f (x + h) é uma função linear de f (x).
Escrevendo y = f (x) , y 0 = f (x0 ) e pondo f (x + h) = η(y), f (x0 +
h) = η(y 0 ), o Teorema Fundamental da Proporcionalidade diz que
η é uma função linear de y se, e somente se, para quaisquer y ∈ R+
e n ∈ N tem-se a implicação

y0 = n · y =⇒ η(y 0 ) = n · η(y).

Em termos da função original f isto signica:

F (x0 ) = n · f (x) =⇒ f (x0 + h) = n · f (x + h).

A implicação acima é, portanto, o critério que nos permitirá decidir


se a função f é ou não do tipo exponencial.

Exemplo 11.2. Capital a juros xos. Seja c(t) o capital no


instante t, resultante da aplicação, a juros xos acumulados con-
tinuamente, de um capital inicial c0 = c(0). Então c(t + h) pode
ser considerado como o capital resultante da aplicação da quantia
inicial c(t) durante o período de tempo h. Logo, aplicando o valor
n · c(t) = c(t0 ) obtém-se, após o período h, n · c(t + h). Portanto,
c(t0 ) = n · c(t) =⇒ c(t0 + h) = nc(t + h). Segue-se que c(t) = c0 · at ,

150
Matemática para o Ensino Médio I AULA
onde a = c(1)/c0 , ou c(t) = co · eαt , onde α = ln(a). Como c(t) é 11
uma função crescente de t, tem-se a > 1, ou seja, α > 0.

Exemplo 11.3. Desintegração Radiotiva. Seja m(t) é a massa,


no instante t, de uma substância radioativa que no início da con-
tagem do tempo era m0 = m(0). Assim, m(t + h) é o que resta da
massa m(t) da substância radioativa depois de decorrido o inter-
valo de tempo h a partir do instante t. É claro que se observarmos
a massa m(t0 ) = n · m(t) , n ∈ N, após o mesmo tempo h, veremos
que restou m(t0 + h) = n · m(t + h). Portanto podemos assegu-
rar que m(t) = m0 eαt para todo t ∈ R. Neste caso, como m(t) é
função decrescente de t, temos α < 0.

Exemplo 11.4. Concentração de uma solução. Este é o pro-


tótipo de uma situação que ocorre em diversas circunstâncias, in-
clusive a eliminação de substâncias na corrente sanguínea humana.
Vamos considerar o caso de um tanque de volume V , no qual se
encontra uma salmoura (solução de água e sal), que mantém ho-
mogêna mediante a ação permanente de um misturador. O tanque
recebe um uxo constante de água enquanto uma torneira escoa
a salmoura em quantidade igual, a cada instante, ao volume de
água que entrou no tanque. Procura-se determinar a fórmula que
exprime o volume f (t) do sal existente no tanque no momento
t, portanto a taxa f (t)/V que mede a concentração da solução
naquele instante. Evidentemente, f (t) é uma função decrescente
de t.
Armamos que f é uma função de tipo exponencial. De fato, sejam
t, t0 ∈ R e n ∈ N tais que f (t0 ) = n · f (t). Fixemos arbitrariamente
um intervalo de tempo h. Devemos mostrar que f (t0 + h) = n ·
f (t + h). Para isto, imaginemos o tanque subdividido em n partes

151
Caracterização das funções
Exponenciais e Logarítmicas

de igual volume V /n. Como a mistura é homogênea, em cada


uma destas partes a quantidade de sal nela contida no instante
t0 é igual a f (t0 )/n, ou seja, igual a f (t). O que ocorre em cada
uma das subdivisões é, em escala 1/n, o mesmo que ocorre no
tanque inteiro. Logo, no instante t0 + h, cada subdivisão vai conter
o volume f (t + h) de sal. No todo, vemos que o volume do sal
contido no tanque inteiro no instante t0 +h é f (t0 +h) = n·f (t+h).
Portanto, se b é o volume do sal contido no tanque no instante
t = 0, a fórmula que exprime a quantidade de sal existente no
tanque no tempo t é f (t)b · at , onde a = f (1)/f (0) < 1, ou seja,
f (t) = b · eαt , com α = ln(a) < 0. Sabemos, do capítulo anterior
que α = f 0 (0)/b, onde f 0 (0) = lim [f (t) − b]/t. Seja v o volume
t−→0
de água que entra no tanque na unidade de tempo. Num tempo t,
a água que entra é vt e , se t for muito pequeno, o sal que sai na
salmoura é aproximadamente (b/v)vt. logo f (t) ≈ b − (b/V )vt e

f 0 (0) = lim [f (t) − b]/t = −bv/V .


t−→0

Assim α = −v/V e, em qualquer instante t, a quantidade de sal


no tanque é f (t) = b · e−(v/V )t .

11.3 Caracterização das funções Logarítmicas

Provaremos a seguir que, entre as funções monótonas injetivas f :


R+ −→ R, somente as funções logarítimicas têm a propriedade de
transformar produto em somas.

Teorema 11.4. Seja f : R+ −→ R uma função monótona injetiva


tal que f (xy) = f (x)+f (y) para quaisquer x, y ∈ R+ . Então existe
a > 0 tal que f (x) = loga (x) para todo x ∈ R+ .

152
Matemática para o Ensino Médio I AULA
Prova: Vamos supor f crescente ( o caso de f decrescente segue 11
analogamente). Temos que f (1) = f (1.1) = f (1) + f (1), logo
f (1) = 0. Provaremos o teorema inicialmente supondo que exista
a tal que f (a) = 1. Depois mostraremos que isso sempre acontece.
Como f é crescente e f (a) = 1 > 0 = f (1), tem-se a > 1. Para
todo m ∈ N vale:

f (am ) = f (a · a · a · . . . · a) = f (a) + f (a) + f (a) + . . . + f (a)


= 1 + 1 + 1 + . . . + 1 = m.

Então temos que

0 = f (1) = f (am · a−m ) = f (am ) + f (a−m ) = m + f (a−m ),

logo f (a−m ) = −m. Se r = m/n com m ∈ Z e n ∈ N, então


rn = m, portanto

m = f (am ) = f (arn ) = f ((ar )n ) = nf (ar ),

e daí f (ar ) = m/n = r. Se x é irracional então, para r, s racionais


tem-se

r < x < s =⇒ ar < ax < as =⇒ f (ar ) < f (ax ) < f (as )


=⇒ r < f (ax ) < s.

assim todo número racional r, menor que x, é também menor que


f (ax ) e todo número racional s maior do que x é também maior
que f (ax ). Segue-se que f (ax ) = x para todo x ∈ R. Portanto
f (y) = loga (y) para todo y > 0.
Consideremos agora o caso geral, em que se tem uma função cres-
cente g : R+ −→ R, tal que

g(xy) = g(x) + g(y)

153
Caracterização das funções
Exponenciais e Logarítmicas

sem mais nenhuma hipótese. Então g(1) = 0, como 1 < 2, te-


remos g(2) = b > 0. A nova função f : R+ −→ R, denida
por f (x) = g(x)/b, é crescente, transforma somas em produtos e
cumpre f (2) = 1. Logo, pela primeira parte da demonstração,
tem-se f (x) = log2 (x) para todo x > 0. Isto signica que, para
todo x > 0 vale

x = 2f (x) = 2g(c)/b = (21/b )g(x) = ag(x) ,

com a = 21/b . Tomando loga de ambos os membros da igualdade


ag(x) vem, nalmente: g(x) = loga (x).

11.4 Conclusão

Vimos propriedades para identicar as funções Exponenciais e Lo-


garítmicas. Estes critérios são muito importante pois permitem
decidir se o modelo matemático para um determinado problema é
uma função exponencial ou uma função logarítmica.

RESUMO


Nesta seção vimos teoremas de caracaterização de funções expo-


nenciais, do tipo exponencial e funções logarítmicas.
O teorema que caracteriza as funções exponenciais tem a seguinte
forma:
Teorema da Caracterização das Funções Exponenciais. Seja
f : R −→ R+ uma função monótona injetiva (isto é, crescente ou
descrescente). As seguintes armações são equivalentes:
(1) f (nx) = f (x)n para todo n ∈ Z e todo x ∈ R;

154
Matemática para o Ensino Médio I AULA
(2) f (x) = ax para todo x ∈ R, onde a = f (1); 11
(3) f (x + y) = f (x) · f (y) para quaisquer x, y ∈ R.
A utilidade do teorema, está no fato de que que se vericarmos qual-
quer uma das armações (1), (2) ou (3) acima, estaremos tratando
de uma função exponencial ( desde que f cumpra as hipóteses do
Teorema). Porém, em alguns casos é mais fácil vericar uma, e em
outros, outra das duas propriedades.
As funções do tipo exponencial, ou seja, funções g : R −→ R
tal que g(x) = bax para todo x ∈ R, onde a e b são constantes
positivas, podem ser caracterizadas pelo seguinte teorema:
Teorema de Caracterização das Funções do tipo Expo-
nencial. Seja g : R −→ R uma função monótona injetiva (isto é
crescente ou decrescente) tal que para todo x, h ∈ R quaisquer, o
acréscimo relativo [g(x+h)−g(x)]/g(x) dependa apenas de h, mas
não de x. Então se b = g(0) e a = g(1)/g(0), tem-se g(x) = bax
para todo x ∈ R.
Além disso de todas as funções g : R −→ R+ monótonas injeti-
vas as únicas que transformam uma progressão aritmética numa
progressão geométrica são as funções do tipo exponencial.
Finalmente, o teorema de caracterização das funcões logarítmicas
é enunciado como segue:
Teorema de Caracterização das Funções Logarítmicas. Seja
f : R+ −→ R uma função monótona injetiva tal que f (xy) =
f (x) + f (y) para quaisquer x, y ∈ R+ . Então existe a > 0 tal que
f (x) = loga (x) para todo x ∈ R+ .
Em outras palavras, as funções logarítmicas são as únicas funções
de R+ em R, monótonas e injetivas que transformam o produto
numa soma, ou seja, f (xy) = f (x)+f (y) para quaisquer x, y ∈ R+ .

155
Caracterização das funções
Exponenciais e Logarítmicas

ATIVIDADES


Atividade. 11.1. detemine nos casos abaixo, a dependência de


h e ou x do crescimento relativo [f (x + h) − f (x)]/f (x).
(a)- f (x) = mx + n, onde m > 0 e n > 0.
(b)- f (x) = x2 + 1.
(c)- f (x) = c · ekx , onde c > 0.
(d)- f (x) é a população, no instante x, de uma cultura de bactérias
mantidas sob condições estáveis.

Atividade. 11.2. Prove que uma função do tipo exponencial


ca determinada quando se conhecem dois de seus valores. Mais
precisamente, se f (x) = b · ax e F (x) = b · Ax são tais que f (x1 ) =
F (x1 ) e f (x2 ) = F (x2 ) com x1 6= x2 então a = A e b = B .

Atividade. 11.3. Dados x0 6= x1 e y0 , y1 não-nulos com o mesmo


sinal, prove que existem a > 0 e b tais que b · ax0 = y0 e b · ax1 = y1 .

Atividade. 11.4. A grandeza y se exprime como y = b · at em


função do tempo t. Sejam d o acréscimo que se deve dar a t para
que y dobre e m o acréscimo de t necessário para que y se reduza
à metade. Mostre que m = −d e y = b · 2t/d , logo d = loga 2 =
1/ log2 a.

REFERÊNCIAS BIBLIOGRÁFICAS


IEZZI, Gelson., Fundamentos de Matemática Elementar: Logarti-


mos vol.2, 3.ed., Editora Atual, 1977.
Lima, E.L., A Matemática no Ensino Médio I, 5.ed., SMB, 2006.

156
AULA

12
Trigonometria:
Noções Elementares

META:
Denir noções elementares da trigonometrias.

OBJETIVOS:
Ao m da aula os alunos deverão ser capazes de:
Denir as relações trigonométricas do triângulo retângulo.
Denir medida de ângulos e arcos.
Trigonometria:
Noções Elementares

12.1 Introdução

A trigonometria teve seu início na antiguidade remota, quando se


acreditava que os planetas descreviam órbitas circulares ao redor
da terra, surgindo aí o interesse em relacionar o comprimento da
corda de uma circunferência com o ângulo central por ela subten-
dido. O objeto inicial da trigonometria era o tradicional problema
da resolução de triângulos, que consiste em determinar os seis ele-
menttos dessa gura (três lados e três ângulos) quando se conhecem
três deles, sendo pelo menos um deles um lado. Só posteriormente,
com o surgimento do Cálculo Innitesimal, e do seu prolongamento
que é a Análise Matemática, surgiu a necessidade de atribuir as
noções de seno, cosseno e suas associadas tangente, cotangente,
secante e cossecante, o status de função real de uma variável real.

12.2 Trigonometria do Triângulo Retângulo

Seja ABC um triângulo retângulo de hipotenusa a e ângulos agu-


dos B̂ e Ĉ , opostos respectivamente, aos catetos b e c. Têm-se as
seguintes denições:

Figura 12.1: Triângulo Retângulo e seus elementos.

158
Matemática para o Ensino Médio I AULA
12
cos B̂ = c
a = (cateto adjacente) ÷ (hipotenusa),
sen B̂ = b
a = (cateto oposto) ÷ (hipotenusa), (12.36)
tg B̂ = b
c = (cateto oposto) ÷ (cateto adjacente),

e, analogamente, cos Ĉ = ab , sen Ĉ = ac , tg Ĉ = cb .


É fundamental observar que cos B̂ e sen B̂ dependem apenas do
ângulo B̂ mas não do tamanho do triângulo retângulo do qual B̂
é um dos ângulos agudos. Com efeito, dois quaisquer triângulos
retângulos que tenham um ângulo agudo igual a B̂ são semelhantes.
Se estes triângulos são ABC e A0 B 0 C 0 , com B̂ 0 = B̂ . Então a
semelhança nos dá

b0 a c0 c
0
= e 0
= ,
a b a a

logo
sen B̂ 0 = sen B̂ e cos B̂ 0 = cos B̂.

Figura 12.2: Triângulos retângulos semelhantes.

Portanto o seno e cosseno pertencem ao ângulo, e não ao eventual


triângulo que o contém. A semelhança de triângulos é a base de
sustentação da Trigonometria. Se organizarmos uma tabela com

159
Trigonometria:
Noções Elementares

valores de cos B̂ para todos os ângulos B̂ , a relação c = a · cos B̂



e o Teorema de Pitágoras b = a2 − c2 nos permitirão determinar
os catetos b, c de um triângulo retângulo, uma vez conhecida a
hipotenusa a e um dos ângulos agudos. Mais geralemente, num
triângulo ABC qualquer, a altura h, baixada do vértice C sobre o
¯ sen B̂ . O Teorema de Pitágoras
lado AB , tem expressão h = BC

a2 = b2 + c2 .

aplicado ao triângulo retângulo ABC , com AB = c, AC = b e


BC = a, nos permite concluir que:

c2 b2 b2 + c2 a2
(cos B̂)2 + (sen B̂)2 = + = = = 1.
a2 a2 a2 a2

Observamos, que para ns de notação vamos escrever cos2 B̂ em


vez de (cos B̂)2 e o mesmo com o seno. A relação fundamental

cos2 B̂ + sen2 B̂ = 1.

mostra que a rigor, basta construir uma tabela de senos para ter a
de cossenos, ou vice-versa. É evidente, a partir da denição, que o
cosseno de uma ângulo agudo é igual ao seno do seu complemento
e vice-versa. Daí a palavra cosseno (seno do complemento). Tam-
bém é claro que o seno e cosseno de um ângulo agudo são números
entre 0 e 1.

12.3 Função de Euler e Medida de Ângulos

Começamos observando que a relação fundamental sen2 α+cos2 α =


1 sugere que, para todo ângulo α, os números cos α e sen α são as
coordenadas de um ponto da circunferência de raio 1 e centro na
origem de R2 .

160
Matemática para o Ensino Médio I AULA
Indicaremos com a notação C essa circunferência, que chamaremos 12
de circunferência unitária, ou círculo unitário. Temos portanto
C = {(x, y) ∈ R2 ; x2 + y 2 = 1}.
A m de denir as funções sen : R −→ R e cos : R −→ R, devemos
associar a cada número real t um ângulo e considerar o cosseno e o
seno daquele ângulo. O número t desempenhará , portanto, o papel
de medida do ângulo. Evidentemente, há diversas maneiras de se
medir ângulos dependendo da que se adota. Há duas unidades
de medida de ângulo que se destacam: o radiano e o grau como
veremos.

Denição 12.1. A função de Euler é a função E : R −→ C , que


faz corresponder a cada número real t o ponto E(t) = (x, y) da
circunferência unitária obtido da seguinte maneira

• E(0) = (1, 0)

• Se t > 0, percorremos sobre a circunfência C , a partir do


ponto (1, 0), um caminho de comprimento t, sempre andando
no sentido positivo( sentido anti-horário). O ponto nal do
caminho será chamado E(t).

• Se t < 0 será a extremidade nal de um caminho sobre C ,


de comprimento |t|, que parte do ponto (1, 0) e percorre C
sempre no sentido negativo ( isto é, sentido horário).

A função de Euler E : R −→ C pode ser imaginada como o pro-


cesso de enrolar a reta, identicada a um o inextensível, sobre a
circunferência C (pensada como um carretel) de modo que o ponto
0 ∈ R caia sobre o ponto (1, 0) ∈ C .
Cada vez que o ponto t descreve na reta um intervalo de compri-
mento l, sua imagem E(t) percorre sobre a circunferência C um

161
Trigonometria:
Noções Elementares

Figura 12.3: Função de Euler.

arco de igual comprimento l. Em particular, como a circunferência


unitária C tem comprimento igual a 2π , quando o ponto t descreve
um intervalo de comprimento 2π , sua imagem E(t) dá uma volta
para todo t ∈ R, tem-se E(t + 2π) = E(t) e, mais geralmente, para
todo k ∈ Z, tem-se E(t + 2kπ) = E(t), seja qual for t ∈ R.
Reciprocamente, se t < t0 em R são tais que E(t) = E(t0 ), isto
signica que quando um ponto s da reta varia de t a t0 sua imagem
E(s) se desloca sobre C , no sentido positivo, partindo de E(t),
dando um número inteiro k de voltas e retornando ao ponto de
partida E(t) = E(t0 ). A distância percorrida é igual a 2kπ , logo
t0 = t + 2kπ , pois o comprimento do caminho percorrido por E(s)
é por denição igual à distância percorrida por s sobre a reta R.
quando t0 < t tem-se k < 0.
Escrevemos A = (1, 0) e O = (0, 0). Para cada t ∈ R, ponhamos
B = E(t). Diz-se neste caso que o ângulo AÔB mede t radianos.

Observação 12.1. Segue da denição algumas observações:

• Pode-se ter B = E(t) com t < 0. Portanto esta forma de

162
Matemática para o Ensino Médio I AULA
12

Figura 12.4: Função de Euler.

medida é orientada: é permetido a uma ângulo ter medida


negativa.

• A medida do ângulo AÔB é determinada apenas a menos


de um múltiplo inteiro de 2π , pois B + E(t) implica B =
E(t + 2kπ) para todo k ∈ Z. Assim por exemplo, o ângulo
de 1 radiano é também um ângulo de 1−2π radianos. De um
modo mais geral, se B = E(t) então B = E(t − 2π) pois há
dois arcos que vão de A = (1, 0) até B ; um de comprimento
|t| e outro de comprimento |t − 2π|.

• De acordo com esta denição, o ângulo AÔB mede 1 radiano


se, e somente se, o arco AB da circunferência C , por ele
subtendido, tem comprimento igual a 1, isto é, igual ao raio
da circunferência. Mais geralmente, numa circunferência de
raio r, a medida de um ângulo central em radianos é igual
a l/r, onde l é o comprimento do arco subtendido por esse
ângulo.

163
Trigonometria:
Noções Elementares

Figura 12.5: Medida do ângulo em radianos.

• A medida do ângulo AÔB em radianos também pode ser


expressa como 2a/r2 , em termos da área a do setor circular
AOB e do raio r.

Mostramos o último ponto da observação. A área do setor circu-


lar AOB é uma função crescente do comprimento l do arco AB .
Como se vê facilmente, se o arco AB 0 tem comprimento n vezes
maior do que o arco AB . Então a área do setor AOB 0 é igual a
n vezes a área de AOB . Segue-se então do Teorema Fundamental
da Proporcionalidade que a área a é uma função linear do com-
primento l: a = c · l, onde c é uma constante. Para determinar
o valor de c, basta observar que, quando o setor é todo o círculo
de raio r, o arco correspondente é toda a circunferência. Tem-se
então a = πr2 e l = 2πr. Logo πr2 = c · 2πr, donde c = r/2.
Portanto a área do setor AOB se relaciona com o comprimento l
do arco AB pela igualdade a = lr/2. Segue-se que

l 2a
= 2.
r r

Como l/r é a medida do ângulo AOB em radianos, concluímos

164
Matemática para o Ensino Médio I AULA
daí que esta medida também vale 2a/r2 , onde a é a área do setor 12
AOB e r é o raio do círculo.
Podíamos também ter denido uma função G : R −→ C pondo
ainda G(0) = (1, 0) e estipulando que, para s > 0, G(s) fosse o
ponto da circunfer6encia unitária obtido a partir do ponto (1, 0)
quando se percorre , ao longo de C , no sentido positivo, um cami-
nho de comprimento 360 s.

E para s < 0, G(s) seria denido de
forma análoga, com o percurso no sentido negativo de C .
A função G : R −→ C tem propriedades semelhantes às de E , pois
 2π 
G(t) = E t
360

para todo t ∈ R. Em particular, G(t0 ) = G(t) se, e somente


se, t0 = t + 360k , onde k ∈ Z. Se A = (1, 0), O = (0, 0) e
B = G(s), diz-se que o ângulo AÔB mede s graus. O ângulo AÔB
mede 1 grau quando B = G(1), ou seja, quando o arco AB tem
comprimento igual a 2π/360. Noutras palavras, o ângulo de um
grau é aquele que subtende um arco igual a 1/360 da circunferência.
Escreve-se 1 grau = 1◦ e 1 radiano = 1rad.
Como a circunferência inteira tem 2π radianos ou equivalente 360
graus, segue-se que 2π rad = 360◦ , ou seja

360 ◦
1rad = = 57, 3graus

É bom ter em mente relações como 180◦ = π rad, 90◦ = π/2rad,


etc.

RESUMO


165
Trigonometria:
Noções Elementares

Nesta seção indtroduzimos os elementos essenciais para a denição


das funções trigonométricas. Começamos denindo a relações tri-
gonométricas no triângulo retângulo, seno, cosseno, e tangente.
Porém estas funções só estão denidas para ângulos entre 0◦ e 90◦ .
Estabelecemos também, as duas maneiras canônicas de medir ar-
cos: radianos e graus. Por m, denimos a Função de Euler que
permite identicar cada ponto da reta com um ponto da circunfer-
ência de raio 1.

ATIVIDADES


Atividade. 12.1. Para medir a largura de um rio de margens


paralelas sem atravessá-la, um observador no ponto A visa um
ponto B na margem oposta( suponha que AB é perpendicular às
margens). De A, ele traça uma perpendicular à linha AB e marca
sobre ela um ponto C , distando 30 m de A. Em seguida ele se
desloca para C , visa o spontos B e A, e mede o ângulo B ĈA = 70o .
Sabendo que a distância, sobre AB , de A à margem do rio é de 3
m e que tg 70o = 2, 75, calcular a lagura do rio.

Atividade. 12.2. Um observador em uma planície vê ao longe o


topo de uma montanha segundo um ângulo de 15o (ângulo no plano
vertical formado por um ponto no topo da montanha, o observador
e o plano horizontal). Após caminhar um distância d em direção
a montanha, ele passa a vê-la segundo um ângulo de 30o . Qual
a altura da montanha? Suponha que o observador encontrou um
ângulo α na primeira medição e β na segunda medição. Determinar
a altura da montanha em função de α, β e d.

166
Matemática para o Ensino Médio I AULA
Atividade. 12.3. (a)- Os lados de um triângulo retângulo estão 12
em progressão aritmética. Qual o cosseno do maior ângulo agudo?
(b)- Os lados de um triângulo retângulo estão em progressão ge-
ométrica. Qual é o cosseno do menor de seus ângulos?

Atividade. 12.4. (a)-Prove que tg(x/2) = sen(x)


1+cos(x) . Calcule as
funções trigonométricas de 15o .
1−t2
(b)- Se tg(x/2) = t prove que sen(x) = 2t
1+t2
, cos(x) = 1+t2
e
tg(x) = 2t
1−t2
.

REFERÊNCIAS BIBLIOGRÁFICAS


CARMO, M.P., Trigonometria e Números Complexos, 1.ed., SBM,


1992.
IEZZI, Gelson., Fundamentos de Matemática Elementar: Trigonome-
tria, vol.3, 2.ed., Editora Atual, 1977.
Lima, E.L., A Matemática no Ensino Médio I, 5.ed., SMB, 2006.

167
AULA

Funções Trigonométricas
13
META:
Denir e estudar as propriedades das funções trigonométricas.

OBJETIVOS:
Ao m da aula os alunos deverão ser capazes de:
Denir as funções trigonométricas elementares.
Identicar as propriedades das funções trigonométricas e esboçar
gráco das mesmas.
Estabelecer a principais relações fundamentais entre as funções
trigonométricas.

PRÉ-REQUISITOS
Denições elementares dos elementos trigonométricos: arco, ân-
gulo, medida de arco e ângulo.
Funções Trigonométricas

13.1 Introdução

As funções trigonométricas surgem de maneira natural, com o sur-


gimento do Cálculo Innitesimal e posteriormente da Análise Ma-
temática, onde surge a necessidade de atribuir às noções de seno,
cosseno, tangente e suas associadas cotangente, secante e cosse-
cante, o status de função real de uma variável real. Uma pro-
priedade fundamental das funções trigonométricas é que elas são
periódicas e desta forma, são especialmente adaptadas para descre-
ver fenômenos de natureza periódica, oscilatória ou vibratória, os
quais abundam no universo: movimento de planetas, som, corrente
elétrica alternada, batimentos cardíacos, etc.
A importânncia das funções trigonométricas foi grandemente re-
forçada com a descoberta de Joseph Fourier, em 1822, de que toda
função periódica ( com ligeiras e naturais restrições) é uma soma
(nita ou innita) de funções do tipo a cos(nx) + b sen(nx).

13.2 Função Seno e Cosseno

Consideremos novamente, como no capítulo anterior a Função de


Euler E : R −→ C , a qual faz corresponder, xado uma origem
A em C , cada número real t um ponto E(t) = (x, y) sobre C da
seguinte maneira: Se x > 0, percorremos uma distância t sobre C
no sentido positivo (anti-horário a partir de A) e E(t) é o ponto
atingido; se x < 0, percorremos uma distância |t| sobre C no sen-
tido negativo ( horário a partir de A) e E(t) é o ponto atingido; e
nalmente se t = 0 então E(t) = A.
Consideremos o sistema de coordenadas Oxy onde a origem do
sistema é o centro de C e tomamos A = (1, 0).

170
Matemática para o Ensino Médio I AULA
Denição 13.1. As funções sen : R −→ R e cos : R −→ R, 13
chamadas de Função Seno e Função Cosseno, respectivamente, são
denidas pondo-se, para cada real t ∈ R:

E(t) = (cos(t), sen(t)).

Noutras palavras, x = cos(t) e y = sen(t) são respectivamente a


abcissa e a ordenada do ponto E(t) da circunferência unitária.

Figura 13.1: Interpretação geométrica das funções seno e cosseno


de um arco t.

13.2.1 Propriedades

Seguem, diretamente da denição as seguintes propriedades:

1. A imagem das funções Seno e Cosseno é o intervalo [−1, 1],


isto é −1 ≤ sen(t) ≤ 1 e −1 ≤ cos(t) ≤ 1. A vericação é
imediata, pois se E(t) está sobre o círculo, sua ordenada e
abcissa só podem variar entre −1 e 1.

2. Se t é do primeiro ou segundo quadrante, então sen(t) é pos-


itivo e se t é do terceiro ou quarto quadrantes, então sen(t)

171
Funções Trigonométricas

é negativo. Já o cos(t) é positivo no primeiro e quarto quad-


rantes e negativo no segundo e terceiro.

3. Se t percorre o primeiro ou quarto quadrantes, então sen(t) é


crescente e é decrescente quando t percorre o segundo e ter-
ceiro quadrantes. A função cos(t) é crescente quando t per-
corre o terceiro e quarto quadrantes e é decrescente quando
t percorre o primeiro e segundo. Para ver estes fatos, basta
observar a denição. É imediato, por exemplo perceber que
se t percorre o primeiro quadrante então E(t) percorre C a
partir de A, de modo que sua abcissa decresce de 1 até 0 (
daí a função cosseno ser descrescente no primeiro quadrante)
e a ordenada de E(t), neste caso cresce de 0 até 1 ( daí a
função seno ser crescente no primeiro quadrante).

4. As funções seno e cosseno são funções periódicas de período


2π . De fato, como vimos anteriormente E(t) = E(t + 2kπ).

Figura 13.2: Funções Seno e Cosseno num mesmo sistema de eixos


coordenados.

172
Matemática para o Ensino Médio I AULA
Os gracos das funções Seno e Cosseno estão representados na 13
Figura 13.2. O gráco da função seno chama senóide ( seu traçado
corresponde a linha mais na na Figura 13.2) e o gráco da função
cosseno chama-se cossenóide ( corresponde a linha mais espessa na
Figura 13.2).

Observação 13.1. Considerando a função de Euler E : R −→ C ,


é claro que se E(t) = (x, y) então E(−t) = (x, −y). Para vericar
este fato, basta usar um argumento geométrico de congruência de
triângulos, conforme Figura 13.3. Da mesma maneira temos que

Figura 13.3: Demostração geométrica das propriedades E(−t) =


(x, −y) e E(t + π) = (−x, −y), onde E(t) = (x, y).

173
Funções Trigonométricas

E(t + π) = (−x, −y), conforme Figura 13.3.


Ainda é possível mostrar que:
π π
E(t + ) = (−y, x), E( − t) = (y, x), E(π − t) = (−x, y)
2 2
Uma função f : R −→
(13.37)
R chama-se função par se
f (x) = f (−x) para todo Temos que para todo t ∈ R
x ∈ R. Se f (x) = −f (−x)
para todo x ∈ R então f é E(t) = (cos(t), sen(t)), E(−t) = (cos(−t), sen(−t)).
dita função ímpar.
Mas como vimos acima, se E(t) = (x, y) então, E(−t) = (x, −y).
Logo, obtemos comparando estas duas expressões para E(−t).

cot(−t) = cos(t), sen(−t) = − sen(t),

para todo t ∈ R. Segue que a função cosseno é par e a função seno


é ímpar. De maneira semelhante, das relações envolvendo E(t)
acima, seguem as seguintes expressões.

cos(t + π) = − cos(t), sen(t + π) = − sen(t),


cos(t + π/2) = − sen(t), sen(t + π/2) = cos(t),
cos(π/2 − t) = sen(t), sen(π/2 − t) = cos(t),
cos(π − t) = − cos(t), sen(π − t) = sen(t).

13.3 Função Tangente

Consideremos o circulo unitário C e c a reta tangente a C no ponto


A. Esta reta será denominada eixo das tangentes.

Denição 13.2. Seja t ∈ R, t 6= π


2 + kπ , k ∈ Z e seja P =
E(t) a imagem no círculo C . Consideramos a reta OP e seja T
sua intersecção com o eixo das tangentes. A medida algébrica do
−→
segmento AT é chamada de tangente de t.
Denominamos a função tangente a função tg : D −→ R que associa
a cada t ∈ D, t 6= π/2 + kπ , o número real AT

174
Matemática para o Ensino Médio I AULA
13

Figura 13.4: Construção geométrica para a determinação da tan-


gente de um arco.

Obsereve que sempre que t = π/2 + kπ , k inteiro o ponto P =


E(t) é de tal forma que o segmento OP não intercepta o eixo das
tangentes e desta forma, não esta denida a tangente.

13.3.1 Propriedades da função Tangente

A função tangente tem as seguintes propriedades:

1. O domínio da função tangente é D = {t ∈ R; t 6= π


2 +
kπ, onde k ∈ Z}.

2. A imagem da função tangente é R, isto para cada y real existe


um t real tal que tg(t) = y .

De fato, dado y ∈ R, seja T sobre o eixo das tangentes tal que


AT = y (segmento orientado). Constuímos a reta OT , ob-
servando que ela intersecta o círculo unitário em dois pontos
P e P 0 , imagens dos reais t, cuja tangente é y .

175
Funções Trigonométricas

3. A tangente é positiva no primeiro e terceiro quadrantes e


negativa no segundo e quarto quadrantes.

4. Se t percorre qualquer um dos quatro quadrantes então tg(x)


é crescente.

5. A função tangente é periódica de período π .

De fato, se tg(t) = AT e k ∈ Z, então tg(t + kπ) = AT , pois


t e t + kπ tem imagens P e P 0 que estão na mesma reta que
contem a origem 0, ou seja, a reta OP e OP 0 são identicas
e logo determinam a mesma intersecção T com o eixo das
tangentes. Segue que tg(t) = tg(t + kπ) para todo t real tal
que t 6= π
2 + kπ e a função tangente é periódica. Seu período
é o menor valor positivo de kπ , isto é, π .

6. O gráco da função tangente é chamado de tangentóide e é


dado como na gura abaixo:

Figura 13.5: Gráco da Função tangente.

176
Matemática para o Ensino Médio I AULA
13.4 Função Cotangente 13
Seja B = (1, 0) o ponto do círculo unitário e d a reta tangente
ao círculo no ponto B . Chamaremos esta reta orientada ( com a
mesma orientação do eixo 0x) de eixo das cotangentes.

Denição 13.3. Dado o número real t, t 6= kπ , k ∈ Z, seja P =


E(t) sua imagem no círculo unitário. Consideremos a reta OP e
seja D sua intersecção com o eixo das cotangentes. Denominamos
contangente de t, e indicamos por cotg(t), a medida algébrica do
−−→
segmento BD.
Denominamos função cotangente a função cotg : D −→ R que
associa a cada t ∈ D, t 6= kπ , o número real BD

Observe que sempre que t = kπ , k inteiro o ponto P = E(t) é de


tal forma que o segmento OP não intercepta o eixo das cotangentes
e desta forma, não esta denida a cotangente de t.

13.4.1 Propriedades da função Cotangente

A função cotangente tem as seguintes propriedades:

1. O domínio da função cotangente é D = {t ∈ R; t 6= kπ, onde k ∈ Z}.

2. A imagem da função cotangente é R.

3. A cotangente é positiva no primeiro e terceiro quadrantes e


negativa no segundo e quarto quadrantes.

4. Se t percorre qualquer um dos quatro quadrantes então cotg(x)


é decrescente.

5. A função cotangente é periódica de período π .

177
Funções Trigonométricas

6. O gráco da função cotangente é dado como na gura abaixo:

Figura 13.6: Gráco da Função Cotangente.

13.5 Função Secante

Denição 13.4. Dado o número real t, t 6= kπ/2, k ∈ Z, seja


P = E(t) sua imagem no círculo unitário. Consideremos a reta
s tangente ao círculo em P e seja S sua intersecção com o eixo
dos cossenos. Denominamos secante de t, e indicamos por sec(t), a
−→
medida algébrica do segmento OS , ou seja, a abcissa OS do ponto
S.
Denominamos função secante a função sec : D −→ R que associa
a cada t ∈ D, t 6= kπ/2, o número real OS .

Observe que sempre que t = kπ/2, k inteiro, o ponto P = E(t)


é de tal forma que a reta tangente a C por P é paralela ao eixo

178
Matemática para o Ensino Médio I AULA
13

Figura 13.7: Construção geométrica para a denição da Secante


de um arco.

dos cossenos e desta forma, não existe o ponto S e assim não está
denida a função secante de t.

13.5.1 Propriedades da função Secante

A função cotangente tem as seguintes propriedades:

1. O domínio da função secante é D = {t ∈ R; t 6= 2 , onde



k ∈ Z}.

2. A imagem da função secante é R − (−1, 1).

3. A secante é positiva no primeiro e quarto quadrantes e ne-


gativa no segundo e terceiro quadrantes.

4. Se t percorre o primeiro ou segundo quadrantes então sec(t)


é crescente e se t percorre o terceiro ou quarto quadrantes
sec(t) é decrescente.

5. A função secante é periódica de período 2π .

6. O gráco da função secante é dado como na gura abaixo:

179
Funções Trigonométricas

Figura 13.8: Gráco da Função Secante.

13.6 Função Cossecante

Denição 13.5. Dado o número real t, t 6= kπ , k ∈ Z, seja


P = E(t) sua imagem no círculo unitário. Consideremos a reta
s tangente ao círculo em P e seja Q sua intersecção com o eixo dos
senos. Denominamos cossecante de t, e indicamos por cossec(t), a
−−→
medida algébrica do segmento OQ, ou seja, a abcissa OQ do ponto
Q.

Denominamos função cossecante a função cossec : D −→ R que


associa a cada t ∈ D, t 6= kπ , o número real OQ.

Observe que sempre que t = kπ , k inteiro o ponto P = E(t) é de


tal forma que a reta tangente a C por P é pararlela ao eixo dos
senos e desta forma não existe o ponto Q e assim não está denida
a função cossecante de t.

180
Matemática para o Ensino Médio I AULA
13

Figura 13.9: Construção geométrica para a denição da Secante


de um arco.

13.6.1 Propriedades da função Cossecante

A função cotangente tem as seguintes propriedades:

1. O domínio da função cossecante é D = {t ∈ R; t 6= kπ, onde k ∈ Z}.

2. A imagem da função cossecante é R − (−1, 1).

3. A cossecante é positiva no primeiro e segundo quadrantes e


negativa no terceiro e quarto quadrantes.

4. Se t percorre o primeiro ou quarto quadrantes então cossec(t)


é decrescente e se t percorre o segundo ou terceiro quadrantes
cossec(t) é decrescente.

5. A função cossecante é periódica de período 2π .

6. O gráco da função cossecante é dado como na gura abaixo:

181
Funções Trigonométricas

Figura 13.10: Gráco da Função Cossecante.

13.7 Relações Fundamentais

Vamos ver que as funções trigonométricas tg, cotg, sec e cossec


podem ser expressas em função das funções sen e cos. Assim,
juntamente com a propriedade elementar, já citada

sen2 (t) + cos2 (t) = 1

mostraremos, que a partir de uma função é possível obter todas as


outras.

Proprosição 13.19. Para todo t real, t 6= π/2 + kπ, k ∈ Z vale a


relação
sen(t)
tg(t) = .
cos(t)
Prova: Se t 6= kπ então a imagem E(t) = P no círculo é diferente
dos ponto (1, 0) e (−1, 0) e desde que, por hipótese t 6= π/2 + kπ ,
então E(t) = P também é diferente dos pontos (0, 1) e (0, −1).

182
Matemática para o Ensino Médio I AULA
13

Figura 13.11: Construção gemométrica para a prova da pro-


sen(t)
priedade tg(t) = cos(t) .

Logo cam sempre denido os triângulos OAT e OP2 P , conforme


a Figura 13.11, que são semelhantes.
Portanto
|AT | |P2 P |
= ,
|OA| |OP2 |
e portanto
| sen(t)|
| tg(t)| = .
| cos(t)|
Agora, analisando a tabela abaixo, vemos que a função tg(t) e o
quociente sen(t)/ cos(t) possuem o mesmo sinal logo vale tg(t) =
sen(t)/ cos(t).
Quadrante Sinal de tg(t) Sinal de sen(t)/ cos(t)
1 + +
2 - -
3 + +
4 - -
No caso em que t = kπ , temos que sen(kπ) = 0 e assim tg(t) =
sen(t)
0= cos(t) , o que conclui a prova.

183
Funções Trigonométricas

Proprosição 13.20. Para todo t real, t 6= kπ, k ∈ Z vale a relação


cos(t)
cotg(t) = .
sen(t)

Prova: A demonstração segue de maneira análoga a proposição


anterior.

Proprosição 13.21. Para todo t real, t 6= π/2 + kπ, k ∈ Z vale a


relação
1
sec(t) = .
cos(t)
Prova: Se t 6= kπ então a imagem E(t) = P no círculo é diferente
dos pontos (1, 0) e (−1, 0) e desde que, por hipótese t 6= π/2 + kπ ,
então E(t) = P também é diferente dos pontos (0, 1) e (0, −1).

Figura 13.12: Construção gemométrica para a prova da pro-


priedade sec(t) = cos(t) .
1

Logo cam sempre denido os triângulos OP S e OP2 P , conforme


a Figura 13.12, que são semelhantes.
Portanto
|OS| |OP |
= ,
|OP | |OP2 |

184
Matemática para o Ensino Médio I AULA
e portanto
1
13
| sec(t)| = .
| cos(t)|
Agora, analisando a tabela abaixo, vemos que a função sec(t) e
o quociente 1/ cos(t) possuem o mesmo sinal logo vale sec(t) =
1/ cos(t).
Quadrante Sinal de sec(t) Sinal de 1/ cos(t)
1 + +
2 - -
3 - -
4 + +
No caso em que t = kπ , temos que sec(kπ) = 1 = cos(t) se k é par,
ou sec(kπ) = −1 = cos(t), se k é ímpar.

Proprosição 13.22. Para todo t real, t 6= kπ, k ∈ Z vale a relação


1
cossec(t) = .
sen(t)
Prova: a demonstração segue de maneira análoga a proposição
anterior.
Segue das proposições acima, o seguinte resultado:

Corolário 13.1. Para todo t real, t 6= kπ/2, k ∈ Z valem as


relações:
1
cotg(t) = tg(t) ,

tg2 (t) + 1 = sec2 (t),


cotg2 (t) + 1 = cossec2 (t),
1
cossec2 (t) = 1+tg2 (t)
,
2
tg (t)
sen2 (t) = 1+tg2 (t)
.

RESUMO


185
Funções Trigonométricas

Nesta seção denimos as funções trigonométricas, extendendo o


conceito das relações seno, cosseno, tangente, e suas derivadas, do
triângulo retângulo à uma função real.
Denição: As funções sen : R −→ R e cos : R −→ R, chamadas
de Função Seno e Função Cosseno, respectivamente, são denidas
pondo-se, para cada real t ∈ R:

E(t) = (cos(t), sen(t)).

Noutras palavras, x = cos(t) e y = sen(t) são respectivamente a


abcissa e a ordenada do ponto E(t) da circunferência unitária.
Usando, eixos orientados, podemos denir ainda outras quatro
funções trigonométricas:
Denição: Seja t ∈ R, t 6= π2 +kπ , k ∈ Z e seja P = E(t) a imagem
no círculo C . Consideramos a reta OP e seja T sua intersecção
−→
com o eixo tas tangentes. A medida algébrica do segmento AT é
chamada de tangente de t.
Denominamos a função tangente a função tg : D −→ R que associa
a cada t ∈ D, t 6= π/2 + kπ , o número real AT .
Denição: Dado o número real t, t 6= kπ , k ∈ Z, seja P =
E(t) sua imagem no círculo unitário. Consideremos a reta OP de
seja D sua intersecção co o eixo das cotangentes. Denominamos
contangente de t, e indicamos por cotg(t), a medida algébrica do
−−→
segmento BD.
Denominamos função cotangente a função cotg : D −→ R que
associa a cada t ∈ D, t 6= kπ , o número real BD.
Denição: Dado o número real t, t 6= kπ/2, k ∈ Z, seja P = E(t)
sua imagem no círculo unitário. Consideremos a reta s tangente ao
círculo em P e seja S sua intersecção com o eixo dos cossenos. De-

186
Matemática para o Ensino Médio I AULA
nominamos secante de t, e indicamos por sec(t), a medida algébrica
−→
13
do segmento OS , ou seja, a abcissa OS do ponto S .
Denominamos função secante a função sec : D −→ R que associa
a cada t ∈ D, t 6= kπ/2, o número real OS .
Denição: Dado o número real t, t 6= kπ , k ∈ Z, seja P = E(t)
sua imagem no círculo unitário. Consideremos a reta s tangente
ao círculo em P e seja Q sua intersecção com o eixo dos senos.
Denominamos cossecante de t, e indicamos por cossec(t), a medida
−−→
algébrica do segmento OQ, ou seja, a abcissa OQ do ponto Q.
Denominamos função cossecante a função cossec : D −→ R que
associa a cada t ∈ D, t 6= kπ , o número real OQ.

ATIVIDADES


Atividade. 13.1. Esboce o gráco da função y = 3 − 2 cos(2x −


π/4) e da função y = cos(x) num mesmo sistema de coordenadas.
Esboce o gráco da função y = 1 + 1
4 sen( x2 + π/2) e y = sen(x)
num mesmo sistema de coordenadas.

Atividade. 13.2. Para que valores de t existe x satisfazendo as


igualdades:
(a)- cos(x) = 2t−1 .
t+2

(b)-sen(x) = t−2 .
t−1

(c)- tg(x) = t2 − 5t + 4.

Atividade. 13.3. Esboçar o gráco, determinar o domínio e o


período da função f (x) = tg(2x + π/6).

Atividade. 13.4. Calcular sen(x) e cos(x) sabendo que 3 cos(x)+


sen(x) = −1.

187
Funções Trigonométricas

Calcular sen(x) e cos(x) sabendo-se que 5 sec(x) − 3 tg2 (x) = 1.

Atividade. 13.5. Se sen(x) + cos(x) = a e sen(x). cos(x) = b,


determinar a relação entre a e b independente de x.

Atividade. 13.6. Dado que sen(x). cos(y) = m, Mostrar que


sen4 (x) + cos4 (x) = 1 − 2m2 e sen6 (x) + cos6 (x) = 1 − 3m2

Atividade. 13.7. Prove que valem as igualdades abaixo e exiba


o domínio em que vale a igualdade.
(a)- (1 + cot2 (x))(1 − cos2 (x)) = 1.
1−cos(x) 1−cos(x)
(b)- sen(x). cos(x) + sen(x) = tg(x) + tg(x).
sen(x) cos(x)
(c)- cossec(x) + sec(x) = 1.
cotg2 (x)
(d)- 1+cotg2 (x)
= cos2 (x).
sen3 (x)−cos3 (x)
(e)- sen(x)−cos(x) = 1 + sen(x) cos(x).
cos(x)+cotg(x)
(f)- tg(x)+sec(x) = cos(x). cotg(x).

REFERÊNCIAS BIBLIOGRÁFICAS


CARMO, M.P., Trigonometria e Números Complexos, 1.ed., SBM,


1992.
IEZZI, Gelson., Fundamentos de Matemática Elementar: Trigonome-
tria, vol.3, 2.ed., Editora Atual, 1977.
Lima, E.L., A Matemática no Ensino Médio I, 5.ed., SMB, 2006.

188
AULA

Fórmulas de Adição e
14
Leis Fundamentais

META:
Obter e demonstrar as relações do seno, cosseno e tangente da
soma e diferenças de arcos.
Demosntrar a lei dos cossenos e a lei dos senos.

OBJETIVOS:
Ao m da aula os alunos deverão ser capazes de:
Provar as relações de soma e diferença de arcos das funções trigonométri-
cas elementares.
Demonstrar as fórmulas da lei dos senos e cossenos.
Aplicar as relações trigonométricas, para resolução de triângulos.

PRÉ-REQUISITOS
Denição e propriedades das funções trigonométricas.
Fórmulas de Adição e
Leis Fundamentais

14.1 Introdução

Nesta seção, vamos deduzir as fórmulas que calculam as funções


trigonométricas da soma e diferença de dois arcos, cujas funções
são conhecidas. Também, vamos estudar duas leis fundamentais
da trigonometria: Lei dos Senos e Lei dos Cossenos. Estas leis têm
importantes aplicações na geometria, como por exemplo, conhe-
cendo os lados de um triângulo qualquer, podemeos calcular seus
ângulos,(através de seus cossenos), alturas, medianas, etc.

14.2 Fórmulas de Adição

As fórmulas clássicas que exprimem cos(α + β) e sen(α + β) em


termos de sen α, sen β , cos α e cos β podem ser demosntradas de
diversas maneiras. Vamos dar uma prova, que parece a mais direta.

Figura 14.1: Fórmula de Adição.

190
Matemática para o Ensino Médio I AULA
Consideremos a construção geométrica da Figura 14.1, onde temos 14
que CB 0 ⊥ OB 0 . Seguem então as seguintes relações:

OA = cos(α + β),
OB 0 = cos(β),
B 0 C = sen(β),
AB = A0 B 0 = sen(α) · sen(β),
OB = cos(α) · cos(β).

Logo

OA = OB − AB = cos(α) · cos(β) − sen(α) · sen(β).

Noutras palavras,

cos(α + β) = cos(α) · cos(β) − sen(α) · sen(β), (14.38)

chamada fórmula do cosseno da soma. Tomando-se −β em vez de


β na fórmula (14.38) acima, e usando o fato que cos(−β) = cos(β)
e sen(−β) = − sen(β), obtemos:

cos(α − β) = cos(α) · cos(β) + sen(α) · sen(β). (14.39)

chamada fórmula do cosseno da diferença. Já vimos que sen(π/2 +


t) = cos(t) e cos(π/2 + t) = − sen(t). Então temos que:

sen(α + β) = − cos(π/2 + α + β).


= − cos(π/2 + α) cos(β) + sen(π/2 + α) sen(β),

ou seja,

sen(α + β) = sen(α) · cos(β) + sen(β) · cos(β). (14.40)

191
Fórmulas de Adição e
Leis Fundamentais

As fórmulas para seno e cosseno do arco duplo são consequências


diretas das fórmulas acima. Assim temos:

sen(2α) = 2 sen(α) · cos(β) cos(2α) = cos2 (α) − sen2 (α).

Exemplo 14.1. Como aplicação das fórmulas de adição, vamos


mostrar como determinar as coordenadas do ponto A0 = (x0 , y 0 ),
obtido do ponto A = (x, y) por meio de uma rotação de um ângulo
θ em torno da origem de R2 . Chamamos de α o ângulo do eixo

Figura 14.2: Coordenadas do Ponto A0 como rotação do Ponto A


em torno da origem R2 .

Ox com o segmento OA e escrevemos r = OA. Então r = OA0 e


se tem:

x = r·cos α, y = r·sen α, x0 = r·cos(α+θ), y 0 = r·sen(α+θ).

As fórmulas de adição fornecem

x0 = r cos α · cos θ − r sen α · sen θ = x cos θ − y sen θ.


y 0 = r cos α · sen θ − r sen α · cos θ = x sen θ + y cos θ.

192
Matemática para o Ensino Médio I AULA
Portanto a rotação de ângulo θ em torno da origem é a função 14
T : R2 −→ R2 denida por

T (x, y) = (x cos θ − y sen θ, x sen θ + y cos θ).

Exemplo 14.2. Outra aplicação das fórmulas de adição consiste


em obter uma parametrização racional da circunferência unitária
C . Começamos observando que para todo x real vale a igualdade
 1 − x 2 2  2x 2
+ = 1.
1 + x2 1 + x2

Ou seja, para todo x real, os termos entre parênteses acima são,


respectivamente, a abcissa e a ordenada de um ponto da circunfe-
rência unitária C , isto é, são o cosseno e o seno de um ângulo β .
Além disso, todo número real x é a tangente de um (único) ângulo
α ∈ − π/2, π/2 . Logo a igualdade acima signica que, para cada


um desses valores de α, existe um β tal que

1 − tg2 α 2 tg α
= cos β e = sen β.
1 + tg2 α 1 + tgα

É fácil mostrar que β = 2α usando as fórmulas de cos(2α) e


sen(2α). Basta substituir tg α por sen α/ cos α no primeiro mem-
bro destas igualdades e fazer simplicações óbvias para ver que

1 − tg2 α 2 tg α
= cos(2α) e = sen(2α).
1 + tg2 α 1 + tg2 α

Equivalentemente:
α
1 − tg2 2 tg α2
cos α = 2
α e sen α = .
1 + tg2 2 1 + tg2 α2

Dado o ponto arbitrário B = (cos α, sen α) da circunferência unitária,


como o ângulo incrito AP̂ B é a metade do ângulo central α = AÔB
que subtende o mesmo arco AB , vemos que tg α2 é a inclinação da

193
Fórmulas de Adição e
Leis Fundamentais

reta P B , onde P = (−1, 0). Mantendo o ponto P xo e fazendo


α
2 variar em (−π/2, π/2), cada semi-reta de inclinação igual a tg α2
corta a circunferência unitária num único ponto B = (cos α, sen α).
Todos os pontos da circunferência podem ser obtidos assim, menos
o próprio ponto P .
A correspondência
 1 − x2 2x 
x 7−→ ,
1 + x2 1 + x2

é uma parametrização racional de C . Para x ∈ Q, o ponto que lhe


corresponde tem ambas as coordenadas racionais.

14.3 Lei dos Senos e Lei dos Cossenos

Dado o triângulo ABC , sejam a, b, c as medidas dos lados BC ,


AC e AB respectivamente. Se ainda h = AP a altura baixada
de A sobre o lado BC . Há duas possibilidades, ilustradas como
na Figura 14.3, conforme o ponto P pertença ao segmento BC ou
esteja sobre seu prolongamento.
No primeiro caso, seja x = BP = c·cos B̂ . O Teorema de Pitágoras
aplicado aos triângulos ABP e AP C fornece as igualdades

c2 = h2 + x2 ,
b2 = h2 + (a − x)2 = h2 + x2 + a2 − 2ax
= h2 + x2 + a2 − 2ac · cos B̂.

Comparando estas igualdades obtemos

b2 = a2 + c2 − 2ac · cos B̂.

No segundo caso x = BP = c · cos(π − B̂) = −c · cos B̂ . ( Note


que cos B̂ < 0, logo −c · cos B̂ é positivo.) Novamente Pitágoras,

194
Matemática para o Ensino Médio I AULA
14

Figura 14.3: Construção Geométrica para a prova da lei dos senos


e cossenos.

aplicado aos triângulos AP B e AP C nos dá:

c2 = h2 + x2 ,
b2 = h2 + (a + x)2 = h2 + x2 + a2 + 2ax
= h2 + x2 + a2 − 2ac · cos B̂.

Daí resulta, como antes, que

b2 = a2 + c2 − 2ac · cos B̂. (14.41)

Esta igualdade é a lei dos cossenos, a qual é um caso particular


do Teorema de Pitágoras, aquele que se tem quando B̂ é reto.
Evidentemente, tem-se

a2 = b2 + c2 − 2bc · cos Â, c2 = a2 + b2 − 2ab · cos Ĉ. (14.42)

195
Fórmulas de Adição e
Leis Fundamentais

As mesmas guras nos dão, no primeiro caso:

h = c · sen B̂ = b · sen Ĉ,

logo
b c
= .
sen B̂ sen Ĉ
No segundo caso temos que

h = b · sen Ĉ,

e
h = c · sen(π − B̂) = c · sen B̂,

logo, novamente
b c
= .
sen B̂ sen Ĉ
Se tomarmos a altura baixada do vértice B sobre o lado AC , ob-
teremos, com o mesmo argumento, a relação

a c
= .
sen  sen Ĉ

Podemos então concluir que, em qualquer triângulo, tem-se

a b c
= = . (14.43)
sen  sen B̂ sen Ĉ

Esta é a lei dos senos. A interpretação geométrica para a razão


a/ sen  é que esta corresponde ao diâmetro do círculo circun-
scrito ao triângulo ABC . De fato, a perpendicular OP , baixada
do centro do círculo circunscrito sobre o lado BC é também me-
diana do triângulo isósceles OBC e bissetriz do ângulo C ÔB , que
é igual a 2Â. Logo C ÔP = Â e daí resulta que α/2 = r sen Â, ou
seja, a/ sen  = 2r = diâmetro do círculo circunscrito ao triângulo
ABC .

196
Matemática para o Ensino Médio I AULA
14

Figura 14.4: Interpretação geométrica da Lei do Senos.

As leis dos senos e cossenos permitem obter os seis elementos de


um triângulo (três ângulos e três lados) dados três deles, desde que
um seja lado.

Exemplo 14.3. Suponha que são dados os lados a e b e o ângulo


 de um triângulo. Usando a leis dos senos

a b b
= de onde se obtém sen B̂ = sen Â
sen  sen B̂ a

Suponhamos, a > b então segue que b


a sen  é um número positivo
menor que 1, logo existe um único ângulo B̂ , menor que dois retos,
cujo seno é igual a b
a sen Â. Em seguida, determina-se o ângulo C
pela igualdade  + B̂ + Ĉ = dois retos. Conhecendo Ĉ , usando a
lei dos cossenos, obtemos
q
c = a2 + b2 − 2ab cos Ĉ.

RESUMO


Nesta seção determinamos as fórmulas para determinar o seno e


o cosseno da soma e da diferença de dois arcos α e β , em função
do seno e cosseno destes dois arcos. Como vimos anteriormente,

197
Fórmulas de Adição e
Leis Fundamentais

através do seno e do cosseno poderemos determinar as demais


funções trigonométricas. A Utilidade destas relações está no fato
de que elas permitem que se expanda o número de arcos que se
podem obter o valor das funções trigonométricas, já que permitem
que se obtenham novos valores a partir de outros já conhecidos.
As fórmulas de adição são:

sen(α + β) = sen(α) · cos(β) + sen(β) · cos(α),


cos(α + β) = cos(α) · cos(β) − sen(α) · sen(β),
cos(α − β) = cos(α) · cos(β) + sen(α) · sen(β),
sen(α − β) = sen(α) · cos(β) − sen(β) · cos(α).

Já a Lei dos Senos, que relaciona os ângulos Â, B̂, Ĉ com os lados
a, b, c as medidas dos lados BC , AC e AB respecitavente, segundo
a fórmula
a b c
= = ,
sen  sen B̂ sen Ĉ
e a lei dos cossenos, onde são válidas as relações

a2 = b2 + c2 − 2bc · cos Â,


b2 = a2 + c2 − 2ac · cos B̂,
c2 = a2 + b2 − 2ab · cos Ĉ,

são muito importante para se determinar os seis elementos de um


triângulo, quando são dados três deles, sendo que ao menos um
deles é um lado.

ATIVIDADES


Atividade. 14.1. Prove que sen(α−β) = sen α cos β−sen β cos α.

198
Matemática para o Ensino Médio I AULA
Atividade. 14.2. Mostre que para todo x ∈ R temos que: 14
(a) sen(x) = sen(π − x) e cos(x) = − cos(π − x).
(b) sen(x) = − sen(x − π) e cos(x) = − cos(x − π).
(c) sen(x) = − sen(2π − x) e cos(x) = cos(2π − x).
(c) sen(x) = cos(π/2 − x) e cos(x) = sen(π/2 − x).

Atividade. 14.3. Prove as seguintes igualdades são verdadeiras


para todo x ∈ R:
(a) sen(3π/2 − x) = − cos(x).
(b) cos(3π/2 − x) = − sen(x).
(c) cos(3π/2 + x) = sen(x).
(d) sen(3π/2 + x) = − cos(x).

Atividade. 14.4. Provamos que , dados a, b ∈ R então

cos(a + b) = cos(a) cos(b) − sen(a) sen(b).

Usando esta fórmula , prove que:


tg(a)+tg(b)
(a) tg(a + b) = 1−tg(a) tg(b) .
tg(a)−tg(b)
(b) tg(a − b) = 1+tg(a) tg(b) .

(c) cot(a + b) = cotg(a) cotg(b)−1


cotg(a)+cotg(b) .

Atividade. 14.5. Calcule cotg(165o ), sec(255o ) e cossec(15o ).

Atividade. 14.6. Se a e b são ângulos agudos e positivos, mostrar


que

sen(a + b) < sen(a) + sen(b).

Atividade. 14.7. Provar que os ângulos internos A, B e C de


um triângulo não retângulo vericam a relação:

tg(A) + tg(B) + tg(C) = tg(A). tg(B). tg(C)

199
Fórmulas de Adição e
Leis Fundamentais

Atividade. 14.8. Mostre que para todo a 6= kπ


2 (k ∈ Z) temos
que 1
sen(a) = cotg(a/2) − cotg(a). Mostre ainda que

1 1 1 1
+ + +. . .+ = cotg(a/2)−cotg(2n a).
sen(a) sen(2a) sen(4a) sen(2n a)

Atividade. 14.9. Prove que valem as relações:


(a) cos(p) + cos(q) = 2. cos p+q cos p−q
 
2 2 .

(b) cos(p) − cos(q) = −2. sen p+q sen p−q


 
2 2 .

(c) sen(p) + sen(q) = 2. sen p+q cos p−q


 
2 2 .

(d) sen(p) − sen(q) = 2. sen p−q cos p+q


 
2 2 .
sen(p+q)
(e) tg(p) + tg(q) = cos(p) cos(q) .
sen(p−q)
(f) tg(p) − tg(q) = cos(p) cos(q) .

Atividade. 14.10. Trasnforme em um produto y = cos(9a) +


cos(5a) − cos(3a) − cos(a).

Atividade. 14.11. Demosntrar que se A, B e C são os ângulos


internos de um triângulo (qualquer ?) então vale a relação:
(a) sen(A) + sen(B) + sen(C) = 4 cos(A/2). cos(B/2). cos(C/2).
(b) cos(A)+cos(B)+cos(C) = 1+4 sen(A/2). sen(B/2). sen(C/2).
(c) sen(2A) + sen(2B) + sen(C) = 4 sen(A). sen(B). sen(C).

REFERÊNCIAS BIBLIOGRÁFICAS


CARMO, M.P., Trigonometria e Números Complexos, 1.ed., SBM,


1992.
IEZZI, Gelson., Fundamentos de Matemática Elementar: Trigonome-
tria, vol.3, 2.ed., Editora Atual, 1977.
Lima, E.L., A Matemática no Ensino Médio I, 5.ed., SMB, 2006.

200
AULA

15
Equações e
Inequações Trigonométricas

META:
Identicar e resolver equações e inequações trigonométricas funda-
mentais.
OBJETIVOS:
Ao m da aula os alunos deverão ser capazes de:
Identicar e resolver uma equação trigonométrica básica.
Identicar e resolver uma inequação trigonométrica.
Denir as funções trigonométricas Inversas das funções Seno, Cosseno
e Tangente.

PRÉ-REQUISITOS
Denição e propriedades das funções trigonométricas.
Equações e
Inequações Trigonométricas

15.1 Introdução

Nesta seção, vamos examinar algumas equações e inequações tri-


gonométricas. Elas aparecem naturalmente na solução de proble-
mas de Geometria quando a incógnita escolhida é um ângulo. Por
exemplo, se de um triângulo retângulo se conhece a hipotenusa a
e a soma dos catetos s, para calcular algum outro elemento dessa
gura, podemos colocar x para um dos ângulos. Teremos então
sen(x) + cos(x) = s/a, a qual é uma equação trigonométrica.

15.2 Equações Fundamentais

As equações fundamentais são sen x = cos a, cos x = cos a e tg x =


tg a. Vamos examinar cada uma separadamente.

15.2.1 sen x = sen a

Para que sen x = sen a é necessário e suciente que as extremidades


dos arcos x e a coincidam ou que sejam simétricas em relação
ao eixo das ordenadas. No primeiro caso x e a têm a mesma
imagem, ou seja, são côngruos, e no segundo caso os arcos x e a
Dados os arcos de medida são suplementares. Em outras palavras, as imagens de x e a no
α e β . Dizemos que α e β
são suplementares se α + círculo unitário, dadas respectivamente por P = E(a) e Q = E(x),
β = π . Dizemos que α e β
são replementares se α + estão sobre a reta r perpendicular ao eixo dos senos. Em resumo,
β = 2π .
a solução deste tipo de equação é dada por:

sen x = sen a ⇐⇒ x = a + 2kπ ou x = (π − a) + 2kπ

Exemplo 15.1. Seja m ∈ [−1, 1], então existe um único y no


intervalo − π2 ≤ y ≤ π
2 tal que sen y = m. Chamaremos este real y

202
Matemática para o Ensino Médio I AULA
15

Figura 15.1: Esquerda: Solução da equação sen x = sen a, no caso


em que a e x são suplementares. Direita: Função Inversa arcsen

de arcsen m (arco seno m), logo arcsen é a função inversa do seno


no intervalo [−π/2, π/2]. (Veja Figura 15.1)
Portanto

π π
y = arcsen(m) ⇐⇒ sen y = m e − ≤y≤ .
2 2

Por exemplo, arcsen(1) = π/2 e arcsen(1/2) = π/6.

15.2.2 cos x = cos a

Para que cos x = cos a é necessário e suciente que as extremidades


dos arcos x e a coincidam ou que sejam simétricas em relação ao
eixo dos cossenos. No primeiro caso x e a têm a mesma imagem,
ou seja, são côngruos, e no segundo caso os arcos x e a são re-
plementares. Em outras palavras, as imagens de x e a no círculo
unitário, dadas respectivamente por P = E(a) e Q = E(x), estão
sobre a reta r perpendicular ao eixo dos cossenos. Em resumo, a
solução deste tipo de equação é dada por:

cos x = cos a ⇐⇒ x = a + 2kπ ou x = −a + 2kπ

203
Equações e
Inequações Trigonométricas

Figura 15.2: Esquerda: Solução da equação cos x = cos a, no caso


em que a e x são replementares. Direita: A função inversa
arccos.

Exemplo 15.2. Se m ∈ [−1, 1], a função inversa do cosseno ,


arccos m, é denida como o único número real y do intervalo [0, π]
tal que cos y = m. Portanto

y = arccos(m) ⇐⇒ cos y = m e 0 ≤ y ≤ π.

15.2.3 tg x = tg a

Para que tg x = tg a com a 6= π/2+kπ , é necessário e suciente que


as extremidades dos arcos x e a coincidam ou que sejam simétricas
em relação à origem. Temos então x = a+kπ . Em outras palavras,
as imagens de x e a no círculo unitário, dadas respectivamente por
P = E(a) e Q = E(x), estão sobre a reta r que passa pela origem.
Em resumo, a solução deste tipo de equação é dada por:

tg x = tg a ⇐⇒ x = a + kπ

204
Matemática para o Ensino Médio I AULA
15

Figura 15.3: Esquerda: Solução da equação tg x = tg a. Direita:


A função inversa y = arctg(m).

Exemplo 15.3. A função inversa da tangente, arctg m, é denida


para todo real m como o único y do intervalo (−π/2, π/2) tal que
tg y = m.
Portanto

π π
y = arctg(m) ⇐⇒ tg y = m e − ≤y≤ .
2 2

15.2.4 A equação a sen x + b cos x = c

A equação a sen x + b cos x = c pode ser resolvida por três proces-


sos. Mostramos, por um método que consiste em dividir a equação

por r = a2 + b2 , que é diferente de zero. A equação toma a forma

a b c
sen x + cos x = .
r r r
a 2 b 2
Como = 1, existe um número real α tal que sen α = a/r
 
r + r

e cos α = b/r. Teremos então

c
sen α · sen x + cos α · cos x = ,
r

205
Equações e
Inequações Trigonométricas

ou seja,
cos(x − α) = c/r,

a qual é de fácil resolução.

Exemplo 15.4. Considere a equação


3 sen x − cos x = 1,


onde neste caso particular a = 3, b = −1 e c = 1. Assim

r = 3 + 1 = 2. Dividindo por 2 a equação obtemos

3 1 1
sen x − cos x = ,
2 2 2

Como cos π/6 = 2
3
e sen π/6 = 1/2, temos que a equação acima
pode ser reescrita na forma

π π 1
sen x · cos − sen · cos x = ,
6 6 2

ou
π 1
sen x − = .
6 2
Obtemos, como vimos anteriormente, que devemos ter x − π/6 =
π/6 + 2kπ ou x − π/6 = π/6 − π/6 + 2kπ , e as soluções de nossa
equação, são portanto

x = π/3 + 2kπ ou x = π + 2kπ.

15.3 Inequações Trigonométricas

As inequações trigonométricas podem ser reduzidas a uma das i-


nequações: sen x > m, sen x < m, cos x > m, cos x < m, tg x > m
e tg x < m, onde m é um número real dado. Tais inequações são
chamadas inequações fundamentais.

206
Matemática para o Ensino Médio I AULA
15.3.1 Inequação do tipo sen x > m 15
Seja P = E(x) a imagem de x no círculo unitário e seja r a reta que
passa por P e perpendicular ao eixo dos senos. Então as imagens
dos reais tais que sen x > m estão na intersecção do círculo unitário
com o semi-plano situado acima de r.

Figura 15.4: Esquema da solução da Inequação da forma sen x >


m.

Análogamente faz-se o estudo da inequação sen x < m.



Exemplo 15.5. Resolva a inequação 0 ≤ sen x < 2 .
3
Observamos
primeiramente que sen x = 0 implica que x = kπ . Então a solução
da equação 0 ≤ sen x, consiste em todos valores de x cuja imagem
no círculo unitário está acima da reta s, que neste caso é o eixo do

cossenos. Por outro lado, sen x = 2 ,
3
implica que uma solução é

x = π/3 + 2kπ . Logo a solução da inequação sen x < 2
3
consiste
em todos os valores de x cuja imagem no círculo unitário está
abaixo da reta s1 que é perpendicular ao eixo do senos e passa
pelo ponto P = E(π/3). Logo combinando estes dois resultados,

207
Equações e
Inequações Trigonométricas

obtemos

π 2π
0 + 2kπ ≤ x < + 2kπ ou + 2kπ < x ≤ π + 2kπ
3 3


Figura 15.5: Esquema da solução da Inequação 0 ≤ sen x < 2 .
3

15.3.2 Inequação do tipo tg x > m

Figura 15.6: Esquema da solução da Inequação da forma tg x > m.

208
Matemática para o Ensino Médio I AULA
Seja P = E(x) a imagem de x no círculo unitário e marcamos no 15
eixo das tangentes o ponto T tal que AT = m. Traçamos então
−→
a reta r = OT . As imagens dos x tais que tg x > m estão na
intersecção do círculo unitário com o ângulo rÔy .

Exemplo 15.6. Considere a inequação tg < 3. Sabemos que

tg π/3 = 3. Determinamos o ponto T no eixo das tangentes

correspondente ao valor 3. Fica entào determinada a reta r,
que passa por T e a origem. Assim analisando a Figura 15.7,
encontramos:

Figura 15.7: Esquema da solução da Inequação da forma tg x <



3.

0 + 2kπ ≤ x < π
3 + 2kπ ou π
2 + 2kπ < x < 4π
3 + 2kπ
ou 3π
2 + 2kπ < x < 2π + 2kπ.
(15.44)

RESUMO


209
Equações e
Inequações Trigonométricas

Vimos as seguintes equações trigonométricas:


(1)

sen x = sen a ⇐⇒ x = a + 2kπ ou x = (π − a) + 2kπ.

(2)
cos x = cos a ⇐⇒ x = a + 2kπ ou x = −a + 2kπ.

(3)
tg x = tg a ⇐⇒ x = a + kπ.

(4) Consideremos a equação a sen x + b cos x = c. Dena r = a2 + b2
e divida a equação a sen x + b cos x = c por r, onde obtemos

a b c
sen x + cos x = .
r r r
a 2 b 2
Como = 1, existe um número real α tal que sen α = a/r
 
r + r

e cos α = b/r. Teremos então

c
sen α · sen x + cos α · cos x = ,
r

ou seja,
cos(x − α) = c/r,

ATIVIDADES


Atividade. 15.1. Estudar a variação de sinal da função f : R −→


R, f (x) = cos(x) − sen(x).

210
Matemática para o Ensino Médio I AULA
Atividade. 15.2. Estudar a variação da função f : R −→ R, 15
f (x) = sen(2x) + cos(2x).

Atividade. 15.3. Resolva as equações trigonométricas:


(a) 2 cos2 (x) − 1 = sen(x).

(b) sen(x − π/3) = 3/2.
(c) 2 sen(x) − cossec(x) = 1.
(d) sen(2x) = sen(x).
(e) cos(3x) − cos(x) = 0

(f) sen(x) − 3 cos(x) = 0.
(g) tg(x) + cotg(x) = 2.

(h) 3 cos(x) + sen(x) = 1

Atividade. 15.4. Determinar os ângulos internos de um triângulo


ABC sabendo-se que

cos(A + B) = 1/2, sen(B + C) = 1/2.

Atividade. 15.5. Resolva as inequações trigonométricas:



(a) sen(x) ≤ − 2 .
3

(b) | sen(x)| ≤ 1/2.


(c) 2 sen2 (x) ≤ sen(x).
(d) cos(2x) + cos(x) ≤ −1.
(e) cos(2x) ≥ cos(x).
cos(x)
(f) cos(2x) ≤ 1.
(g) | tg(x)| ≤ 1.

Atividade. 15.6. Os lados de um triângulo são dados pelas ex-


pressões a = x2 + x + 1, b = 2x + 1 e c = x2 − 1. Demonstrar que
um dos ângulos e 120o .

Provar que num triângulo ABC retângulo em A, vale a relação


(a − b)2 = c2 − 4absen2 (Ĉ/2).

211
Equações e
Inequações Trigonométricas

Atividade. 15.7. Qual é a relação entre os lados a, b e c de um


triângulo ABC para que se tenha:( Suponha que a ≥ b e a ≥ c)
(a) ABC retângulo? (R:a2 = b2 + c2 )
(b) ABC acutângulo? (R:a2 < b2 + c2 )
(c) ABC obtusângulo? (R:a2 > b2 + c2 )

REFERÊNCIAS BIBLIOGRÁFICAS


CARMO, M.P., Trigonometria e Números Complexos, 1.ed., SBM,


1992.
IEZZI, Gelson., Fundamentos de Matemática Elementar: Trigonome-
tria, vol.3, 2.ed., Editora Atual, 1977.
Lima, E.L., A Matemática no Ensino Médio I, 5.ed., SMB, 2006.

212

Você também pode gostar